Download as pdf or txt
Download as pdf or txt
You are on page 1of 150

Multiple-Choice Questions in

Medical Physiology
For Postgraduate Medical Entrance Examinations
[Revised May 2013]

E.S.Prakash, MBBS, MD
_____________________________________________________________________________________
Written and Published May 2013 by E.S.Prakash.

Copyright © 2013, all rights reserved by E.S.Prakash. Except as permitted under the United States
Copyright Act of 1976, no part of this publication may be reproduced or distributed in any form or by any
means or stored in a database or retrieval system without prior written permission from the author.

This version has not yet been peer reviewed.

150 Pages

Please avoid using previous editions of this book.

Disclaimer: Science and medicine constantly change and while I have tried to check information presented
here with sources known to be reliable, I cannot guarantee that this book is error free. I encourage you to
crosscheck information contained herein with other sources. This book is not written to be used to directly
guide the care of patients. Therefore, I cannot accept any responsibility for any consequences that may arise
from the use of information contained here in the care of patients – E.S.Prakash

1
E.S.Prakash. MCQs in Medical Physiology, May 2013
2 minutes please
Think of this book as a tool to help you assess how well you have learnt medical physiology. It was written primarily
for use in revision courses for doctors who are preparing for competitive postgraduate (PG) medical entrance
examinations in India, but undergraduate medical students and postgraduate physiology students may find this helpful.

The intent here is to use multiple-choice questions (MCQ) as a means to help the reader revise some key facts, test
understanding of concepts and the ability to apply them, and thereby improve one’s understanding of human
physiology and its relevance to clinical medicine. Thus, besides the traditional select the single best response type of
MCQ which is the question type used in most entrance exams, I have frequently included questions solely intended for
learning and practice and so these may not be equivalent in standard to questions commonly appearing in entrance
examinations. Also, the difficulty level of questions vary across entrance exams and I’ve attempted to simulate it here.

As MCQs with multiple correct answers enable more material to be revised with fewer questions, I have also
frequently used such questions; further, this helps us get into and sustain the important habit of carefully reading all
options in a question. Some entrance exams use the select all correct answers (multiple true-false) type of MCQ, so
some practice with this type of question is needed.

In general, a significant fraction of the questions that I have seen appear on PG entrance examinations test conceptual
understanding and meaningful learning. However, some test knowledge of facts that are not of significance to a
primary care physician, and while this can be partly rationalized, an undesirable backwash effect of this on students is
it tends to encourage rote learning over indepth learning of important concepts. I have focused on the immediate
‘learning needs’ of the target audience, as well as to exemplify the relevance of medical physiology in clinical
practice.

For the purpose of preparing and revising for PG entrance examinations, I recommend Ganong’s Review of Medical
Physiology (abbreviated as WFG in the rest of this book), published by Mc Graw Hill.

For more practice questions in physiology, I recommend the following sources:


• Self-Study Questions in WFG
• National Medical Series for Independent Study. Physiology. Eds. Bullock J, Boyle J, Wang MB. Williams &
Wilkins, 2001 (or a newer edition)
• Ryan JP and Tuma RF. PreTest – Physiology, 13th ed. Mc Graw Hill.
• Questions from previous PG entrance exams, if they are publicly available.

Clarifications, Follow-up Notes, Some more practice questions: I’ll post them on my website at
http://esprakash.wordpress.com/mcqmedphy/

Images and Schematic Diagrams accompanying this book: You can download them from the above webpage; these
figures render better on PowerPoint Slides rather than on a 2-column Word Document.

I’ve greatly benefitted from review of this book by students who used it in the past, and I welcome suggestions for
improving this book. If you spot an error, please let me know. I dedicate this book to the memory of Dr William F
Ganong.

E.S.Prakash, MBBS, MD
Associate Professor of Physiology
Mercer University School of Medicine,
Macon, Georgia, USA
E-mail: dresprakash@gmail.com
Website: http://esprakash.wordpress.com
Editor & Publisher, Medical Physiology Online [ISSN 1985-4811]
http://www.medicalphysiologyonline.org

2
E.S.Prakash. MCQs in Medical Physiology, May 2013
Table of Contents
Topic Page
2 minutes please 2
General Physiology 4
Cell Physiology; Intercellular Communication 11
Physiology of Nerve and Muscle 14
Central Nervous System 24
Endocrinology and Reproduction 39
Gastrointestinal Physiology 48
Circulating Body Fluids 52
Cardiovascular Physiology 56
Pulmonary Physiology 68
Renal and Acid-Base Physiology 82
Critical Care Physiology 93

Self-Scorers
1 – General Physiology 98
2 – Physiology of Nerve and Muscle 99
3 – Central Nervous System 101
4 – Endocrinology and Reproduction 104
5 – Gastrointestinal Physiology 108
6 – Cardiovascular Physiology 110
7 – Pulmonary Physiology 112
8 – Renal and Acid-Base Physiology 114

Blast from the Past 116


More Questions for Self-Study 131

3
E.S.Prakash. MCQs in Medical Physiology, May 2013
1. General Physiology 7. In a healthy adult weighing 70 kg, plasma
volume was estimated to be 3000 ml. His
In each of the following questions, select the hematocrit was 40%. His blood volume would be
single best response unless instructed otherwise. about:
A. 5000 ml
1. ECF volume is determined by: B. 5200 ml
A. plasma [Na] C. 5400 ml
B. plasma protein concentration D. 5600 ml
C. the amount of sodium in the ECF
8. The water content of lean body mass is:
2. The chief determinant of plasma osmolality is A. 30 ml/100 g
A. plasma [Na] B. 50 ml/100 g
B. plasma [glucose] C. 70 ml/100 g
C. blood [urea] D. highly variable
D. plasma [albumin]
9. The following values are obtained on a sample
3. The volume of distribution of intravenously of plasma from a child that has clinical evidence
administered sucrose in a healthy 70-kg man is of dehydration.
about:
A. 3.5 liters Plasma [Na] = 135 mmol/L
B. 10.5 liters Plasma [glucose] = 400 mg/dL
C. 14 liters Blood urea nitrogen = 100 mg/dL.
D. 28 liters
The osmolality of plasma is expected to be close
4. Which of the following markers administered to:
intravenously distributes exclusively in A. 290 mOsm/kg H2O
intracellular fluid? B. 310 mOsm/ kg H2O
A. Evans blue dye C. 330 mOsm/kg H2O
B. Heavy water D. 350 mOsm/kg H2O
C. Sucrose
D. None of the above 10. What percentage of osmolality of plasma in a
healthy, well hydrated individual is attributable to
5. Which compartment does the term “sucrose sodium and its accompanying anions?
space” refer to? A. 30 %
A. Extracellular fluid (ECF) B. 50 %
B. Interstitial fluid (ISF) C. 70 %
C. Intracellular fluid (ICF) D. 90 %
D. Plasma
11. Which of the following contributes least to the
6. 100 mg of sucrose is injected intravenously into osmolality of plasma?
an adult male weighing 70 kg. The plasma A. Glucose
concentration of sucrose after mixing is 0.01 B. Proteins
mg/ml. If 5 mg of sucrose has been metabolized C. Sodium
during this period, the ECF volume in this D. Urea
individual is approximately:
A. 6 liters 12. In the steady state, the value of which of the
B. 9.5 liters following variables is the same in ICF and ECF?
C. 14 liters A. pH
D. 17.5 liters B. osmolality
C. concentration of proteins
4
E.S.Prakash. MCQs in Medical Physiology, May 2013
D. number of osmoles 19. Which of the following exerts the greatest
osmotic effect on a mole-mole basis?
13. The body fluid compartment that contains A. Dextran
more osmotically active particles (in relation to B. Hydroxyethyl starch
other fluid compartments in the same individual) C. Albumin
is: D. Fibrinogen
A. intracellular fluid
B. plasma 20. Connexins do not allow the passage of:
C. interstitial fluid A. polypeptides
B. Na ions
14. Normal red blood cells from a healthy C. Ca ions
individual were placed in each of the following D. inositol trisphosphate
solutions and observations made after 1 hour. E. amino acids
Cells would have most likely have lysed in red
blood cells placed in: 21. Of the following substances, the lipid bilayer
A. 0.3% NaCl per se (i.e., without proteins) is most permeable
B. 0.9% NaCl to:
C. 1.2% NaCl A. sodium ions
B. urea
15. Which of the following solutions is C. glucose
hypertonic? D. water
A. 0.9 % NaCl
B. 5% dextrose 22. The rate of diffusion of a substance across the
C. 20% mannitol cell membrane is inversely proportional to:
D. Distilled water A. concentration gradient for the substance
B. diffusion coefficient
16. Two liters of 0.9% NaCl is administered to a C. surface area available for diffusion
12-year old boy with moderate isotonic D. thickness of the membrane
dehydration. What is the expected change in ICF
volume after NaCl administration? 23. Which one of the following is an example of
A. No change passive transport?
B. Increase by 0.5 liter A. Calcium efflux by calcium pump
C. Increase by 2 liters B. Na-Ca exchanger
D. Decrease by 0.5 liter C. Potassium efflux through potassium leak
E. Decrease by 2 liters channels
D. Calcium sequestration in sarcoplasmic
17. ICF volume decreases when dehydration is: reticulum
A. isotonic
B. hypertonic 24. Most of the ATP generated in nerve cells is
C. hypotonic utilized to energize the:
A. Na-Ca exchanger
18. If the intent is to replenish total body water in B. H-ATPase in the cell membrane
a dehydrated individual which one of the C. Na-K ATPase
following should be administered intravenously? D. synthesis of proteins
A. 0.9% NaCl
B. 5% dextrose solution 25. Which of the following is incorrectly
C. Albumin matched?
D. 10% glucose solution A. Na-K ATPase: antiport
E. Distilled water B. H-ATPase: uniport
C. SGLT: symport
5
E.S.Prakash. MCQs in Medical Physiology, May 2013
D. Ca-ATPase: biport Thus, urea concentration of 60 mg/dL corresponds
to a [BUN] of 28 mg/dL.
26. The core body temperature of an experimental
animal is raised from 98ºF to 106ºF by passive
heating. Eventually, it dropped back to 99ºF. Normally, plasma osmolality is chiefly due to Na
What is the gain of the temperature regulation and its accompanying anions Cl and HCO3.
system in this instance? Normally, serum osmolality ranges from 280–295
A. Zero mOsm/Kg H2O.
B. One
C. -7 4-6. Intravenously administered sucrose
D. Infinity distributes throughout ECF (plasma + interstitial
fluid). ECF volume in a 70-kg healthy adult is
27. Which of the following is not true about about 14 L (20% of body weight). The volume of
negative feedback control systems? interstitial fluid is about 10.5 L (75% of ECF
A. Output is one of the inputs to the system volume) and plasma volume is about 3.5L (25%
B. It is based on a ‘set-point’ for the controlled of ECF volume). Heavy water distributes
variable. throughout body water. Sucrose, inulin and
C. The system corrects “errors” mannitol distribute exclusively in the ECF. Evans
D. The ‘set point’ of the system cannot be blue dye stays in the plasma.
changed by inputs other than the controlled
variable Typical values in a healthy adult male weighing
70 kg are as given below:
Answers: General Physiology Compartment Volume Marker
1C 2A 3C 4D 5A Total body water 42 L D2 O
6B 7A 8C 9C 10D ICF 28 L -
11B 12B 13A 14A 15C ECF 14 L Sucrose
16A 17B 18B 19C 20A Interstitial fluid 10.5 L -
21D 22D 23C 24C 25D Plasma 3.5 L Evans blue
26C 27D
Indicator-dilution principle:
Answer Explanations: Volume of distribution of the indicator equals the
1. Children and adults have the same amount injected (A) divided by the concentration
concentration of sodium in plasma. Yet their ECF (C) in plasma (of the indicator) after mixing.
volumes are greatly different. Thus, ECF volume
is proportional to the amount of sodium in ECF. In the above example,
First, sucrose distributes throughout ECF.
2. The concentration of sodium in ECF is Amount of sucrose injected = 100 mg
quantitatively the most important determinant of Amount metabolized = 5 mg
plasma osmolality. Amount remaining in ECF = 95 mg
Concentration after mixing = 0.01 mg/ml
Plasma osmolality (mosm/Kg H2O) = Volume of distribution of sucrose
2 [Na+] + [glucose] / 20 + [BUN] ×18/50 = 95 mg / 0.01 mg/ml = 9500 ml = 9.5 L
(mmol/L) (mg/dL) (mg/dL)
7. Blood volume
Blood urea and urea nitrogen: = plasma volume × [100 / (100–Hct)]
The formula of urea is NH2CONH2
Molar mass of urea is 60 g; each molecule of urea 8. Body mass = fat mass + lean body mass.
has 2 nitrogen atoms. The water content of lean body mass (fat-free
The mass of nitrogen in urea is 2 × 14 = 28 g mass) is relatively constant and is about 70 ml/100
g. For example, in an individual weighing 70 kg
6
E.S.Prakash. MCQs in Medical Physiology, May 2013
and whose total body water is measured to be 42 Osmotic pressure is the pressure that would be
L (= 42 kg), lean body mass = 42 / 0.7 = 60 kg, required to stop water flux (osmosis) across a
and fat mass is 10 kg. Fat is relatively anhydrous. semi permeable membrane.
Body fat % = 10/70 × 100 = 14%. Body fat
percentage is greater in women compared to men. Osmotic pressure P = CRT (Van’t Hoff equation),
where,
12. ICF is much more acidic than ECF. For C = concentration of osmoles
example, in muscle cells the pH is about 6.8. The R = a constant
steady state osmolality (i.e., concentration of T = temperature in Kelvin
osmotically active particles) of all body fluid
compartments must be the same. The fact that, in In the steady state, the osmolality of all body
a healthy adult, ICF volume is twice as large as fluids is identical; that is, osmotic pressure across
ECF volume should indicate that the absolute the plasma membranes of cells in the steady state
number of osmoles is much greater in the ICF. is zero and there would be no net water flux
(osmosis) across the cell membrane.
13-16. What is osmosis?
Osmosis is the movement of water across a Osmolality is a colligative property that depends
semipermeable membrane permeable to water but upon the number of solute particles, and not the
not to solutes, from a solution with lesser size of the particles. To illustrate, the contribution
concentration of osmoles to a solution with a of 1 Na ion and 1 albumin molecule toward
greater concentration of osmoles. This continues osmolality of plasma is the same. Since the molar
until osmotic equilibrium (i.e., the osmolality of concentration of proteins in plasma is very low
either compartment is equal) is attained. (60 g/L) compared to that of Na (140 mM) and Cl
(100 mM), one can understand why plasma
What is an osmole? proteins contribute very little to the osmolality of
An osmole (effective osmole) is an osmotically plasma compared to Na and its accompanying
active particle potentially capable of causing ions.
osmosis. Examples include sodium ion, chloride
ion, protein anions, and phosphate ion. A solute to Colloid osmotic pressure of plasma: While
which the cell membrane is not as freely proteins are present in plasma at a concentration
permeable as it is to water will function as an of 60-80 g/L, they are not normally present in
effective osmole. For example, in comparison to significant concentrations in the interstitium.
water, the cell membrane is relatively Thus, the osmotic pressure of plasma proteins
impermeable to sodium and chloride ions, (called colloid osmotic pressure or oncotic
mannitol. The amount of ions crossing the cell pressure) is much greater than the osmotic
membrane through channels and transporters is pressure of proteins in the interstitium. This
much smaller relative to osmotically driven water oncotic pressure gradient across the capillary
fluxes. restrains fluid filtration and favors reabsorption of
fluid into the capillary.
What is an ineffective osmole?
If the cell membrane is permeable to a solute (for Hemolysis begins when normocytes are placed in
example, urea), the substance will move across 0.5% NaCl and is complete in 0.3% NaCl. In
the membrane until its concentration is exactly the contrast, when RBCs are placed in hypertonic
same on both sides of the membrane. In such an saline, they lose water and diminish in size.
instance, osmosis (net movement of water) does
not occur. However, note that urea does indeed Why is 0.9% NaCl called an isotonic solution?
function as an effective osmole in the renal 0.9% (precisely 0.85%) NaCl has the same
medullary interstitium. osmolality as normal human plasma (about 290
mOsm/kg H2O, please see the calculation below);
further, when it is infused into a normal human
7
E.S.Prakash. MCQs in Medical Physiology, May 2013
(i.e., one with an ECF osmolality of 290
mOsm/kg H2O), it does not cause any change in Is 5% dextrose isotonic or hypotonic?
the steady state volume of red blood cells or other Molar mass of dextrose (D-glucose, C6H12O6) is
cells; i.e. because it does not change the steady 180 g
state osmolality of normal human plasma, it is an 5% solution contains 50 grams of dextrose per
isotonic solution. liter of the solution
50g = 50/180 mol = 0.277 mol = 277 mmol = 277
Calculate the osmolality of 0.85% NaCl. mOsm/L ~ 270–280 mOsm/kg H2O
0.85% NaCl contains 0.85 g of NaCl per deciliter
of the solution. As 5% dextrose has approximately the same
= 8.5 g/ L of the solution osmolality as normal human plasma, it is an
Molar mass of NaCl = 58.5 g isosmotic solution. However, when dextrose is
1 mol of NaCl = 58.5 g infused, it is metabolized and the net effect (over a
8.5 g = 8.5/58.5 mol = 0.145 mol = 145 mmol/L period of time, especially when large volumes are
Each Na is accompanied by 1 chloride ion infused) is that of adding water to plasma. This
Therefore, total concentration of osmoles “excess” water can enter cells. Thus, 5% dextrose
(osmotically active particles) = 2 × 145 = 290 is of value in replenishing ICF volume in
mOsm/L intracellular dehydration. Also, note 5% dextrose
is hypotonic because it dilutes plasma in the
16-17. ECF volume changes steady state.
When the terms dehydration and overhydration
are used without further qualification, they are In well hydrated individuals, the hypo-osmolality
typically used to refer to ECF volume contraction that occurs when hypotonic dextrose solutions are
and ECF volume expansion respectively. The infused is sensed by osmoreceptors and secretion
term dehydration can be used to refer to a of antidiuretic hormone is promptly inhibited to
reduction in total body water as well. However, excrete ‘excess’ water.
hypovolemia and dehydration are not
synonymous. Hypovolemia specifically refers to a To summarize, tonicity of a solution refers to the
reduction in blood volume. Blood volume is more effect of an administered solution on the steady
critically regulated than ECF volume. state osmolality of normal human plasma.

Classification of dehydration*: 19. Starling’s law of filtration


Type Example of a cause
Isosmotic Blood loss
Hyperosmotic ADH deficiency
Starling described forces that affect fluid flux
Hypoosmotic Adrenocortical insufficiency across capillaries.
*Overhydration is classified likewise.
Fluid movement = Kf [(Pc + πi) – (Pi + πc)],
Changes in ICF volume in various types of where,
dehydration: Kf = capillary filtration coefficient
Type ECF ICF Pc = hydrostatic pressure in the capillaries
volume volume
Isosmotic Decreases No change
Pi = hydrostatic pressure in the interstitium
Hyperosmotic Decreases Decreases πc = capillary colloid osmotic pressure
Hypoosmotic Decreases Increases πi = colloid osmotic pressure in the interstitium

18. The key phrase here is ‘body water’ which Normally, Pc is the principal force favoring
includes ICF as well as ECF. In the steady state, filtration. The osmotic pressure of plasma is
hypertonic dehydration is associated with a normally about 25 mm Hg higher relative to the
reduction in ECF as well as ICF volume. So osmotic pressure of interstitium because ‘plasma’
therapy must be aimed at replenishing total body proteins (colloids) are limited to plasma; proteins
water, not merely ECF volume.
8
E.S.Prakash. MCQs in Medical Physiology, May 2013
in plasma restrain fluid filtration into the understanding but there is some risk of teleologic
interstitium. fallacies.

Albumin is quantitatively the most important Take temperature regulation as an example. The
contributor to the colloid osmotic pressure of ‘controller’ is the hypothalamus. The fact that a
plasma. 1 g of albumin in 100 ml of plasma exerts rise in body temperature leads to a fall in
an osmotic effect of 6 mm Hg; the same temperature back toward 98.6ºF suggests that
concentration of globulins will exert a pressure of temperature is the controlled variable, and this
only about 1.5 mm Hg. operates as a negative feedback control system.
This means that there must be a ‘sensor’ for
22. Fick’s law of diffusion: temperature. The hypothalamus is in fact able to
Diffusion rate (J) = DA dc/dx sense core body temperature.
D: Diffusion coefficient
A: Area available for diffusion When there is an infection, often the release of
dc: concentration gradient cytokines as part of the immunologic response act
dx: thickness of membrane on the brain to raise the ‘set-point’ of the
temperature regulation system to a higher level so
The diffusion coefficient is affected by factors that a higher than normal body temperature is
such as temperature, and the permeability of the maintained and results in fever. The control
membrane to the molecule/ion in question. system is then said to be ‘reset’ to a higher
Permeability of a membrane to an ion/molecule is operating level. However, once the infection and
in turn affected by the number of ion channels or the immune response resolve and the
transporter molecules available to transport the concentration of cytokines and prostaglandins that
species in question. reset the temperature set point upward resolve, the
set point returns to 98.6ºF indicating that resetting
23. Ion flux through ion channels is a passive of a control system is not necessarily permanent.
process; i.e. it occurs down a concentration
gradient and requires no input of free energy. ***************************************
Supplement: Classification of mechanism of
24. A large fraction of ATP (nearly 70%) transport across cell membranes:
synthesized in neurons is used to energize the Na-
K ATPase and maintain ion gradients across the Mechanism Examples
nerve cell membrane. Simple diffusion Diffusion of oxygen,
carbon dioxide, anesthetic
gases, nitric oxide through
25. Ca-ATPase is a uniport because it transports lipid layer
one species. It is a primary active transport Facilitated diffusion Glucose entry into cells
process as it uses ATP as the source of energy. through GLUT 1-5;
passage of ions through
ion channels*
26. Gain = correction / error; in this example, the Primary active Na-K ATPase, H-K
correction is 7 degrees i.e. from 106ºF down to 99 transport ATPase, Ca-ATPase, H-
ºF, and the error (deviation from the original value ATPase
of 98ºF) is 1ºF. The gain of a negative feedback Secondary active Na-glucose cotransporter
control system is negative. If the error is zero, transport (SGLT), Na-amino acid
cotransport, Na-H
gain is infinite. Guyton and colleagues suggested exchanger, Na-Ca
that the renal ‘pressure-natriuresis’ mechanism for exchanger
controlling body fluid volumes has infinite gain.
Simple diffusion, by definition, is diffusion
27. As far as biological systems are concerned, the occurring through the lipid bilayer. Facilitated
application of control system theory is a means to diffusion occurs through protein molecules in the
cell membrane (ion channels or transport
9
E.S.Prakash. MCQs in Medical Physiology, May 2013
proteins). Some authors consider ion flux through
ion channels also as an instance of “simple Name Definition Example
diffusion” – this distinction is a matter of taste. Uniport 1 species Ca-ATPase,
transported GLUT*
Symport 2 species Na-glucose
By definition, a primary active transport (also cotransport) transported cotransport,
process is driven by hydrolysis of ATP. in the Na-K-2Cl
same cotransport
Notes about the Na-K ATPase: direction
• It is a primary active transport process Antiport (also 2 species Na-K
• The pump is present in the cell membrane countertransport) transported ATPase,
in opposite Na-H
• It is ubiquitous i.e., present in all cells directions exchanger,
• It pumps 3 Na out of the cell and 2 K ions in. Cl-HCO3
Thus it makes the inside of the cell negative exchanger.
with respect to exterior; i.e. it contributes to a *GLUT – glucose transporter
small extent (about 4 mV) to the genesis of
RMP. Carrier mediated transport – this term refers to
• It plays an important role in maintaining cell transport processes in which the transport species
volume. If the pump is inhibited as can happen physically attaches to a carrier molecule, and is
when ATP is limiting or pharmacologically carried by it. Carrier mediated transport processes
(with digoxin) intracellular Na increases also can be active or passive. Na-K ATPase (an active
increasing the size of cells. transport process) and glucose transport via
• Digoxin, a cardiac glycoside inhibits this GLUT (a passive transport process) are both
pump. Inhibition of the pump leads to an examples of carrier-mediated transport.
increase in cytosolic Ca and this augments the
force of contraction of cardiac muscle cells. Different authors use the word transport in this
• About 70% of the ATP generated in nerve context differently. Some do not use the word
cells is used to energize the Na-K ATPase. transport to refer to ion movement through ion
channels; for them transport entails physical
A secondary active transport process utilizes an attachment of the transported species to the
ion gradient as a source of energy rather than transporter molecule.
ATP. The Na-Glucose cotransporter (SGLT)
utilizes the energy of the Na gradient (Na battery) Other modes of transport: Exocytosis,
to drive the uphill transport of glucose from ECF endocytosis, transcytosis (vesicular transport).
to ICF. Note that the species that is actively Exocytosis is triggered by a rise in intracellular
transported by SGLT is glucose. The Na-amino calcium. Proteins injected into the circulation
acid symporter is similar. The term ‘secondary’ often have been endocytosed into vesicles by
refers to the fact that the energy source (the endothelial cells to be exocytosed as vesicles into
sodium ion gradient), which drives this process, the interstitium. This process called transcytosis
depends upon normal operation of a primary or vesicular transport requires an input of free
active transport process – the Na-K pump that energy.
generates a Na ion gradient. If the Na-K pump *************************************
fails due to lack of ATP or any other reason, then,
the Na gradient will be gradually reduced and all
secondary active transport processes powered by
the Na gradient will also be affected.

Nomenclature of transporters based on the


direction of movement and the number of
species of transported. Note: this is not a
classification of mechanism of transport.
10
E.S.Prakash. MCQs in Medical Physiology, May 2013
2. Cell Physiology; Intercellular toxin
Communication Forskolin Directly activates adenylate
cyclase
Types of intercellular communication:
Type Mediators
Neurocrine Neurotransmitters acting across
Some notes on G-proteins:
synapses 1. They have 3 subunits (α, β, γ);
Endocrine Hormones acting on target cells 2. Each subunit is distinct (heterotrimeric)
located at distant sites 3. They are guanylyl nucleotide (GTP / GDP)
Paracrine Hormones acting locally without binding proteins
entering the general circulation 4. The α subunit has inherent GTPase activity
Gap Cells are electrically coupled by 5. The β and γ subunits are coupled to effectors
junctions gap junctions which allow passage molecules like adenylyl cyclase, ion channels.
of ions and small molecules 6. G proteins couple hormone binding to the
Juxtacrine Cells linked to adjacent cells receptor to an intracellular signaling pathway.
through cell adhesion molecules In essence, they are signal coupling proteins.
and cell adhesion molecule 7. Hormone receptors that are coupled to G-
receptors in extracellular matrix proteins are called G-protein coupled
receptors (GPCR)
Components of signal transduction pathways and
their functions: 8. GPCR have seven transmembrane domains;
Components Functions thus they are often called seven-helix
Hormone / Signal receptors or serpentine receptors
neurotransmitter 9. There are several types of G proteins; Gs is
Receptor Recognizes the signal stimulatory G protein and it activates adenylyl
Signal coupling Coupling ligand binding cyclase; Gi is inhibitory G protein and it
proteins to intracellular signaling inhibits adenylyl cyclase; activation of Gq
Second Intracellular molecules leads to activation of the membrane bound
messengers that amplify extracellular enzyme phospholipase C.
hormonal (first 10. Other families of proteins such as ras
messenger) signals resemble G-proteins in structure and function.
several fold 11. Mutations in G proteins are implicated in the
Effector Enzymes, channels,
development of cancer.
molecules transporters, that mediate
the physiologic effects of
the hormone Select the single best response.

Signal transduction cascades and second 1. How many transmembrane domains do G-


messengers: protein coupled receptors have?
Signal transduction Second messenger(s) A. 2
cascades B. 3
Adenylate cyclase cAMP C. 7
cascade D. 12
Phosphatidyl inositol Inositol trisphosphate
pathway (IP3), DAG, and Ca 2. How many subunits do G proteins have?
Guanylyl cyclase cGMP A. One
Hormones acting upon Hormone-receptor B. Two
intracellular receptors complex
C. Three
Drugs / toxins used in G-protein research:
D. Four
Drug/toxin Mechanism of action
Cholera toxin ADP-ribosylation of α subunit 3. Which G protein activates adenylyl cyclase?
of Gs, activation of adenylate A. Gs
cyclase B. Gq
Pertussis Inhibition of α subunit of Gi C. Gi
11
E.S.Prakash. MCQs in Medical Physiology, May 2013
D. Gl 11. Which one of the following stimulates the
release of calcium from the endoplasmic
4. Which subunit of G protein has intrinsic reticulum?
GTPase activity? A. IP3
A. Alpha B. PTH
B. Beta C. 1, 25 (OH)2 – D3
C. Gamma D. DAG
D. Delta
12. Protein kinase C is activated by:
5. Which of the following hormones does not act A. IP3
via a G-protein coupled receptor? B. diacylglycerol (DAG)
A. TRH C. cAMP
B. Angiotensin II D. guanylyl cyclase
C. ADH
D. Thyroxine 13. All of the following chemical messengers act
via receptor tyrosine kinases except:
6. Which toxin inhibits the function of Gi? A. insulin
A. Cholera toxin B. ANP
B. Pertussis toxin C. epidermal growth factor
C. Forskolin D. platelet derived growth factor
D. Saxitoxin
14. Guanylyl cyclases are activated by:
7. Which receptor does not span the cell A. CO, NO and ANP
membrane seven times? B. NO, ANP and ET1
A. Rhodopsin C. AII, NO and ANP
B. Beta-adrenergic receptor D. ANP, ET1 and AII
C. Insulin receptor
D. M2 receptor 15. Which of the following hormones mediate(s)
its effects by activating nuclear / cytosolic
8. The second messengers in the phosphatidyl receptors? (select all correct answers)
inositol cascade are: A. Thyroxine
A. IP3, DAG and calcium B. Retinoic acid
B. protein kinases A (PKA) and C C. Estradiol
C. cAMP and cAMP dependent PKA D. Progesterone
D. cGMP and calcium E. Aldosterone
F. Cortisol
9. Most of the calcium in the endoplasmic G. 1, 25 (OH)2 D3
reticulum (ER) is sequestered by:
A. calmodulin 16. Which of the following is matched
B. caldesmin incorrectly?
C. calbindin A. Microfilaments: actin, myosin
D. calsequestrin B. Intermediate filaments: vimentin, keratin
C. Microtubules: clathrin
10. Cytosolic calcium concentration in unexcited D. Cytoskeleton: spectrin, ankyrin
cells is about:
A. 2.5 mM 17. Which intercellular junctions directly allow
B. 10 mM the passage of small molecules and ions between
C. 150 µM the cytosol of one cell and its neighbor without
D. 100 nM movement into interstitial fluid?
A. Gap junctions
12
E.S.Prakash. MCQs in Medical Physiology, May 2013
B. Focal adhesions ventricle of the heart contracts as one unit because
C. Zonula occludens gap junctions electrically couple all muscle fibers
D. Desmosomes in the ventricle. However, connexins do not allow
large molecules such as proteins to pass between
18. Which of the following statements regarding cells.
regulation of cell cycle is incorrect?
A. This serves to regulate transition of the cell 18. Deficiency of the tumor suppressor protein
from one phase of the cell cycle to the next. p53 allows entry into M phase and facilitates the
B. At restriction points, cyclins always promote growth of tumors, and a number of cancers are
transition from G0 to S phase. associated with deficiency of p53.
C. Defects in DNA synthesis normally allow
progression from G2 to M phase. ********************************************
D. Deficiency of p53 allows progression to M
phase.

Answers: Cell Physiology; Intercellular


Communication
1C 2C 3A 4A 5D
6B 7C 8A 9D 10D
11A 12B 13B 14A 15all
16C 17A 18C

Explanations:
10. ECF [Ca2+] = 2.5 mM
Cytosolic [Ca2+] in unexcited cells = 100 nM
ECF [Ca2+] / cytosolic [Ca2+] = 25000
A transient increase in intracellular calcium is
used as a signal to activate or inactivate
intracellular processes such as exocytosis, muscle
contraction. [Ca2+]i is quickly normalized by the
sarcoplasmic-endoplasmic reticulum calcium
pump (SERCA pump) and a Na-Ca antiporter
(also called the Na-Ca exchanger, NCX) in the
cell membrane. Sustained increases in
intracellular calcium trigger apoptosis,
culminating in cell death.

16. Microtubules have a 9 + 2 fibrillar structure.


They are made up of tubulin, dynein, and kinesin.
The mitotic spindle is made up of microtubules.
Colchicine arrests cells in metaphase by inhibiting
polymerization of microtubules.

17. Gap junctions (electrical synapses; connexons)


are made up of a protein called connexin; these
are large diameter channels that allow the passage
of ions and other small molecules such as amino
acids and glucose. Thus, they electrically couple
adjacent cells. They are found in visceral smooth
muscle and heart and between some neurons. The
13
E.S.Prakash. MCQs in Medical Physiology, May 2013
3. Physiology of nerve and muscle cells ENa = - 61 log [Na]i / [Na]o
= - 61 log [10/140] = + 61 mV
Resting membrane potential (RMP) refers to
the transmembrane potential (i.e., the potential Similarly, for K:
difference across the cell membrane) in the steady Ek = - 61 log [K]i / [K]o
state (unexcited state). RMP is present across all Ki = 140 mM, Ko = 5 mM, EK = - 92 mV
cells.
Equilibrium potentials for various ions across
Origin of RMP: nerve cell membranes:
1. The cell membrane is impermeable to organic ICF ECF Equilibrium
anions and proteins present in ICF. (mM) (mM) potential
2. In neurons for example, in the resting state, (mV)
Na 10 140 + 60
the cell membrane is quite permeable to K,
K 140 5 - 90
(about 50 times more permeable to K than Cl 10 120 - 70
Na). K exits the cell down its concentration Ca 100 nM 2.5 + 130
gradient, making the inside of the cell
negative with respect to exterior. Excitability:
3. Charge separation occurs across the Excitability is the response to a threshold stimulus
membrane (a thin capacitor) leaving the with a propagated action potential. Nerve and
inside of the membrane negative with respect muscle are excitable. Other tissues are not.
to outside.
4. Additionally, the Na-K ATPase mechanism The physiologic basis of excitability: high
contributes a small extent to making the potassium permeability at rest (and therefore an
inside of the cells negative. RMP in the range of –50 to –90 mV) and presence
of voltage gated Na channels are essential.
Magnitude of RMP in different tissues
Cell / tissue Magnitude of RMP What is the effect of hyperkalemia on RMP of
(inside negative) cardiac muscle cells?
Nerve cells - 70 mV
Normally, when plasma [K+] = 5 mM, RMP of
Skeletal muscle - 90 mV
GI smooth muscle Variable; - 40 to - 60 mV cardiac muscle cells = - 90 mV.
Cardiac muscle - 90 mV When plasma [K+] = 10 mM,
SA node - 70 mV RMP = - 61 log [140/10] = - 70 mV
Red blood cells - 10 mV That is, hyperkalemia makes RMP less negative.
SA node has an RMP of -70 mV. In this situation,
Driving forces for ion fluxes across membranes ventricular cells would be expected to compete
are the electrical and chemical (concentration) with SA node to pace the heart.
gradients. This clearly increases the risk of cardiac
arrhythmias.
Equilibrium potential: the membrane potential
at which net transmembrane flux of a particular Functional organization of neurons:
ion is zero because the electrical gradient
counterbalances the chemical gradient. Each neuron has the following zones:

Example, Na flux across the membrane would Receptive zone: afferent input converges on the
stop when the membrane potential reaches + 60 dendrites and soma.
mV.
Integrator: all afferent input is integrated by the
Equilibrium potential (given by Nernst equation) soma (cell body) and if the axon hillock is
of Na: depolarized to threshold, a full-fledged action
potential occurs.
14
E.S.Prakash. MCQs in Medical Physiology, May 2013
They are all-or-none; i.e., they occur with a
The axon transmits the nerve impulse (action constant size.
potential) down to the nerve terminal where
neurotransmitter is released for synaptic Excitability during various phases of the action
transmission. The action potential also sweeps potential:
back through the cell body wiping it clean of all
input and rendering it refractory to all input for a Absolute refractoriness: No matter how strong
short time. the stimulus, a nerve is absolutely refractory to
stimulation during the action potential until
Nerve conduction speed is increased several fold repolarization brings the membrane back to firing
by myelination of axons. level. The basis for absolute refractoriness is
voltage and time inactivation of voltage gated Na
If the nerve is stimulated with the cathode (using a channels.
subthreshold stimulus), it produces small
depolarizations (called catelectrotonic potentials). Relative refractory period: In this period, only a
These small depolarizations are local responses; suprathreshold stimulus would trigger an action
i.e. they are not propagated through the nerve potential. The reason why a stronger stimulus is
fiber. The stimulus that is just adequate to result in required for excitation is because the stimulus has
a propagated action potential is called threshold to overwhelm the repolarizing current.
stimulus.
Supernormal phase: a weaker stimulus would
The membrane potential at which voltage gated trigger an action potential.
Na-channels open all at once to result in a full
fledged action potential is called firing level or Subnormal phase: during afterhyperpolarization,
threshold level and it is about -55 mV in nerve a stronger stimulus would be required to bring the
fibers. Within a millisecond of opening, Na membrane to threshold.
channels get deactivated; i.e. close and cannot be
opened until the membrane potential comes back Stimulus: a transient change in one of the
down to firing level. On the other hand, voltage parameters of the environment.
gated K channels open allowing rapid efflux of K.
This repolarizes the membrane; i.e., brings it back Stimulus types: electrical, chemical, mechanical
to RMP.
Stimulus parameters:
Differences between local & action potentials: 1. Intensity: subthreshold, threshold,
Local potentials (also called electrotonic suprathreshold
potentials) may be depolarizing (also called 2. Duration: typically (in milliseconds)
catelectrotonic potentials) or hyperpolarizing 3. Frequency: 1-100 Hz
(also called anelectrotonic potentials). Excitatory 4. Rise time: time in which stimulus intensity
postsynaptic potentials and motor end plate rises to its maximum value.
potentials are examples of catelectrotonic
responses. Inhibitory postsynaptic potentials are Threshold stimulus: The minimum stimulus
anelectrotonic responses. Electrotonic potentials intensity that will elicit an action potential in an
are graded responses (i.e. proportional to stimulus excitable tissue under a given set of conditions.
intensity) that occur with subthreshold stimuli,
and their magnitude is typically a few mV. They All or none law: If a stimulus sufficiently intense
undergo spatial and temporal summation. They as to bring the membrane to threshold, the
are not propagated. intensity of the stimulus has no bearing on the size
of the action potential.
In contrast, action potentials are propagated
responses that occur with “threshold stimuli”. Physiologic basis of the all-or-none law:
15
E.S.Prakash. MCQs in Medical Physiology, May 2013
Action potential occurs only when the firing level 10 ms. In contrast, in slow skeletal muscle, twitch
is reached because the voltage-gated sodium duration is typically greater than 100 ms.
channels that allow a massive influx of Na open
only at the firing level. In cardiac muscle, when average heart rate is 75
beats per minute, average twitch duration is 800
Effect of stimulus intensity on electrical ms; i.e., cardiac muscle is “slower” than skeletal
response of nerve and muscle (all-or-nothing): muscle. The twitch duration in the smooth muscle
of sphincters is longer. Smooth muscle is thus the
Stimulus Electrical ‘slowest’ of three muscle types.
Intensity response in nerve
/ muscle Tetanizing stimulus frequency: In skeletal
Subthreshold No action muscle, with high frequency stimulation, it is
potential possible to summate the effects of multiple stimuli
Threshold Action potential and increase the force of muscle contraction.
Suprathreshold Action potential
Tetanus is the most forceful and sustained muscle
of the same
strength
contraction (i.e. with no relaxation in between).
To tetanize skeletal muscle, it must be stimulated
at a minimum frequency called the tetanizing
frequency that depends on the duration of
contraction. The minimum stimulus frequency
required for tetanizing the muscle is the reciprocal
of the contraction period expressed in seconds.

For example, if the contraction period is 100 ms


(0.1 s), tetanizing frequency is 1/0.1 = 10 Hz.

The figure above depicts action potential in a


muscle fiber (top panel) and mechanical response
(bottom panel) on the same time scale in skeletal
muscle. Whereas the duration of the action
potential is only about 5 ms, the duration of the
mechanical response is much longer. Thus, it is The figure above depicts the effect of an increase
possible to summate the mechanical responses to in the frequency of contraction of skeletal muscle
successive stimulation of muscle. In cardiac on force. With increasing frequency of stimulation
muscle, this is not possible. at a given intensity, the force of successive
contractions increase because of the beneficial
Muscle twitch: A muscle twitch is contraction effect of previous contractions; more calcium is
followed by relaxation occurring in response to a available for later contractions than earlier ones.
single stimulus. This is called the staircase phenomenon, Treppe
Twitch duration = contraction time + relaxation or Bowditch effect. With a decrease in frequency
time. Twitch duration is quite variable and of stimulation, a gradual decrease in force of
skeletal muscle is classified as fast or slow, contraction is also observed.
depending upon twitch duration. The duration of
muscle twitch in fast muscle could be as short as Axoplasmic transport: This refers to transport of
molecules in the cytoplasm of the axon. This is
16
E.S.Prakash. MCQs in Medical Physiology, May 2013
not to be confused with conduction of the nerve C. Initial segment
impulse, which is much faster. D. Node of Ranvier

Fast anterograde transport (i.e., from the cell 6. The site of origin in the nerve of conducted
bodies to axon terminals) is brought about by impulses is the:
microtubules (kinesin and several other proteins A. dendritic zone
are involved). B. axon hillock
C. node of Ranvier
Axoplasmic transport Speed D. terminal buttons
(mm/day)
Fast anterograde transport 400 7. In motor neurons, the portion of the cell with
Slow anterograde transport 0.5–10 the lowest threshold for the production of a full-
Retrograde transport 200 fledged action potential is:
A. initial segment
B. soma
Most if not all of the Questions in this Section
C. dendritic zone
have only one correct answer.
D. node of Ranvier
1. Fast anterograde transport occurs at a rate of
8. Action potential conduction velocity is
about:
influenced by:
A. 40 mm/day
A. axon diameter
B. 400 mm/day
B. temperature
C. 1 mm/day
C. myelination
D. 10 mm/day
9. In the CNS, the membranes that wrap around
2. The resting cardiac muscle cell is most
myelinated neurons are those of:
permeable to:
A. Schwann cells
A. Na
B. oligodendroglia
B. K
C. endothelial cells
C. Ca
D. astrocytes
D. Cl
10. The duration of action potential in a skeletal
3. The membrane potential of cardiac muscle cells
muscle fiber is typically:
is most affected by even a small change in plasma
A. 5 ms
concentration of:
B. 25 ms
A. Na
C. 200 ms
B. K
D. 250 ms
C. Cl
D. Ca
11. When heart rate is about 75 beats per minute,
what is the mean duration of action potential in
4. Hypokalemia would be expected to result in:
ventricular muscle cells?
A. increased neuronal excitability
A. 5 ms
B. a more negative RMP
B. 25 ms
C. no change in RMP
C. 200 ms
D. a decrease in firing level of neurons
D. 250 ms
5. The number of Na channels per square
12. The force of muscle contraction cannot be
micrometer of membrane in myelinated
increased by:
mammalian neurons is maximum in the:
A. increasing the frequency of activation of
A. Cell body
motor units
B. Dendritic zone
17
E.S.Prakash. MCQs in Medical Physiology, May 2013
B. increasing the number of motor units activated
C. increasing the amplitude of action potentials in 19. The activity of which contractile protein is
motor neurons altered to regulate smooth muscle contraction?
D. recruiting larger motor units A. Actin
B. Myosin
13. An excitable cell has an RMP of -70 mV and a C. Calmodulin
firing level of - 50 mV. This cell would be D. Tropomyosin
inexcitable when its membrane potential is:
A. - 30 mV 20. Smooth muscle lacks:
B. - 55 mV A. actin
C. - 70 mV B. myosin
D. - 90 mV C. troponin
D. tropomyosin
14. For the same conditions (as in the above
question), the cell would be most excitable when 21. An example for nonsyncytial smooth muscle
its membrane potential is: is:
A. - 30 mV A. iris
B. - 55 mV B. sphincter of Oddi
C. - 70 mV C. vas deferens
D. - 90 mV D. uterus

15. Although the equilibrium potential of Na is + 22. The twitch duration in fast type skeletal
60 mV, the membrane potential does not reach muscle fibers is about:
this value during the overshoot because: A. 10 ms
A. the concentration gradient of Na is reversed B. 50 ms
B. Na channels undergo rapid inactivation C. 100 ms
C. K efflux commences immediately after the D. 250 ms
upstroke
D. the membrane is impermeable to Na 23. The major source of calcium for contraction of
skeletal muscle is:
16. Which one of the following statements about A. ECF
electrotonic potentials is incorrect? B. cytosol
A. They are graded responses C. mitochondria
B. They are local (non-propagated) responses D. sarcoplasmic reticulum (SR)
C. They may be depolarizing or hyperpolarizing
D. They are produced by a threshold stimulus. 24. Ryanodine receptor is located in the:
A. sarcolemma
17. Thin filaments do not contain: B. T-tubule
A. actin C. terminal cisterns of sarcoplasmic reticulum
B. myosin D. cytosol
C. troponin
D. tropomyosin 25. Which of the following blocks the ryanodine
receptor?
18. The ATPase activity of which of the following A. Dantrolene
proteins is altered to regulate skeletal muscle B. Curare
contraction? C. Cocaine
A. Actin D. Hexamethonium
B. Myosin
C. Troponin 26. Which of the following slow the relaxation
D. Tropomyosin process in skeletal muscle?
18
E.S.Prakash. MCQs in Medical Physiology, May 2013
A. Slow myosin ATPase A. type II muscle fibers
B. Inhibition of Ca-Mg ATPase B. single-unit smooth muscle
C. ATP depletion C. orbicularis oculi
D. soleus
27. Rigor mortis is due to:
A. damage to actin & myosin 34. Gradation of force in skeletal muscle is not
B. rapid sequestration of Ca in ER achieved by:
C. increased myosin ATPase A. increasing intracellular calcium
D. ATP depletion B. recruitment of motor units
C. asynchronous firing of motor units
28. Staircase phenomenon (Treppe) is due to: D. varying release of calcium from SR
A. increased availability of intracellular calcium
B. summation 35. Regarding the ionic basis of action potential in
C. tetanus cardiac muscle cells, which one of the following
D. increased excitability is incorrect?
A. Phase 0: Na influx
29. Which of the following statements regarding B. Phase 1: K influx
type I muscle fibers is incorrect? C. Phase 2: Ca influx
A. They are rich in myoglobin D. Phase 3: K efflux
B. Their oxidative capacity is high
C. Their myosin ATPase activity is high 36. When heart rate is about 75 beats/min, the
D. Their glycolytic capacity is moderate duration of absolute refractory period of cardiac
muscle is:
30. Which of the following is incorrect about A. 50 ms
type II muscle fibers? B. 100 ms
A. They are called slow fibers C. 200 ms
B. Their myosin ATPase activity is high D. 300 ms
C. They contain little myoglobin
D. They are rich in glycolytic enzymes 37. When heart rate is about 75 beats/min, the
duration of ventricular systole is about:
31. Which of the following statements is A. 0.1 s
incorrect? B. 0.2 s
A. Contraction against a constant load with C. 0.3 s
approximation of the ends of the muscle is
called isotonic contraction. 38. Which of the following muscle types is the
B. Contraction can occur without an appreciable ‘fastest’?
decrease in the length of the muscle. A. Skeletal muscle
C. Isometric contractions do work whereas B. Smooth muscle
isotonic contractions do not. C. Cardiac muscle
D. Muscles can lengthen while doing work.
39. Which of the following characteristics is
32. The smallest amount of muscle that can exhibited only by skeletal muscle?
contract in response to excitation of a single motor A. Gradation of force production
neuron is: B. Refractoriness
A. 1 muscle fiber C. Beneficial effect
B. a muscle fasciculus D. Staircase phenomenon
C. the entire muscle E. None of the above
D. all muscle fibers supplied by that neuron
40. Non-linearity of length-tension relationship is
33. The size of the motor unit is smallest in: most evident in:
19
E.S.Prakash. MCQs in Medical Physiology, May 2013
A. skeletal muscle D. Aγ fibers
B. smooth muscle E. C fibers
C. cardiac muscle
48. The nerve fiber type most susceptible to
41. In adults, intravesical pressure rises suddenly conduction block by pressure is:
when intravesical volume reaches about: A. type A
A. 100 ml B. type B
B. 200 ml C. type C
C. 300 ml
D. 400 ml 49. Neuromuscular transmission is impaired in
conditions listed below (in List A), however, the
42. The latch-bridge mechanism in smooth muscle pathogenesis of muscle weakness is different in
is responsible for: each. Match items in List A with those in List B.
A. fast muscle twitch List A
B. sustained muscle contraction A. Cobra venom intoxication
C. excitation-contraction coupling B. Myasthenia gravis
D. unstable membrane potential C. Botulism
D. Lambert-Eaton syndrome
43. The minimum stimulus strength that produces
a compound action potential in nerve or muscle is: List B
A. rheobase 1. Antibodies to nicotinic Ach receptors in the
B. chronaxie motor end plate
C. twice rheobase 2. Toxin inhibits Ach release from presynaptic
D. twice chronaxie nerve terminals in skeletal muscle
neuromuscular junction
44. Which of the following accelerates contraction 3. Competitive blockade of nicotinic AchR in
as well as relaxation of heart muscle? skeletal muscle motor end plate
A. Norepinephrine 4. Antibodies to voltage-gated calcium channels
B. Calcium in presynaptic terminals
C. Digoxin
D. Potassium 50. Denervation hypersensitivity:
A. is due to upregulation of neurotransmitter
45. Which of the following statements about receptors in the denervated structure
cardiac muscle is incorrect? B. does not occur in smooth muscle
A. Summation cannot occur C. partly explains the release phenomenon that
B. It is a ‘fast’ muscle occurs in neurological diseases
C. Slow myosin ATPase activity
D. It contains myoglobin Answers: Physiology of Nerve & Muscle Cells
1B 2B 3B 4B 5D
46. Which nerve fiber type is most susceptible to 6B 7A 8all 9B 10A
conduction block by local anesthetics? 11D 12C 13A 14B 15BC
A. Type A 16D 17B 18B 19B 20C
B. Type B 21A 22A 23D 24C 25A
C. Type C 26all 27D 28A 29C 30A
31C 32D 33C 34D 35B
47. Nociceptors (nerve endings signaling pain) 36C 37C 38A 39E 40B
signal through: 41D 42B 43A 44A 45B
A. Aα fibers 46C 47CE 48A 49 50AC
B. Aβ fibers 49: 1B, 2C, 3A, 4D
C. Aδ fibers
20
E.S.Prakash. MCQs in Medical Physiology, May 2013
Answer Explanations: mitochondria,
1. Fast anterograde transport from the cell bodies capillary
to axons is brought about by kinesin. It occurs at a density)
speed of 400 mm/day.
Fast muscle fibers (type II) have high myosin
4. You can predict this by using the Nernst ATPase activity and their twitch duration is short,
equation. Hypokalemia makes cells less excitable whereas slow fibers (type I) have slow myosin
by reducing RMP (i.e. making it more negative). ATPase activity, and the twitch duration is
correspondingly longer.
5. Reference: Ch 2, p 59 In: WFG 22nd ed.
31. Types of muscle contraction
12. The reason is action potentials are all or none Isotonic Isometric
and occur with a constant magnitude. contraction contraction
Tension developed Tension
21. Gap junctions are absent from nonsyncytial is constant; muscle developed;
shortens Muscle does not
smooth muscle (example, smooth muscle of iris).
shorten
This makes possible fine graded contractions of
External work is “External work” is
the iris and regulation of pupil diameter. done not done
Example: Walking, Trying to lift a
23. Skeletal muscle depends on the SR as the bending, running heavy weight,
major source of calcium. Cardiac muscle and sustained
smooth muscle depend on influx of calcium from handgrip
ECF and Ca-induced calcium release for
contraction.
W = Force × displacement; since the muscle ends
27. Note that ATP is required for muscle are fixed in an isometric contraction, no ‘external
contraction as well as relaxation. work’ is done in an isometric contraction.

28. At or above threshold intensity, an increase in 32. A motor unit is the smallest amount of muscle
the frequency of stimulation (successive that can contract in response to excitation of a
stimulation rather) produces a gradual increase in single motor neuron.
the amplitude of successive contractions. This is
called staircase phenomenon or Treppe or 33. Motor unit: an alpha motor neuron and all
Bowditch effect. It occurs in skeletal as well as muscle fibers innervated by it.
cardiac muscle. In cardiac muscle, it is attributed Innervation ratio (IR): number of muscle fibers
to increased availability of intracellular calcium in a motor unit.
(beneficial effect of previous contractions). In small motor units: IR 3-10
In large motor units: IR >> 100
29. Classification of skeletal muscle fibers
Type I Type II Motor point: point where nerve enters muscle
Other names Slow, Fast, (anatomical)
oxidative, glycolytic,
red white According to the size principle, small motor units
Myosin Slow Fast
are recruited first and large motor units that
ATPase rate
Ca-pumping Moderate High
consist of large rapidly conducting axons are
capacity of SR recruited for more forceful contractions.
Oxidative High Low
capacity Slow motor Fast motor unit
(myoglobin, unit
content of

21
E.S.Prakash. MCQs in Medical Physiology, May 2013
Small, slow Large, rapidly • Note: Refractory period length depends upon
conducting conducting the duration of the action potential. For
axons axons example, when HR is about 150 bpm, the
Innervate slow Innervate fast duration of the action potential is definitely
muscle fibers fatigable muscle shorter.
(type I) fibers (type II)
38. Note that the terms slow and fast are applied
34. Gradation of force in skeletal muscle is to muscle based on twitch duration. Twitch
achieved by: duration is the duration of a single contraction
• recruitment of motor units followed by relaxation. In slow type skeletal
• ↑ the frequency of stimulation of motor units muscle fibers, it could be as long as 100 ms. The
• asynchronous firing of motor units twitch duration in cardiac muscle is normally 800
• variations in preload. ms (at a HR of 75 beats/min). The twitch duration
could be as short as 10 ms as it is in ‘fast’ skeletal
Gradation of force is not achieved by varying muscle. The minimum twitch duration in the heart
calcium release from the sarcoplasmic reticulum is about 300 ms (at a heart rate of 200/min).
because each action potential that enters the T Smooth muscle is perhaps the slowest. It is
tubule releases a constant amount of calcium from capable of sustained contraction (tonic
the terminal cisterns of the SR. contraction).

36. Cardiac muscle fibers are refractory during the 40. Some important definitions:
upstroke of the action potential and until at least Preload: the load on a muscle before it contracts.
repolarization is half complete. This is called In the ventricle, the preload (end-diastolic fiber
effective (or absolute) refractory period, and the length) varies with the end-diastolic volume.
heart cannot be excited during this time no matter
how strong the stimulus is. In the relative Afterload: the load which contracting muscle has
refractory period, a supranormal stimulus can to overcome before it shortens.
evoke an action potential. Because of the long
duration of absolute refractory period, diastole Within physiologic limits, an increase in end-
commences before the heart is reexcitable. Thus, diastolic fiber length increases tension developed
summation of the type seen in skeletal muscle (force of ventricular contraction). This is the
cannot occur in cardiac muscle and why cardiac Frank-Starling law of the heart.
muscle cannot be tetanized. This is a safety Similarly, in skeletal muscle, there is a linear
feature since the heart has to fill with blood relationship between preload and force of
between successive contractions. contraction when other factors such as recruitment
are held constant.
37. Events in the cardiac cycle
• Electrical (depolarization and repolarization) However, in smooth muscle, length-tension
are followed by mechanical events (systole relationships are nonlinear over the physiologic
and diastole) range. Consider for example the smooth muscle of
• Cardiac cycle time: RR interval (variable) the bladder. Refer to the cystometrogram in a
normal human (Fig 38-25, p. 727, Ch 38, WFG,
• Action potential duration: variable
2005). Note that an increase in intravesical
• If average HR is 75 bpm, then average cardiac
volume from 0 to 50 ml is accompanied by an
cycle time = 0.8 s = 800 ms
increase in intravesical pressure. However, an
• Average action potential duration ~ 250 ms increase in intravesical volume from 50 up to 380
• Duration of absolute refractory period ~ 200 ml is not accompanied by an increase in
ms intravesical pressure. This means that the bladder
• Relative refractory period ~ 50 ms is not ‘shortening’ in this range because if the

22
E.S.Prakash. MCQs in Medical Physiology, May 2013
bladder shortened, the intravesical pressure would muscle). The average twitch duration in the heart
have increased. is 800 ms when HR is 75 bpm.

Second, in a hollow viscus, wall tension T = Pr. 47. Fast pain is transmitted through myelinated A-
i.e., P = T/r. When intravesical volume increases delta fibers, and ‘slow pain’ is the result of
from 50 to 350 ml, the bladder is stretched and its transmission through unmyelinated C fibers.
radius increases. Stretch (the stimulus) elicits
some tension (the response) in the wall of the ***************************************
bladder. This is called passive tension because it
does not cause the radius to decrease but merely
enables it to withstand the distending pressure. On
the other hand, when intravesical volume is
increased to 400 ml, the bladder wall has been
adequately stretched and a strong reflex
contraction of the bladder occurs leading to
shortening (active tension) of the detrusor and
emptying of the bladder

Total tension = active + passive tension.

As far as this question is concerned, this


discussion is intended to emphasize the fact that
the relation between length (in this example,
intravesical volume) and tension is quite
unpredictable in smooth muscle.

42. The twitch duration in sphincters and smooth


muscle is particularly longer. Latch bridges are
actomyosin complexes that detach slowly; this is
made possible by a slow myosin ATPase. In turn,
this enables maintenance of tension for longer
periods without consumption of more ATP.

43. The strength-duration curve is a plot of the


strength of the electrical stimulus required to elicit
compound action potentials in nerve or muscle
against the duration of the stimulus. Rheobase is
the minimum stimulus strength that will produce a
response. Chronaxie has been defined as the
minimum stimulus duration at which twice the
rheobasic current will produce a response.

44. Norepinephrine has a positive chronotropic


(increases rate of contraction), inotropic (increases
force of contraction), and lusitropic effect (i.e., it
accelerates relaxation) on the heart.

45. Cardiac muscle is “slow” (i.e., its twitch


duration is longer compared to fast skeletal

23
E.S.Prakash. MCQs in Medical Physiology, May 2013
4. Functions of the Central Nervous 7. Presynaptic inhibition occurs at:
System A. axoaxonal
B. axosomatic
Most of the questions in this section have only C. axodendritic synapses
one correct answer, as should be apparent from
the stem of the question; however, some questions 8. Renshaw cell inhibition of alpha motoneurone
may have more than one correct answer – please is an example of:
read all options! A. negative feedback inhibition
B. feedforward inhibition
1. Which of the following types of cells work as C. presynaptic inhibition
scavenger cells in the CNS? D. postsynaptic inhibition
A. Microglia E. indirect inhibition
B. Oligodendroglia F. direct inhibition
C. Ependymal cells
D. Astrocytes 9. Which of the following is/are examples of
ligand-gated ion channels?
2. Which is the commonest type of synapse in the A. Nicotinic Ach receptor
CNS? B. GABA-activated Cl channels
A. Axosomatic synapse C. IP3 receptor
B. Axodendritic synapse D. Glycine receptor
C. Axoaxonal synapse
10. Which of the following statements about
3. Excitatory postsynaptic potentials (EPSP) are synaptic potentials is incorrect?
not produced by: A. They are propagated down the postsynaptic
A. opening of Na channels neuron
B. closure of K channels B. They undergo spatiotemporal summation
C. opening of calcium channels C. They are analogous to generator potentials and
D. opening of chloride channels end-plate potentials
D. IPSP hyperpolarize the postsynaptic neuron
4. Inhibitory postsynaptic potentials (IPSP) may E. They are proportional to the amount of
be produced by: transmitter released by the presynaptic neuron.
A. closure of Na channels
B. closure of Ca channels 11. Is conduction of nerve impulse in motor axons
C. opening of K channels unidirectional or bidirectional? Explain.
D. opening Cl channels
12. Most excitatory neurotransmission in the brain
5. The minimum time for transmission across one is believed to be mediated by:
synapse is: A. glutamate
A. 0.5 ms B. glycine
B. 1 ms C. GABA
C. 1.5 ms D. GnRH
D. 2 ms
13. The inhibitory amino acid neurotransmitters in
6. Inhibitory postsynaptic potentials are an the CNS are:
example of: A. glutamate and glycine
A. postsynaptic inhibition B. glutamate and aspartate
B. presynaptic inhibition C. GABA and glycine
C. direct inhibition D. aspartate and glycine
D. indirect inhibition
14. The GABA-A receptor is a:
24
E.S.Prakash. MCQs in Medical Physiology, May 2013
A. Na channel C. Muller’s law of specific nerve energies
B. Cl ion channel D. Bell-Magendie Law
C. Ca channel
D. cation channel 22. The relation between the magnitude of
sensation and stimulus intensity is given by the:
15. IPSPs due to chloride influx are produced by: A. law of specific nerve energies
A. acetylcholine B. labeled line principle
B. GABA C. Weber-Fechner law
C. glutamate D. law of projection
D. substance P
23. A single sensory axon and all of its peripheral
16. Which of the following neurotransmitters has branches constitute a:
both excitatory and inhibitory effects? A. receptive field
A. Glycine B. sensory unit
B. GABA C. dermatome
C. Aspartate D. sensory nerve
D. Glutamate
24. Intensity of a sensory stimulus (whether
17. Which of the following is an excitotoxin? threshold or subthreshold) is encoded by all of the
A. Glutamate following except:
B. Glycine A. size of generator potentials
C. Acetylcholine B. frequency of action potentials in sensory
D. Substance P neurons
C. recruitment of sensory units
18. Inhibitory interneurons in the spinal cord D. size of action potentials
release:
A. glycine 25. Which of the following is a rapidly adapting
B. GABA sensory receptor?
C. substance P A. Muscle spindle
D. neuropeptide Y B. Carotid sinus
C. Pain receptor
19. Neurons mediating direct inhibition in the D. Pacinian corpuscle
spinal cord usually secrete:
A. glycine 26. A typical example of a monosynaptic reflex is:
B. GABA A. stretch reflex
C. substance P B. superficial abdominal reflex
D. glutamate C. withdrawal reflex
D. light reflex
20. Tetanospasmin inhibits the release of:
A. GABA 27. The receptor for the stretch reflex is the:
B. glutamate A. muscle spindle
C. glycine B. Golgi tendon organ
D. acetylcholine C. Pacinian corpuscle

21. “No matter where a particular sensory 28. In the stretch reflex, the afferents are:
pathway is stimulated along its course to the A. I a fibers
cortex, the conscious sensation produced is B. I b fibers
referred to the location of the receptor”. This is: C. II fibers
A. the law of projection D. A delta fibers
B. Weber-Fechner law
25
E.S.Prakash. MCQs in Medical Physiology, May 2013
29. The term ‘small motor nerve system’ refers to: 37. All neural influences affecting muscle
A. Ia afferents from muscle spindle contraction ultimately funnel through:
B. α-motor neurons A. α-motor neurons
C. γ-motor neurons B. γ-motor neurons
C. corticospinal tract
30. In the stretch reflex, relaxation of antagonist D. basal ganglia
muscles is due to:
A. reciprocal innervation 38. Which of the following is the best example of
B. presynaptic inhibition a polysynaptic reflex?
C. Ia fiber stimulation A. Stretch reflex
D. antidromic conduction B. Axon reflex
C. Inverse stretch reflex
31. The muscle spindle is made to contract by D. Withdrawal reflex
stimulation of:
A. α-motor neurons 39. In humans, spinal shock is characterized by:
B. γ-motor neurons A. hypertonia
B. hyperreflexia
32. A muscle can normally be made to contract by C. spastic paralysis
stimulation of which of the following neurons D. loss of autonomic function
innervating that muscle?
A. α-motor neurons 40. Which of the following is not a feature of
B. γ-motor neurons lower motor neuron lesions?
C. Either one of the above A. Muscle atrophy
B. Fasciculations
33. The Jendrassik’s maneuver facilitates stretch C. Fibrillations
reflexes by: D. Denervation hypersensitivity
A. exciting alpha motor neurons E. Spastic paralysis
B. increasing gamma efferent discharge
C. releasing glycine 41. Locomotion generators in spinal cord are
turned on by pattern generators in the:
34. The Hoffmann reflex (H reflex) is typically A. suppressor strip
recorded from: B. midbrain
A. soleus C. medulla
B. orbicularis oculi D. pons
C. diaphragm
D. thenar muscles 42. The crossed extensor response is seen in:
A. healthy humans
35. The receptor for the inverse stretch reflex is: B. the phase of spinal shock
A. muscle spindle C. chronic paraplegia
B. Golgi tendon organ
C. C fiber 43. The mass reflex:
D. delta fibers A. is a sign of a central inhibitory state
B. occurs in the decerebrate animal
36. Impulses from Golgi tendon organ pass to C. is due to irradiation of afferent impulses
inhibitory interneurons in the spinal cord via: D. is a manifestation of ‘release of spinal
A. I a reflexes’ from inhibitory control by higher
B. I b centers
C. II a
D. C fibers 44. Which of the following fibers has the greatest
threshold?
26
E.S.Prakash. MCQs in Medical Physiology, May 2013
A. Touch C. Visual pathway
B. Pain D. Auditory pathway
C. Pressure
D. Cold 53. Axons of ganglion cells in the retina terminate
in the:
45. The gate theory of pain was proposed by: A. lateral geniculate nucleus
A. Charles Sherrington B. pretectal nucleus
B. Wall and Melzack C. suprachiasmatic nucleus
C. Weber-Fechner D. superior colliculus

46. Anterolateral cordotomy does not interfere 54. The light reflex is integrated in the:
with perception of: A. midbrain
A. pain B. frontal eye field
B. pressure C. medulla
C. temperature D. primary visual area
D. fine touch
55. Which one of the following is not a
48. Joint position sense is transmitted by: component of the near response?
A. anterior spinothalamic tract A. Pupillary constriction
B. lateral spinothalamic tract B. Convergence of the visual axes
C. dorsal column-medial lemniscal system C. Increase in convexity of the lens
D. Ciliary muscle relaxation
49. Which of the following is/are termed
‘synthetic senses’? 56. Which of the following statements about
A. Vibratory sensibility Argyll-Robertson pupil is incorrect?
B. Fine touch A. Light reflex is absent
C. Joint position sense B. Accommodation reflex is present
D. Stereognosis C. The lesion is in the Edinger-Westphal nucleus
E. Two-point discrimination D. It occurs in neurosyphilis

50. Impaired stereognosis in the absence of a 57. Central visual fields are mapped with:
detectable defect in touch and pressure sensation A. tangent screen (campimeter)
points to a lesion in: B. perimetry
A. the dorsal column
B. the medial lemniscus 58. Impedance matching is a function of:
C. the parietal lobe posterior to the postcentral A. scala media
gyrus B. endolymph
D. any of the above C. ear ossicles and tympanic membrane
D. cochlear nucleus
51. Ablation of somatosensory area (SI) does not
significantly impair: 59. What is the amplification provided by the
A. joint position sense lever action of the auditory ossicles and the large
B. touch localization size of the tympanic membrane compared to the
C. two-point discrimination oval window?
D. pain perception A. 5 times
B. 10 times
52. Which one of the following neural pathways C. 22 times
is predominantly an uncrossed pathway? D. 45 times
A. Somatosensory pathway
B. Gustatory pathway 60. Endolymph is produced by:
27
E.S.Prakash. MCQs in Medical Physiology, May 2013
A. stria vascularis
B. inner hair cells 68. Which of the following reflexes is always
C. sustentacular cells absent in the blind?
D. outer hair cells A. Vestibulo-ocular reflex
B. Nystagmus
61. Which of the following structures is involved C. Visual accommodation
in the spatial localization of sound? D. Light reflex
A. Spiral ganglion
B. Hair cells 69. Saccades are programmed in the:
C. Superior olivary complex A. medial longitudinal fasciculus
D. Cochlear nerve B. frontal cortex
C. hypothalamus
62. Which of the following functions as a D. superior colliculus
frequency analyzer in hearing?
A. Reissner’s membrane 70. In which layer of the cortex do specific
B. Tectorial membrane thalamic afferents end?
C. Basilar membrane A. Layer VI
D. Middle ear ossicles B. Layer IV
C. Layer I
63. Inner hair cells in the organ of Corti are D. Layer II
depolarized by:
A. K influx 71. In which layer of the cerebral cortex do non-
B. Na influx specific thalamocortical projections end?
C. Ca influx A. Layer I-IV
D. closure of K channels B. Layer II
C. Layer IV
64. Hyperpolarization of receptor cells activates: D. Layer VI
A. phototransduction
B. auditory signaling 72. Alpha block is produced by:
C. gustation A. mental arithmetic
D. olfactory signaling B. stimulation of ascending reticular activating
system
65. Which of the following changes occur in rod C. opening one eye only
cells when rhodopsin is activated by light?
A. Increase in cGMP 73. A 4–7 Hz rhythm normally occurs in:
B. Deactivation of phosphodiesterase A. hypothalamus
C. Depolarization of rod cells B. hippocampus
D. Decreased release of neurotransmitter C. parieto-occipital area
D. frontal area
66. Linear acceleration in the vertical plane is
signaled by hair cells in: 74. A low-frequency, high amplitude,
A. anterior semicircular canal synchronized cortical rhythm occurs during:
B. saccule A. stage 1 & 2 slow-wave sleep
C. posterior semicircular canal B. stage 3 & 4 slow-wave sleep
D. utricle C. REM sleep
D. narcolepsy
67. Taste projection area is located in the:
A. foot of the postcentral gyrus 75. Delta waves (0.5-4 Hz) are seen in:
B. precentral gyrus A. stage 1 sleep
C. cingulate gyrus B. stage 4 sleep
28
E.S.Prakash. MCQs in Medical Physiology, May 2013
C. an awake adult with eyes open A. Gross movements
D. an awake adult with eyes closed B. Temperature regulation
C. Hopping and placing reaction
76. Which of the following is not a characteristic D. Vestibulo-ocular reflex
of REM sleep?
A. Slow waves 83. Which of the following is not a feature of
B. Hypotonia corticospinal tract disease?
C. Dreaming A. Cogwheel rigidity
D. PGO spikes B. Spasticity
C. Plantar extensor response
77. Gamma oscillations in EEG, which occur D. Exaggerated deep tendon reflexes
when an individual is aroused and focuses
attention on something, occur at a frequency of: 84. All neural influences affecting muscle
A. 0–0.4 Hz contraction ultimately funnel through:
B. 4–7 Hz A. α-motor neurons
C. 0–12 Hz B. γ-motor neurons
D. 30–80 Hz C. corticospinal tract
D. basal ganglia
Control of Posture and Movement:
78. Guarding is an example of: 85. Lesions of the ventral corticospinal tract result
A. stretch reflex in:
B. flexor withdrawal reflex A. difficulty with balance
C. clasp knife effect B. deficits in skilled voluntary movements
D. α-γ linkage
86. Righting reflexes are integrated for the most
79. The procedure in which the hindbrain and the part in the:
spinal cord are separated from the rest of the brain A. medulla
by transection at the superior border of pons is B. midbrain
called: C. spinal cord
A. decortication D. cerebral cortex
B. decerebration
C. deafferentation 87. The cerebellum receives sensory input from
D. rhizotomy the:
A. labyrinth
80. Which of the following is pathognomonic of B. proprioceptors
decerebration? C. eyes
A. Exaggerated standing D. all of the above
B. Babinski sign
C. Absence of saccades 88. Climbing fiber input to the cerebellum comes
D. Relief of spasticity by cerebellectomy from:
A. superior olivary complex
81. Which type of rigidity is clinically more B. red nucleus
common? C. inferior olivary complex
A. Decorticate rigidity D. locus ceruleus
B. Decerebrate rigidity
C. Extensor rigidity 89. Feedforward inhibition has been well
D. Cog-wheel rigidity described in neural circuits in the:
A. spinal cord
82. Which of the following is absent in B. cerebellum
decorticate animals? C. stretch reflexes
29
E.S.Prakash. MCQs in Medical Physiology, May 2013
D. nociceptive pathway
98. Which of the following is/are an example(s) of
90. The doll’s eye reflex is integrated in the associative learning?
A. cerebral cortex A. Habituation
B. medial longitudinal fasciculus B. Sensitization
C. thalamus C. Classical conditioning
D. medulla D. Avoidance responses

91. Hypothalamus does not play a prominent role 99. Intercortical transfer of memory occurs via:
in the regulation of: A. corpus callosum
A. food and water intake B. anterior commissure
B. temperature C. posterior commissure
C. respiration D. each of the above
D. osmolality
100. A long lasting facilitation of transmission in
92. Lesions of ventromedial hypothalamus neural pathways following a brief period of high
typically result in: frequency stimulation is termed
A. anorexia A. long-term potentiation (LTP)
B. obesity B. post-tetanic potentiation (PTP)
C. hypersexuality C. sensitization
D. amnesia D. habituation

93. The circadian rhythm generator in the 101. Which receptor is directly involved in long-
hypothalamus receives inputs chiefly from: term potentiation?
A. retina A. Cholinergic receptors
B. superior colliculus B. P2 receptor
C. thalamus C. NMDA receptor
D. medial geniculate nucleus D. GABA-B receptor

95. When ambient temperature is about 20ºC, 102. Emotional responses often outlast the
body heat is lost chiefly by: duration of the stimulus because:
A. radiation and conduction A. limbic cortex is made up of three layers
B. vaporization of sweat B. neocortex cannot modify emotional behavior
C. respiration C. limbic circuits have prolonged afterdischarge
D. urination and defecation.
103. Disinhibition is a striking feature of disease
96. Which of the following is a heat conserving of:
mechanism? A. frontal lobe
A. Panting B. temporal lobe
B. Sweating C. occipital lobe
C. Curling up in a ball D. parietal lobe
D. Insensible water loss
104. The categorical hemisphere is chiefly
97. The process of; i.e., the conversion of short- concerned with:
term memories to long-term memories occurs in A. language
the: B. recognition of faces
A. hippocampus C. stereognosis
B. amygdala D. spatiotemporal relations
C. cerebral cortex
D. hypothalamus
30
E.S.Prakash. MCQs in Medical Physiology, May 2013
105. The representational hemisphere is Classification of synaptic inhibition based on
concerned with: structure of neural circuits:
A. spatiotemporal relations Negative feedback inhibition: Renshaw cell
B. stereognosis inhibition is a classic example of negative
C. recognition of faces feedback inhibition. Renshaw cells are inhibitory
D. recognition of musical themes interneurons in the spinal cord.

Answers: Functions of the CNS


1A 2B 3D 4all 5A
6AC 7A 8ADF 9all 10A
11 12A 13C 14B 15B
16A 17A 18A 19A 20C
21A 22C 23B 24D 25D
26A 27A 28AC 29C 30A
31B 32C 33B 34A 35B
36B 37A 38D 39D 40E
41B 42C 43CD 44B 45B
46D 48C 49ADE 50C
51D 52B 53all 54A 55D Feedforward inhibition occurs in cerebellar
56C 57A 58C 59C 60A circuits.
61C 62C 63A 64A 65D
66B 67A 68C 69BD 70B 11. In the intact human, the action potential is
71A 72ABC 73B 74B 75B initiated at the axon hillock and propagates
76A 77D 78B 79B 80A through the axon membrane to the nerve terminal.
81A 82C 83A 84A 85A However, if we stimulated axons at the median
86B 87D 88C 89B 90B nerve with a threshold stimulus applied through
91C 92B 93A 95A electrodes at the elbow, action potential in excited
96C 97A 98CD 99D 100A axons would travel orthograde to the nerve
101C 102C 103A 104A 105all terminal as well as retrograde into the cell body
located in the spinal cord. In a motor axon, it is
Answer explanations: anterograde transmission to the axon terminal that
2. 98% synapses in the CNS are axodendritic. can release neurotransmitter. Retrograde
conduction of the impulse to the cell body
3. Opening of chloride ion channels in depolarizes it sweeping it clean, and rendering it
postsynaptic membrane would produce refractory to stimulatory input for a definite
hyperpolarization of postsynaptic neurons. period of time.

6. Classification of synaptic inhibition: 16. Glycine, an inhibitory neurotransmitter in the


Direct Postsynaptic Glycine spinal cord, also has excitatory actions on NMDA
Inhibition inhibition producing receptors. In the absence of glycine, glutamate
IPSP on cannot exert its excitatory effects on the NMDA
postsynaptic receptor. See p. 110, Ch 4, WFG, 2005.
neurons
17. Classification of glutamate receptors:
Indirect Presynaptic GABA
inhibition inhibition producing
• Metabotropic
inhibition at • Ionotropic
axoaxonal • NMDA receptor (Na & Ca channel)
synapses • AMPA receptor

31
E.S.Prakash. MCQs in Medical Physiology, May 2013
Uncurbed excitatory neurotransmission In a teleological sense, if the carotid sinus adapted
permanently to an increase in arterial pressure, it
Increase in glutamatergic transmission wouldn’t play a role in the long term control of
arterial pressure. It is important to note that
Reason: Glutamate reuptake defective? adaptation can occur at the level of receptors
(peripheral resetting) as well in the neural circuits
Increase in intracellular calcium in postsynaptic in the CNS processing information from the
neurons receptor (this is called central resetting).

Neuron damage and death 26. Even in the stretch reflex, an interneuron is
interposed between the Ia afferent and the alpha
18. Golgi bottle neurons and Renshaw cells are motoneurone innervating the antagonist muscle;
two classic interneurons in spinal cord. Both however, this is perhaps the best answer for the
release glycine. Both cause direct inhibition of question.
postsynaptic neurons.
31, 32. Muscle spindle is a stretch receptor.
20. Tetanospasmin produces spastic paralysis by Contraction of the intrafusal fibers stretches the
preventing the release of glycine from inhibitory central portion of the intrafusal fiber (muscle
interneurons in the spinal cord. spindle). The afferents from the muscle spindle
pass via Ia and II fibers. The stretch reflex
Botulinum toxin causes flaccid paralysis by operates to regulate muscle length.
inhibiting the release of Ach from the
neuromuscular junction. A-γ motor neurons innervate the intrafusal fibers
whereas the extrafusal fibers (the regular
Strychnine blocks glycine receptors. contractile units of skeletal muscle) are made to
contract by stimulation of alpha motor neurons.
22. Weber-Fechner Law (Power law):
Magnitude of the sensation felt (R) = KSA where A-α motor neurons constitute the final common
S is stimulus intensity. path for all neural influences causing muscle
contraction.
Muller’s law of specific nerve energies (the
labeled line principle): Monosynaptic stretch reflex (example: knee jerk)
Each sensory pathway starting from the receptor Stimulus Stretch of muscle
up to the cortex is labeled for transducing a Receptor Muscle spindle
particular sensory modality. Afferents Ia and II fibers
Integration Spinal cord
Bell Magendie law: In spinal nerves, dorsal roots Response Contraction of muscle
are sensory and ventral roots are motor. Latency 20 ms
Significance It assesses the integrity of the
25. Touch receptors: reflex arc and higher
Rapidly-adapting Pacinian corpuscle influences upon it
touch receptor Meissner’s corpuscle Diminished or Lower motor neuron type
Slowly-adapting Merkel’s disks lost in disease, phase of spinal
touch receptor Ruffini endings shock, deafferentation,
Exaggerated in With Jendrassik’s maneuver
Touch: Pacinian corpuscle and conditions increasing
gamma motor neuron
Vibration: Pacinian corpuscle
discharge, upper motor
Pressure is sustained touch. Vibration is repetitive neuron type disease, chronic
touch stimuli. Pressure is presumably coded by spinal animals / humans
slowly adapting touch receptors. H reflex The pathway for this reflex is
the same as any monosynaptic
32
E.S.Prakash. MCQs in Medical Physiology, May 2013
reflex (MSR); it is a sensitive shock. It is a sign of release of spinal lower motor
means of assessing integrity neurons from the inhibitory control of higher
of MSR when reflexes are not centers.
clinically elicitable, as can
occur in acute onset 39. Spinal shock is presumably due to sudden loss
weakness. However, it is
of tonic bombardment of α-motor neurons by
recorded only from soleus (by
EMG). The H reflex is absent supraspinal neurons.
in Guillain-Barre syndrome.
40. Spasticity is a release phenomenon. It occurs
35. Golgi tendon organ is a stretch receptor located in as a result of release of brain stem motor neurons
series with collagen fibers in the muscle tendon; it is from inhibitory control by higher centers (i.e. the
innervated by Ib fibers cerebral cortex and the basal ganglia)

43. The mass reflex is a sign of a central


excitatory state; it is presumably a manifestation
of release of spinal lower motor neurons from
Inverse stretch reflex (autogenic inhibition) inhibitory control by higher centers. Denervation
supersensitivity may contribute to the occurrence
Excessive stretch of the mass reflex.

Activation of Ia afferents 45. The gate theory of pain posits that there is a
mechanism to facilitate or inhibit transmission of
Forceful contraction of muscle nociceptive impulses at the level of the spinal
cord. The putative gate is thought to be located at
Stretch activation of Golgi tendon organ in tendon the substantia gelatinosa of the spinal cord. Higher
(activation of Ib afferents)
centers exert influences on this gate. For example,
Muscle relaxation serotonin released from raphe spinal neurons
inhibits onward transmission of nociceptive inputs
38. Characteristics of withdrawal reflexes: (endogenous analgesia system).

Stimulus: Noxious stimulus 46 and 48. Ascending pathways and the sensory
Integration: Spinal cord modalities they code:
Control: Normally suppressed
by higher centers Dorsal column – Fine touch, touch
Response: Withdrawal of lemniscal system localization, two-point
affected body part discrimination, vibration,
Exaggerated in Chronic spinal joint position sense
animals as a result of Anterior Pressure, crude touch
“release” from spinothalamic
inhibitory control tract
from higher centers. Lateral Pain, temperature
Thus the crossed spinothalamic
extensor component tract
of the response is
seen only in chronic 49. Synthetic senses are synthesized in the sensory
paraplegics (i.e. association area in the brain (posterior parietal
following recovery cortex) from component senses. Stereognosis is a
from spinal shock) good example of a synthetic sense.
The crossed extensor response, which occurs 50. Since touch and pressure are both intact, one
due to irradiation of afferent stimuli, occurs only could presume that all sensory pathways right up
in spinal animals that have recovered from spinal
33
E.S.Prakash. MCQs in Medical Physiology, May 2013
to the somatosensory cortex are intact. Impaired
stereognosis must therefore be due to a defect 63. Hair cells are depolarized by K influx because
posterior to the somatosensory area (i.e. in the the endolymph is rich in K.
posterior parietal cortex).
66.
51. Ablation of somatosensory area does not result Stimulus Sensory receptor
in impairment of pain perception since the mere Linear acceleration in Saccule
perception of pain does not require the cortex. the vertical plane
Cortical processing is concerned primarily with Linear acceleration in Utricle
finer aspects of information processing including the horizontal plane
discrimination of subtle differences in stimulus Rotational acceleration Semicircular canals
in the plane of the
intensity, localization of stimulus.
rotation
53. Visual pathway: Retina >> Ganglion cells >> 67. There is no separate taste area in the brain as
Lateral geniculate nucleus >> Primary visual there is for other special senses. Impulses from
cortex (V1), Area 17 taste buds finally reach the foot of the postcentral
gyrus.
Pathway for accommodation:
Retina >> Ganglion cells >> Lateral geniculate 68. Think about how the accommodation reflex is
nucleus >> Primary visual cortex (V1), Area 17 elicited. We ask a person to look at an object
>> Frontal eye field (Area 8) >> Edinger- placed nearby after he looks at an object placed
Westphal nucleus (preganglionic parasympathetic farther away. Obviously, this is integrated in the
neurons) >> Ciliary ganglion (parasympathetic multiple areas in the cerebral cortex (visual cortex
postganglionic neurons) >> Iris, ciliary muscle followed by the frontal eye field) and then
projects to the Edinger-Westphal nucleus in the
Pathway for entrainment of circadian midbrain. Thus, a person who is blind (on both
rhythms: Ganglion cells >> Suprachiasmatic eyes) because of a defect in the visual pathway
nucleus (SCN) >> other relays in the anywhere beyond the optic chiasm will not have
hypothalamus >> brain stem reticular formation this reflex.
>> autonomic efferents
70, 71. Types of thalamocortical projections:
56. Light reflex absent; accommodation reflex Specific: function to transmit information to
present – Argyll Robertson pupil; lesion in the specific areas in the cortex; results in sensory
pretectal nucleus of the midbrain where light perception; they end in layer IV of the cortex
reflex is integrated. Nonspecific: function to arouse the individual;
end in layers I-IV of the cortex.
Light reflex present; accommodation reflex absent
– Wernicke’s pupil. In this situation, the lesion 70-75. Notes on EEG:
could be present anywhere beyond the exit of the In an adult human at rest with mind wandering
fibers from the optic nerve to the pretectal and eyes closed, the most prominent component
nucleus. of the EEG is a fairly regular pattern of waves at a
frequency of 8-12 Hz, and amplitude of about 50
59. Amplification by lever action of ear ossicles = microvolts when recorded from the scalp. This
1.3. Ratio of surface area of tympanic membrane pattern is the alpha rhythm. It is most prominent
and oval window = 17; total amplification = 17 × in the parieto-occipital area.
1.3 = 22
The alpha rhythm is replaced by a fast irregular
60. Endolymph is rich in K like ICF; low voltage beta rhythm (18–30 Hz). This
Perilymph is rich in Na; like ECF; phenomenon is called alpha block. A breakup of
Endolymph and perilymph do not mix.
34
E.S.Prakash. MCQs in Medical Physiology, May 2013
the alpha rhythm is produced by any kind of An overview of control of posture and
sensory stimulation such as mental arithmetic. movement
• What is meant by ‘posture’
Large amplitude, regular 4–7 Hz waves called the • What is meant by ‘movement’
theta rhythm occurs in children and is generated • What are the posture regulating pathways?
in the hippocampus in experimental animals. • What are the movement regulating pathways?

Stages of sleep: Stages I – IV of NREM (slow Animal preparations used to study the
wave sleep) followed by rapid eye movement regulation of posture:
(REM) sleep
• Spinal preparation: spinal cord transection at
Sleep spindles (occur in stage II sleep) midthoracic level
Frequency: 10–14 Hz; amplitude: 50 microvolts. • Decerebrate preparation: transection of the
neuraxis at the superior border of pons
EEG waves Frequency
Delta 0.5–4 Hz • Midbrain preparation: transection of the
Theta 4–7 Hz neuraxis on top of the midbrain
Alpha 8–13 Hz • Decortication: removal of cerebral cortex
Beta 14–30 Hz alone leaving subcortical structures intact
Gamma 30–80 Hz
What does each of the following mean?
Delta waves occur in deep sleep (stages III and • Upper motor neuron (UMN)
IV). • Lower motor neuron (LMN)
• Pyramidal pathway
REM sleep is called paradoxical sleep because it • Extrapyramidal pathway
is marked by rapid, low voltage, irregular EEG • Lateral system pathways
activity. The threshold for sensory arousal from • Medial system pathways
REM sleep is sometimes greater than NREM
sleep presumably because the brain is actively
processing information during REM sleep
(dreams).

EEG during sleep:


Stage Characteristics
Stage 1 Alpha rhythm
Stage 2 Sleep spindles and K
complexes (beginning of
slowing; reduction in
alpha content)
Stages 3 & 4 Characterized by slow
waves (theta and delta)
REM sleep High frequency, low The mechanism of decerebrate rigidity:
amplitude rhythm similar
to the wakeful state

Ponto-geniculo-occipital (PGO) spikes


Origin: Pons
Termination: Occipital cortex
Neurotransmitter: Acetylcholine
Function: Generation of dream imagery?

Overview for Questions 78-88


35
E.S.Prakash. MCQs in Medical Physiology, May 2013
Lateral system pathways
• Lateral corticospinal tract
Classification of upper motor neurons: • Rubrospinal tract
• Posture regulating upper motor neurons Medial system pathways:
• Movement regulating upper motor neurons • Ventral corticospinal tract
• Reticulospinal tract
Posture regulating upper motor neurons: • Vestibulospinal tract
Origin: Brain stem • Tectospinal tract
Termination: On alpha and gamma motor
neurons in spinal cord Lateral system pathways are movement
Controlled by: Inhibitory inputs from cerebral regulating pathways. If we could manage to cut
cortex (suppressor strip – 4s)
in the anterior edge of the
only lateral corticospinal tract axons (for example,
precentral gyrus and basal by sectioning pyramids in monkeys), posture
ganglia. would not be affected but the animal would lose
its ability to perform fine skilled movements. The
Movement mediating / regulating upper motor affected muscles would become hypotonic.
neurons:
Origin: Corticospinal tract (layer V, Medial system pathways are posture-regulating
pyramidal cell axons) from pathways.
primary motor cortex, sensory
cortex, and premotor area. Other descending monoaminergic pathways
Termination: On alpha-motor neurons / • Raphespinal pathways
interneurons innervating alpha
motor neurons in spinal cord
• Ceruleospinal pathway
Lesions: limited to the corticospinal and
corticobulbar tracts produce One way of classifying descending motor
“weakness” rather than pathways is to classify them as lateral system
paralysis and the affected pathways and medial system pathways.
musculature is generally
“hypotonic”. Isolated The lateral system pathways include the lateral
involvement of corticospinal corticospinal tract and the rubrospinal tract (which
tract is not clinically common. originates in the red nucleus). The neurons of the
lateral corticospinal tract typically (though not
Level of integration of principal postural reflexes: necessarily) end straight on alpha-motor neurons
Level Postural reflexes innervating skeletal muscles of the distal
Spinal cord Stretch reflexes, withdrawal
musculature used for fine, skilled voluntary
reflexes
Medulla Labyrinthine righting reflexes,
movements; otherwise, they end on interneurons
tonic neck reflexes that regulate the alpha-motor neurons innervating
Midbrain Righting reflexes these muscles. The lateral system pathways also
Cerebral Hopping and placing reactions, course laterally (i.e., in the lateral funiculus) in the
cortex optokinetic reflexes spinal cord.

Classification of movements: The medial system pathways course medially in


Reflexive All reflexes the spinal cord and end (at various levels)
movements typically on interneurons in the medial aspect of
Rhythmic Swallowing the ventral horn of the spinal cord. Furthermore,
movements these interneurons, on which the medial system
Voluntary Writing, talking, skilled pathways end, connect with their counterparts on
movements movements the contralateral aspect of the spinal cord
contributing to balance and posture. They mainly
Descending motor pathways:
control axial muscles. The ventral corticospinal
36
E.S.Prakash. MCQs in Medical Physiology, May 2013
tract, the tectospinal tract, the reticulospinal tract 93. Circadian rhythms are 24-h biological rhythms
and the vestibulospinal tracts are all medial entrained by the light-dark cycle. They are paced
system pathways. in the suprachiasmatic nucleus (SCN) in the
hypothalamus. The SCN receives neural input
Functional divisions of the cerebellum from retina. In blind individuals, circadian
Division Function rhythms are “free running” and have a periodicity
Spinocerebellum Maintenance of of about 25 hours.
(midline & paravermal posture
zone) 97. Notes on memory:
Vestibulocerebellum Maintenance of Memory is retention and storage of learnt
(flocculonodular lobe) posture and
information.
equilibrium
Neocerebellum Coordination of • Implicit memory is reflexive (nondeclarative).
(lateral cerebellar voluntary It is nondeclarative in that one is not aware of
hemispheres) movement its execution. Examples include skills one has
perfected.
• Output from the Purkinje cells to the deep • Explicit memory, (which is declarative),
cerebellar nuclei is always inhibitory. requires conscious recall of events or facts. It
• Output from the deep cerebellar nuclei to the requires processing in the hippocampus.
thalamus is excitatory. • Working memory is a form of short term
• Cerebellum facilitates stretch reflexes and memory used to plan actions; example, dialing
usually hypotonia occurs in cerebellar lesions. a phone number one has just seen on the
• Selective ablation of the flocculonodular lobe phone book. It is stored in prefrontal cortex.
of the cerebellum has been shown to abolish • Consolidation refers to the formation of new
the symptoms of motion sickness. The long term memories from short-term
symptoms of motion sickness occur due to memories, and it occurs in the hippocampus.
excessive stimulation of the vestibular system The hippocampus is rich in NMDA receptors.
via its connections to the brain stem and the NMDA receptors are a type of glutamate
flocculonodular lobe of the spinal cord. See receptors.
p.184, Ch 9, WFG, 2005.
• Short-term memory is prone to erasure; long-
88. The only source of climbing fiber input to the term memory (‘true memory’) is resistant to
cerebellum is the inferior olivary complex. There erasure.
is evidence that this pathway is activated when a
new motor task is learnt. Mechanisms implicated in memory:
 Post-tetanic potentiation
90. This is the vestibulo-ocular reflex, used as a  Long-term potentiation
test of brain stem integrity, in patients who are  Changes in synaptic strength
comatose. The absence of the doll’s eye reflex in  Increases in synaptic contacts
comatose patients is a grave prognostic sign.  Formation of new neurons from stem cells in
the olfactory bulb and hippocampus may play
92. Hypothalamus and food intake: a role.
Satiety center works by inhibiting feeding center
Feeding center located in Lateral Post-tetanic potentiation: Enhanced postsynaptic
hypothalamus potentials after a brief tetanizing train of stimuli in
Satiety center located in Ventromedial the presynaptic neuron.
hypothalamus
Lesions in ventromedial Obesity Long-term potentiation: Long-lasting
nucleus lead to
facilitation of transmission in neural pathways
Lesions in lateral Anorexia
following a brief period of high-frequency
hypothalamus lead to
stimulation. This process is important for
37
E.S.Prakash. MCQs in Medical Physiology, May 2013
consolidation, i.e. the formation of long term limbic system of “reward centers” and
memories. “punishment centers”
• “We may apply this concept to reform
Conditions which influence consolidation society” – Skinner
(formation of new LTM from STM)
• Repetition of stimulus (facilitates) Habituation: The response to a benign stimulus
• Sleep facilitates consolidation repeated over and over gradually decreases.
• Convulsions, ECT (inhibit) This is due to decreased release of excitatory
• Anesthetics, tranquilizers (inhibit) neurotransmitter from the presynaptic neuron. The
• Hypoxia (inhibits) opposite response where the response to a noxious
• Inhibitors of protein synthesis (inhibit) stimulus repeated over and over gradually
increases is termed sensitization.
98. Types of learning:
Associative learning Nonassociative Both habituation and sensitization are examples of
learning nonassociative learning because the organism
Associating a Learning about a learns about a single stimulus.
conditioned stimulus single stimulus
with a neutral stimulus 99. Limbic system:
Examples include Examples include • The seat of emotions
classical (Pavlovian) habituation and
conditioning, and sensitization
• It is three layered
operant / instrumental • It is phylogenetically older compared to
conditioning neocortex
• It has reciprocal connections with cortex
Classical conditioning making it possible to control emotions.
(Pavlovian conditioning) • It has connections with reticular activating
• Conditioned stimulus (food) system; thus, emotions can impact upon
• Conditioned response (salivation) visceral function.
• Conditioned reflex • There may be considerable reverberation of
• Neutral stimulus (bell ringing) impulses in limbic circuits (Papez circuit); this
• Pairing the neutral stimulus with the may be why emotions last longer and are
conditioned stimulus several times… memorized.
• The neutral stimulus (bell ringing) eventually
elicits the same response as the conditioned **************************************
stimulus (food).
• Conditioned reflexes (or behavior) are classic
examples of associative learning.

Instrumental conditioning
(Synonymous with operant conditioning)
• This phenomenon was described by
B.F.Skinner.
• Here the animal learns by operating on the
environment.
• The main learning point here is that animal
behavior is determined by the likely
consequences of that behavior. From
experiments conducted with the Skinner box
arose concepts such as the presence in the

38
E.S.Prakash. MCQs in Medical Physiology, May 2013
5. Endocrinology & Reproduction C. exercise
1. Which of the following hormones is not a D. protein meal
glycoprotein?
A. TRH 8. Which of the following hormones has intrinsic
B. FSH lactogenic activity?
C. LH A. TSH
D. hCG B. MSH
C. GH
2. Which of the following is not synthesized from D. dopamine
prepro-opiomelanocortin (pre-POMC)?
A. MSH 9. A human growth hormone variant (hGH-V) is
B. ACTH expressed primarily in:
C. Endorphin A. liver
D. Dynorphin B. spleen
C. kidney
3. Which of the following hormones does not act D. placenta
through G-protein coupled receptors?
A. Dopamine 10. Check all correct statements.
B. Epinephrine A. The beta cell releases equimolar amounts of
C. Angiotensin II insulin and C-peptide
D. ACTH B. NSILA is due to circulating C-peptide
E. Retinoic acid C. C-peptide has a shorter half-life compared to
F. Thyroxine insulin
D. C-peptide level in plasma provides an index of
4. The group of chemical messengers whose B-cell function in patients receiving
actions are known to be mediated by receptor exogenous insulin.
tyrosine kinases includes:
A. angiotensin II, ANP and ET 1 11. Which of the following increases insulin /
B. EDRF, ANP and AVP glucagon molar ratio the most?
C. FSH, LH and GHRH A. A large carbohydrate meal
D. insulin, EGF, IGF 1, PDGF B. Intravenous glucose
C. Overnight fast
5. Selective section of the pituitary stalk in an D. A small protein meal
experimental animal usually increases the E. Low carbohydrate diet
secretion of which of the following hormones?
A. Growth hormone 12. Arrange the following steps in the order in
B. FSH which they occur in the thyroid gland.
C. LH 1. Coupling reaction
D. Prolactin 2. Iodination
3. Endocytosis of thyroglobulin
6. The term ‘neurohormone’ is applied to: 4. Oxidation of iodine
A. oxytocin and vasopressin 5. Iodine uptake
B. NO and CO 6. Cleavage of thyroglobulin
C. glycine and glutamate 7. Secretion of T4
D. FSH and LH 8. Iodide trapping

7. Growth hormone secretion is increased by all of 13. Bone resorption is induced by abnormally
the following except: elevated levels of:
A. cortisol A. IGF-1
B. hypoglycemia B. insulin
39
E.S.Prakash. MCQs in Medical Physiology, May 2013
C. estrogens
D. glucocorticoids 21. In humans, the most abundant adrenocortical
hormone is:
14. Check all correct statements regarding A. cortisol
oxytocin. B. corticosterone
A. It is essential for milk ejection C. deoxycorticosterone
B. It increases uterine contractility D. DHEA sulfate
C. It facilitates ejaculation of semen
D. It facilitates sperm transport in the uterus 22. The major adrenal androgen is:
A. etiocholanolone
15. Calcitonin inhibits the activity of: B. testosterone
A. osteoblasts C. dihydrotestosterone
B. osteoclasts D. DHEA sulfate
C. parafollicular cells E. androstenedione
D. thyroid
23. In which zones is corticosterone converted to
16. Somatostatin inhibits the secretion of: aldosterone?
A. insulin A. Zona glomerulosa
B. glucagon B. Zona fasciculata
C. growth hormone C. Zona reticularis
D. gastrin
24. The hormone that is synthesized only in the
17. Hans Selye defined “stressors” specifically as zona glomerulosa is:
stimuli that increase the secretion of ______ in A. DHEA
healthy humans or laboratory animals. B. aldosterone
A. CRH C. corticosterone
B. ACTH D. deoxycorticosterone
C. TSH
D. cortisol 25. Injection of pharmacologic doses of ACTH in
healthy experimental animals is least likely to
18. Which of the following organelles is a major produce an increase in:
site of steroidogenesis? A. cortisol
A. Peroxisomes B. lymphocyte count
B. Ribosomes C. skin pigmentation
C. Smooth endoplasmic reticulum D. aldosterone secretion
D. Rough endoplasmic reticulum
26. Glucocorticoids decrease the number of
19. Which of the following hormones has the circulating:
highest mineralocorticoid activity? A. eosinophils
A. Cortisol B. lymphocytes
B. Corticosterone C. platelets
C. Deoxycorticosterone D. red blood cells
D. Cortisone
27. Which of the following does not occur as a
20. Which of the following hormones has consequence of adrenalectomy?
maximum glucocorticoid activity? A. Decreased ability to excrete a water load
A. Cortisol B. Glucose intolerance
B. Corticosterone C. Increased vulnerability to hypotension
C. Deoxycorticosterone D. Reduced alertness
D. Cortisone E. Dextrose fever
40
E.S.Prakash. MCQs in Medical Physiology, May 2013
35. The fetal zone of the adrenal cortex in the
28. Insulin sensitivity is not reduced in: fetus predominantly secretes:
A. glucagon-secreting tumors A. progesterone
B. tumors of adrenal medulla B. androstenedione
C. tumors of adrenal cortex C. DHEA
D. hypopituitarism D. DHEA sulfate

29. Which of the following peptides has been 36. The principal estrogen secreted by the
shown to have significant incretin effects? fetoplacental unit is:
A. Vasoactive intestinal polypeptide A. estriol (E3)
B. Somatostatin B. estradiol (E2)
C. Glucagon like polypeptide C. estrone (E1)
D. Pancreatic polypeptide
37. 21 hydroxylase deficiency results in:
30. Which of the following is the first of a new A. congenital adrenal hyperplasia
class of incretin mimetics approved for use in the B. testicular feminization syndrome
treatment of type II diabetes? C. male pseudohermaphroditism
A. Exenatide
B. Adiponectin 38. The enzyme deficiency that accounts for most
C. Resistin instances of the virilizing form of congenital
D. Leptin adrenal hyperplasia is:
A. 21 hydroxylase
31. Which of the following hormone peaks during B. 17α hydroxylase
the mid-luteal phase? C. 11β hydroxysteroid dehydrogenase
A. Inhibin B D. 3β hydroxysteroid dehydrogenase
B. FSH
C. LH 39. The hypertensive form of congenital adrenal
D. GnRH hyperplasia is due to a deficiency of:
A. 11β hydroxylase
32. Which of the following causes luteolysis? B. 21 hydroxylase
A. PGF2 alpha C. 3β-hydroxysteroid dehydrogenase
B. Progesterone D. DHEA sulfokinase
C. Inhibin B
D. Relaxin 40. Which of the following almost guarantees
‘maleness’?
33. With a sensitive radioimmunoassay, hCG can A. Presence of Y chromosome
be detected in the blood of a pregnant woman as B. Functional embryonic testes
early as ___ days post conception. C. Absence of Barr body in neutrophils
A. 2 days D. Well developed male external genitalia
B. 6 days
C. 10 days Answers: Endocrinology & Reproduction
D. 14 days 1A 2D 3EF 4D 5D
6A 7A 8C 9D 10AD
34. Human chorionic gonadotropin is structurally 11A 13D 14all 15B
and functionally similar to: 16all 17B 18C 19C 20A
A. LH 21D 22D 23A 24B 25B
B. FSH 26AB 27B 28D 29C 30A
C. growth hormone 31A 32A 33B 34A 35D
D. inhibin 36A 37A 38A 39A 40D

41
E.S.Prakash. MCQs in Medical Physiology, May 2013
Answer explanations: • Contraction of vas deferens and propulsion of
sperm toward urethra (during ejaculation)
1. TRH is a tripeptide
17. Hans Selye defined stress as any stimulus that
2. Pre-proopiomelanocortin (POMC) derivatives increases the release of ACTH in experimental
are: animals or normal humans.
• Beta-endorphin
• MSH (alpha, beta and gamma) 21. DHEA levels peak in the third decade of life
• ACTH and decline thereafter. Thus, it has been looked
• Beta-lipotropin (LPH) upon as a marker of aging. There is no evidence at
• CLIP the moment that DHEA supplementation reverses
changes associated with aging. However, the most
3. Retinoic acid, thyroxine, steroid hormones act abundant adrenocortical hormone in plasma is
mainly through receptors located in the cytosol or DHEAS. Thus, it is appropriate to use plasma
nucleus of target cells. However, recent evidence DHEAS level as an index of functioning
suggests that at least some of the physiologic adrenocortical mass.
effects of steroid hormones like estrogens,
aldosterone are mediated via receptors located in 26. Glucocorticoids decrease the number of
the cell membrane. These actions have come to be eosinophils, basophils and lymphocytes in the
therefore called ‘nongenomic actions’ of steroid circulation.
hormones.
Glucocorticoids, particularly in pharmacologic
5. Normally, dopamine from the hypothalamus doses suppress cell-mediated immunity by
inhibits the release of prolactin. multiple mechanisms including:
• inhibiting IL-2 gene transcription
6. Neurohormones are hormones synthesized and • inhibiting T cell proliferation
secreted by neurons into the general circulation • inducing apoptosis in lymphocytes

10. Non-suppressible insulin like activity 27. Mechanism of dextrose fever in


(NSILA) is insulin activity that is not suppressed adrenocortical insufficiency:
by anti-insulin antibodies. It is due to insulin-like Suppose large amounts of 5% dextrose (an
growth factors IGF-I (somatomedin C) and IGF- isoosmotic solution) is administered,
II. ↓
Dextrose is metabolized
12. Steps in thyroid hormone synthesis: ↓
1. Iodine uptake (Iodide trapping) The water added dilutes plasma; however water
2. Oxidation of iodine excretion is defective
3. Iodination of tyrosine residues ↓
4. Coupling of iodotyrosines Plasma osmolality decreases
5. Endocytosis of thyroglobulin ↓
6. Cleavage of thyroglobulin Water enters cells
7. Secretion of T4 ↓
Water influx into temperature-sensitive neurons
14. Functions of oxytocin: disrupts function of the hypothalamic thermostat
• Milk ejection ↓
• Uterine contractions at the time of labor Fever
• Uterine contractions at the time of sexual
intercourse promote sperm transport in the Based on the same mechanism (entry of water into
uterus cells), cerebral edema can occur as a complication
of a reduction in osmolality in this condition.
42
E.S.Prakash. MCQs in Medical Physiology, May 2013
Mullerian inhibiting substance from the
28. Diabetogenic hormones (hormones which embryonic testes causes regression of Mullerian
when present in excess reduce insulin sensitivity ducts.
and thus increase plasma glucose levels):
• Epinephrine Development of the male external genitalia
• Norepinephrine requires the presence of significant amounts of
• Glucagon (especially in individuals with dihydrotestosterone (the biologically most active
DKA) androgen) and normal androgen receptors.
• Growth hormone
• Cortisol Barr bodies are visible as drumstick chromosomes
• Thyroxine (particularly in individuals in only about 1-15% of neutrophils. Barr bodies
predisposed to either type 1 or 2 diabetes) are absent in individuals with Turner’s syndrome.
However, patients with Turner’s syndrome are not
29. Incretins are gastrointestinal hormones that male.
increase insulin secretion in response to an oral
glucose load. Gender is assigned at birth on the basis of external
Examples: genitalia. The presence of well-developed male
external genitalia at birth signifies that genetic sex
• Glucose dependent insulinotropic polypeptide
(GIP) is the most important incretin is male, testes are functional, and
dihydrotestosterone was present in adequate
• Glucagon-like polypeptide (GLP) has also
amounts. The purpose of the question is to clarify
been shown to have incretin effects.
the difference between the terms genetic sex,
Gonadal sex and phenotypic sex.
30. Exenatide is an incretin mimetic. It is a GLP
analog and has been approved for use in type 2 Note the exact meaning of these terms:
diabetes. 44 XY (male) 44XX (female)
Genetic sex
31. Inhibin from the ovary inhibits the release of Testes (male) Ovaries
FSH from the pituitary. Gonadal sex (female)
Phenotypic sex Male external Female external
34. HCG is a glycoprotein with 2 subunits α & β (at birth) genitalia genitalia
The α subunit is identical with that in TSH, LH &
FSH. The β subunit confers specificity. When Supplement on steroidogenesis
[hCG] is high, stimulation of TSH receptors could
result in mild hyperthyroidism. HCG maintains Biosynthesis of steroid hormones – The best
the corpus luteum of pregnancy; it is luteotropic in way to learn this is to use the figure on
contrast to LH which is luteinizing; indeed both Steroidogenesis in Ganong’s Review of Medical
HCG and LH act upon the same receptor. Physiology in the chapter on the adrenal cortex.
You can think of the scheme operating in the zona
35. WFG, 2005, pp. 450, Fig 23-37 fasciculata as the basic scheme. Other cells in
which steroidogenesis occurs differ from this
40. The Y chromosome contains the testis- basic scheme in some way: ZG, ZR, theca interna
determining gene (or SRY); this gene encodes a cell, granulosa cell (follicular phase, luteal phase);
transcription factor that is essential for the Leydig cell; fetal zone of the adrenal cortex;
development of embryonic testes. Presence of Y adipose tissue; aldosterone sensitive distal
chromosome merely dictates the formation of nephron etc. Some notes are below.
embryonic testes.
What happens in the placenta?
Testosterone, one of the hormones produced by Placenta’s priority is to make enough
embryonic testes, induces the formation of male
internal genitalia but not external genitalia.
43
E.S.Prakash. MCQs in Medical Physiology, May 2013
progesterone, thereby taking over the function of of aldosterone synthesis and secretion by the ZG.
the corpus luteum and maintaining pregnancy. The stimulatory effects of ACTH on the ZG are
transient if at all. However, with ACTH
Cholesterol deficiency from longstanding hypopituitarism, the
Side chain cleavage enzyme ZG may also diminish in size.
Pregnenolone Fetal adrenal DHEA sulfate

Progesterone Fetal adrenal Cortisol Cholesterol

What happens in the fetal zone of adrenal


cortex? – This zone is recruited to produce Pregnenolone
DHEAS and 16-OH DHEAS, and it sends this
back to the placenta.
Progesterone
Cholesterol
Side chain cleavage enzyme
Deoxycorticosterone
Pregnenolone
17-hydroxylase Corticosterone

17-hydroxy pregnenolone 18-OH corticosterone


17, 20 - lyase
Aldosterone
Dehydroepiandrosterone (DHEA)
Adrenal sulfokinase The last three steps are catalyzed by a single
enzyme aldosterone synthase (aldo synthase;
Dehydropepiandrosterone sulfate (DHEAS) CYP aldo; CYP 11B2) – this is an enzyme with 3
enzyme activities – 11 beta-hydroxylase 2, 18-
Placenta makes estradiol and estriol starting hydroxylase and and 18-oxidase .
from DHEAS.
Fetal DHEAS Placenta Estradiol What happens in zona fasciculata (ZF) and
zona reticularis (ZR)? (Fetal as well as adult)

16-OH DHEAS Placenta Estriol ACTH is the principal trophic hormone for the ZF
and ZR.
The fetal zone of the adrenal cortex in the fetus,
which constitutes about 80% of the gland, Cholesterol
degenerates at the time of birth. The major Cholesterol desmolase; scc enzyme; CYP450scc
function of the fetal zone is to produce DHEAS. Pregnenolone
The placenta synthesizes estradiol and estriol from 3-beta hydroxysteroid dehydrogenase
DHEAS and maintains pregnancy. Excretion of Progesterone
estriol in the urine of a pregnant woman is one 21-hydroxylase (CYP 21B)
integrated index of the health of the fetoplacental 11-Deoxycorticosterone (11-DOC)
unit. For more information on this topic, see 11ß-hydroxylase (CYP 11B1)
WFG. Corticosterone

What happens in zona glomerulosa? How is cortisol produced in the ZF?


17-hydroxy-pregnenolone
Aldosterone synthase is normally expressed only
in the zona glomerulosa. 17-alpha hydroxylase is 3-beta hydroxysteroid dehydrogenase
not expressed in this zone. Angiotensin II and 17-hydroxy-progesterone
changes in plasma [K] are the principal regulators 21-hydroxylase
44
E.S.Prakash. MCQs in Medical Physiology, May 2013
however, that DHEA sulfate and androstenedione
11-Deoxycortisol are in themselves biologically weak as androgens.
11ß-hydroxylase (CYP 11B1) They need to be converted to testosterone and or
Cortisol
dihydrotestosterone or they must be present in
very high concentrations to have effects on the
Below is the other pathway that is particularly androgen receptor. Note that all androgens exert
prominent in the ZR (under the stimulatory their effects via the same androgen receptor.
influence of ACTH) – the synthesis and secretion
of DHEA sulfate and androstenedione (adrenal Congenital adrenal hyperplasia (CAH)
androgens). Both of them function as The two most important causes include:
prohormones - precursors to the synthesis of more • 21 hydroxylase deficiency (commonest)
potent androgens testosterone and DHT, as well as • 11-beta hydroxylase 1 deficiency
estrogens.
Pathogenetic sequence:
Cholesterol to 1. Block in synthesis of glucocorticoids and or
Pregnenolone to aldosterone
17-hydroxypregnenolone to 2. Increase in ACTH (as a result of loss of
Dehydroepiandrosterone (DHEA) to negative feedback effects of cortisol)
Dehydroepiandrosterone sulfate (DHEAS) 3. Compensatory bilateral adrenal hyperplasia
4. The block remains
Normally, DHEAS (and not cortisol) is the most 5. Increased synthesis of adrenal androgens
abundant adrenocortical hormone in the
circulation. Salt loss and Addisonian crises with severe
deficiency of 21 hydroxylase (salt losing form of
Why does virilization occur in CAH due to 21 CAH) accompanied by adrenogenital syndrome in
hydroxylase deficiency? newborns (males as well as females). Screening a
This can be predicted using the steroidogenesis newborn for possible congenital adrenal
pathway in the adrenal cortex, and based on the hyperplasia is done by estimating serum 17-
fact that cortisol is the most potent negative hydroxyprogesterone. If it is elevated then it
feedback regulator of ACTH release from the should be confirmed by estimating aldosterone
pituitary. and cortisol.

Abnormality - 21-hydroxylase deficiency Manifestations in males:


• Accentuation of male characteristics
Loss of negative feedback effect of cortisol on • Precocious pseudopuberty
ACTH secretion from the pituitary and CRH
release by hypothalamus Manifestations in females:
• With mild deficiency of 21 hydroxylase, the
main consequence of clinical significance may
Excess progesterone & excess 17-hydroxy- be virilization at the time of birth (simple
progesterone, which are diverted to the synthesis virilizing form of CAH)
and secretion of • Precocious pseudopuberty
• Nonclassic CAH (late onset): virilization in
postpubertal females
DHEA sulfate and androstenedione (large
amounts) Clinical presentation depends on severity of
deficiency:
1. Salt losing form – with possible Addisonian
Masculinizing effects in XX fetuses and females crises
when present in large concentrations. Note,
45
E.S.Prakash. MCQs in Medical Physiology, May 2013
2. Ambiguous genitalia at birth in genetic Enzyme Consequences of deficiency
females (female pseudohermaphroditism); described: classic (salt-losing
adrenogenital syndrome form) of CAH; simple
3. Precocious pseudopuberty in males virilizing form of CAH; and
4. Virilization in postpubertal females non-classic (late onset) form of
CAH.
11β- Cortisol deficiency; virilization
Summary of Consequences of deficiencies
hydroxylase due to excessive synthesis of
of some enzymes involved in (CYP11B1) adrenal androgens;
steroidogenesis in the adrenal cortex: hypertension due to
mineralocorticoid activity of
Enzyme Consequences of deficiency high levels of 11-
Cholesterol Fatal in utero as no steroid deoxycorticosterone
desmolase hormones can be synthesized
(side-chain Principles of diagnosis of disorders of sexual
cleavage differentiation: These are the most important
enzyme) facts one needs to know in order to understand
Steroidogenic This enzyme is not expressed and diagnose disorders of sexual differentiation.
acute in the placenta, and its
regulatory deficiency has been described 1. For our purposes, it is reasonable to think of
protein to be in some cases compatible development of female internal and external
(STAR) with life. STAR is expressed in
genitalia as a default genetic program in
the adrenal cortex, testes and
ovaries and is needed to
intrauterine development regardless of genotype.
transport cholesterol to the
inner mitochondrial membrane 2. However, the development of normal ovaries
in these tissues. STAR from the bipotential gonad requires the presence
deficiency results in congenital of 2 normal X chromosomes and the absence of
lipoid adrenal hyperplasia. the sex-determining region of the Y-chromosome
3-beta Rare; this isoform is expressed (SRY). Several genes are required for the
hydroxysteroid in the fetal zone of the adrenal development of ovaries – therefore, development
dehydrogenase cortex and the gonads but not of ovaries should not be thought of as a default
(type 2) in the placenta. It is pathway.
characterized by deficiency of
cortisol, aldosterone and
3. If the SRY gene is present, whether on the Y
neither estrogens nor
testosterone can be chromosome (normal), or hiding in a X
synthesized. In genetic males, chromosome, it dictates the formation of
the result is development of embryonic testes.
female external genitalia.
Affected infants typically 4. The differentiation of the gonadal ridge may be
present with salt losing crises. limited by deficiency of factors such as
17α- Rare, no sex hormones, female steroidogenic factor (SF-1), a nuclear receptor
hydroxylase genitalia develop regardless of involved in transcriptional regulation of many
(CYP17) genetic sex, characterized by genes including those need for steroidogenesis in
an increase in plasma the ovary and the testes.
corticosterone, hypertension &
hypokalemia
5. Embryonic testes may or may not be
21 95% congenital adrenal
hydroxylase hyperplasia is due to functional. Functional embryonic testes secrete
(CYP21) deficiency of this enzyme; testosterone (Leydig cells) and Mullerian
however, manifestations vary Inhibiting Substance (or Polypeptide, MIS or
depending on severity of MIP, from Sertoli cells).
enzyme deficiency. Three
types of presentations are
46
E.S.Prakash. MCQs in Medical Physiology, May 2013
6. Like in the adrenal cortex, steroidogenic acute Finasteride is a more potent inhibitor of the type 2
regulatory protein (STAR) is expressed in the 5-alpha reductase; however, finasteride has also
gonads and is required for normal synthesis of been used in the management of hirsutism in
steroid hormones. As for intrauterine females.
development, this is particularly important in
males because embryonic testes produce 9. T and DHT mediate their effects via the same
testosterone. AR. DHT-AR complexes are more stable. This
explains why DHT is biologically the most potent
7. Placenta does not normally express STAR and androgen.
still synthesizes progesterone. Since STAR
deficiency, at least, in some cases is compatible 10. The embryonic ovary does not produce any
with life, it is likely because some of the hormones, and estrogens are not required for
progesterone from the placenta is channeled into intrauterine development of female internal or
other sites of steroidogenesis (fetal zone of the external genitalia.
adrenal cortex, definitive adrenal cortex, testes
and ovaries) and progesterone may serve as the 11. The critical period of differentiation of
starting point for steroidogenesis in these tissues genitalia in intrauterine life is 8-12 weeks. T
in this particular condition. This does not mean synthesis by the Leydig cell during this time is
STAR is not necessary, but is a mechanism that under the control of hCG (LH receptor). After
potentially provides partial compensation for mid-gestation, fetal pituitary LH modulates fetal T
STAR deficiency. synthesis and affects the growth of the
differentiated penis.
6. With normal androgen receptors, testosterone
(T) induces the development of 'male internal 12. Genetic sex - male (46 XY), female (46 XX);
genitalia' (epididymis, vas deferens, seminal gonadal sex (male - if testes are present; female -
vesicles) from the Wolffian duct (mesonephric if ovaries are present); phenotypic sex - based on
duct). external genitalia - male, female, ambiguous.

7. MIS (MIP) acts in a paracrine fashion and 13. Gender assignment at birth is based on
induces regression of ipsilateral Mullerian duct. external genitalia (male; female; ambiguous)
In the absence of MIS (MIP), the Mullerian ducts
develop into Fallopian tubes, uterus, uterine 14. Gender identity: Identification of self as male
cervix and the upper third of the vagina. In or female. This is affected by one's perception of
contrast, the lower 2/3rd of vagina is formed by his/her genitalia in comparison to others, how the
septation of the urogenital sinus. child is raised (as female or male). The size of the
phallus is a particularly important determinant.
8. Dihydrotestosterone (DHT), formed from T by
the action of testosterone 5-alpha reductase (type 15. Remember applications of knowing the basic
2), is required for the development of male scheme of steroidogenesis, and feedback
external genitalia from the urogenital sinus into responses to a defect somewhere in the pathway.
(growth of the male phallus, urethra, complete
fusion of labioscrotal swellings to form the When you consider all of this along with
scrotum). Of course, the concentration of mechanisms involved in puberty and the actions
androgen receptors (AR) and their sensitivity to of sex hormones, then we have most of the factual
androgens should be normal for this to happen. premises needed for understanding the phenotype
The formation of DHT occurs inside target cells. in most disorders of sexual differentiation and
therefore diagnosing them.
Normally, the type 2 5-alpha reductase
predominates in the genital tract, and the isoform Learning Activity: Using these principles,
of the enzyme in the pilosebaceous unit is type 1.] describe how you would suspect and diagnose

47
E.S.Prakash. MCQs in Medical Physiology, May 2013
each of the following conditions, as well as 6. Gastrointestinal Physiology
evaluate a female presenting with amenorrhea. As
you describe, note each of the following: genetic In each of the following questions, select the
sex, gonadal sex, internal genitalia, external single best response.
genitalia, how the child is likely to be raised (as
male or female). Consider: changes at the time of 1. Motor neurons in the myenteric plexus that
puberty; gender identity; the possibility of stimulate the contraction of smooth muscle in the
malignancies; reproductive life; genitourinary gastrointestinal tract release:
function, as well as ethical issues that may arise in A. acetylcholine
management. B. vasoactive intestinal polypeptide
• CYP21 (21 hydroxylase) deficiency; C. nitric oxide
• Klinefelter's syndrome D. somatostatin
Turner's syndrome; E. enkephalins
• Complete resistance to androgens;
• Complete deficiency of testosterone 5-alpha 2. Postganglionic parasympathetic neurons
reductase; innervating the gastrointestinal smooth muscle are
• Defect in embryonic testes; located in:
• Selective deficiency of MIP; A. myenteric plexus
B. submucosal plexus
• Aromatase deficiency (CYP 19);
C. paravertebral ganglia
• 17-beta HSD deficiency
D. prevertebral ganglia
**********************************************
3. The term “brain of the gut” is used to refer to
the:
A. autonomic ganglia
B. enteric nervous system
C. migratory motor complex
D. interstitial cells of Cajal

4. Which of the following hormone(s) is/are


normally released by the stomach into the
systemic circulation?
A. Ghrelin
B. Gastrin
C. Pepsinogen
D. Secretin
E. Vasoactive intestinal polypeptide

5. Which of the following inhibits gastric acid


secretion by parietal cells in the stomach?
A. Prostaglandin E2
B. Gastrin
C. Acetylcholine
D. Histamine

6. Enterocytes are replenished by mitotically


active undifferentiated cells located in:
A. Brunner’s glands
B. crypts of Lieberkuhn
C. Peyer’s patches
48
E.S.Prakash. MCQs in Medical Physiology, May 2013
D. gut associated lymphoid tissue 14. Physiologically, the most important
choleretic(s) is/ are:
7. The pattern of electrical and motor activity in A. bile salts
the gastrointestinal tract during periods of fasting B. CCK
is called: C. secretin
A. basic electrical rhythm D. gastrin
B. migrating motor complex (MMC)
C. peristalsis 15. Bile salts are essential for absorption of fat
D. segmentation because they:
A. solubilize dietary lipids in micelles
8. The major humoral mediator of meal- B. increase surface tension
stimulated enzyme secretion is: C. are hydrophobic
A. secretin D. contain cholesterol
B. CCK
C. GIP 16. Micelles in bile are formed by:
D. gastrin A. bile salts and phospholipids
B. bile acids and bile salts
9. The major humoral mediator of pancreatic duct C. cholesterol and bile salts
cell secretion of bicarbonate rich juice is: D. cholesterol and phospholipids
A. secretin
B. CCK 17. The total circulating bile salt pool is
C. somatostatin approximately:
A. 35 mg
10. The major humoral mediator of gall bladder B. 3.5 g
contraction in response to a fat meal is: C. 150 mg
A. CCK D. 30 g
B. gastrin
C. secretin 18. In which of the following segments in the
D. somatostatin splanchnic circulation is pressure least in a
healthy individual at rest?
11. The most potent stimulus for release of A. Hepatic arteriole
secretin from the duodenum is: B. Hepatic sinusoid
A. a reduction in duodenal lumen pH to < 4.5 C. Hepatic vein
B. peptides D. Terminal branches of portal vein in the liver
C. fatty acids with > 8 carbons E. Venule in the small intestinal mucosa
D. carbohydrates
19. Quantitatively, the most important enzyme in
12. Which of the following statements regarding the digestion of fat is:
somatostatin is incorrect? A. lingual lipase
A. It inhibits release of growth hormone B. gastric lipase
B. It inhibits release of insulin C. pancreatic lipase
C. It decreases blood flow to the intestine D. lipoprotein lipase
D. It stimulates gastric acid secretion
20. Which of the following is a bile acid
13. Which is the most important cholagogue? synthesized by the hepatocyte?
A. Secretin A. Sodium taurocholate
B. CCK B. Chenodeoxycholic acid
C. Gastrin C. Deoxycholic acid
D. GIP D. Lithocholic acid

49
E.S.Prakash. MCQs in Medical Physiology, May 2013
21. Which of the following transporters is present C. tonic contraction
only in the sinusoidal membrane of hepatocytes? D. migrating motor complex
A. Bile Salt Export Pump
B. LDL-cholesterol receptors 28. Normally, there is a net secretion of which ion
C. MDR 3 (Flippase) in the colon?
D. Multidrug Resistance associated Protein - 2 A. Sodium
(MRP-2) B. Potassium
C. Chloride
22. The Oral Rehydration Solution is helpful in D. Calcium
rehydration in diarrheas because:
A. Na-glucose symporter is unaffected in 29. In clinically significant diarrhea, typically, the
toxigenic diarrheas most abundant anion in stool is:
B. 1Na-1K-2Cl symporter is located only in the A. acetate
basolateral membrane B. bicarbonate
C. cAMP regulated Cl secretion is reduced C. butyrate
D. toxins inhibit facilitated glucose transport D. chloride

23. Normally, the rate-limiting step in the 30. Normally, most of the water in the GI lumen is
metabolism of bilirubin is: absorbed from:
A. uptake by ligandin A. stomach
B. conjugation with glucuronic acid B. duodenum
C. secretion into bile C. jejunum
D. colon
24. Which of the following is absorbed in the
intestine independent of sodium? Answers – Gastrointestinal Physiology
A. Glucose 1A 2A 3B 4AB 5A
B. Amino acids 6B 7B 8B 9A 10A
C. Galactose 11A 12D 13B 14A 15A
D. Fructose 16A 17B 18C 19C 20B
21B 22A 23C 24D 25BD
25. The type(s) of contraction that normally 26D 27A 28B 29B 30C
occur(s) only in the colon is/are: (choose one or
more answers) Explanations:
A. peristalsis 4. Ghrelin is released from the stomach during the
B. antiperistalsis fasting state and it stimulates food intake by an
C. segmentation action on the hypothalamus (it is an orexigenic
D. mass action contraction signal unlike leptin which signals satiety). Ghrelin
is also a stimulator of the growth hormone
26. The most frequent type of movement in the secretagogue receptor (GHS-R) in the anterior
small intestine in the digestive state is: pituitary, a receptor that mediates growth
A. peristalsis hormone release.
B. antiperistalsis
C. slow wave 7. The migrating motor complex (MMC) sweeps
D. segmentation contraction secretions from the mid-stomach through to the
terminal ileum. The MMC presumably serves to
27. The pattern of intestinal motility that hastens prepare the gut for the next meal; i.e. it is the
transit in the small intestine in the digestive state “housekeeper of the small intestine”. It is
is: augmented by motilin.
A. peristalsis
B. segmentation contraction
50
E.S.Prakash. MCQs in Medical Physiology, May 2013
13 and 14. Choleretics stimulate secretion of bile; There are regional variations in the frequency of
since this is osmotically driven, bile salts are the basic electrical rhythm (BER); for example in
themselves amongst the most potent choleretics. the stomach the pacemaker is located in the
Cholagogues (e.g. cholecystokinin) increase flow middle of the body of the stomach, and its
of bile from the liver or gallbladder into the frequency is 3-4 per minute.
duodenum. CCK enhances bile flow into the
intestine by stimulating contraction of the gall Rate of peristalsis: The rate of peristalsis is quite
bladder. variable (2–25 cm/min). Each peristaltic wave is
propagated usually for a short distance (4–5 cm).
17. The total bile salt pool of 3.5 g recycles On the other hand, the rate of MMC is about 5
repeatedly via the enterohepatic circulation six to cm/min. The MMC operates from mid stomach
eight times per day. until the terminal ileum. It is not propagated any
further.
18. Blood flows down a pressure gradient from:
venule in small intestinal mucosa → portal vein Antiperistalsis occurs as part of the vomiting
→ branches of portal vein in the liver → hepatic response. But otherwise, normally, antiperistalsis
sinusoid → hepatic vein. is observed only in the ascending colon. It is
actually a sequence of peristalsis-antiperistalsis
19. This is evident from the fact that chronic sequences; the antiperistalsis is said to be 'weak'
pancreatic exocrine insufficiency results in and eventually the peristaltic wave overrides and
steatorrhea. In contrast, fat digestion and sweeps content aborally. Like segmentation
absorption is not significantly affected by contractions do, peristalsis-antiperistalsis
deficiency of the other lipases as long as sequences in the ascending colon allow more Na
pancreatic lipase is available in adequate amounts. and Cl and water to be absorbed in the colon.
Normal antiperistalsis in the colon cannot
20. Cholic and chenodeoxycholic acids are propagate across a competent ileocecal valve.
primary bile acids; deoxycholic acid and
lithocholic acids are formed in the intestine 28. The secretion of K in the distal colon is
(secondary bile acids) by the action of bacteria on stimulated by aldosterone. Hypokalemia can
primary bile acids. result from diarrheal losses of K in individuals
with chronic diarrhea or those with severe acute
25-27. Gastrointestinal motility: You can build diarrhea. The diarrhea of VIPoma may be
your ideas along the following three lines: accompanied by hypokalemia.
• Digestive vs. interdigestive motility
• Propulsive movements vs. mixing movements 30. Digestion and absorption of nutrients normally
• Anatomical: stomach, duodenum, proximal occur mainly in the jejunum. Active absorption of
colon, sigmoid colon etc. amino acids, glucose is coupled to a Na-gradient,
and since we know that the intestine cannot
Digestive Interdigestive maintain an osmotic gradient across its epithelium
motility motility and water movement follows osmotically - it can
Peristalsis MMC be deduced that most of the water in the intestine
Segmentation is absorbed in the jejunum. There is experimental
Haustration evidence in support of this. In a healthy adult,
about 5.5 liters of water is absorbed in the
Propulsive Mixing movement jejunum (out of 9 liters absorbed from the GI
movement (fosters digestion lumen).
& absorption)
Peristalsis Segmentation *************************************
MMC Haustral shuttling

51
E.S.Prakash. MCQs in Medical Physiology, May 2013
7. Circulating Body Fluids on MCHC. Hyperchromia is uncommon but it
may occur in hereditary spherocytosis.
Red blood cell indices
Working definitions:
• What are the RBC indices?
Normocytes MCV WNL
• How are they derived? Macrocytes MCV > 95 fL
• Mention normal values for each. Microcytes MCV < 80 fL
• How are they used in the differential diagnosis Hypochromia MCHC < 25 g/dL
of anemias?
• Calculate RBC indices given that RBC count In each of the following questions, select all
= 5 million/mm3, blood Hb = 15 g/dL, and correct answers.
PCV = 45%
1. Macrocytes have an MCV > than:
RBC Derivation Normal A. 80 fL
Index values B. 95 fL
MCH Hb in g/dL × 10 29–32 pg C. 90 fL
RBC in D. 85 fL
millions/mm3
MCV PCV × 10 80–95 fL
2. Each red blood cell contains approximately:
RBC in
millions/mm3 A. 20 pg
MCHC Hb in g/dL × 100 32–35 g/dL B. 25 pg
PCV C. 30 pg
MCH, mean corpuscular hemoglobin; MCV, mean D. 35 pg of hemoglobin
corpuscular volume; MCHC, mean corpuscular hemoglobin
concentration; fL, femtoliter; pg, picogram. 3. The most reliable RBC index is:
A. MCV
I use this ‘trick’ below to recall the right formula for B. MCH
each index, in case I forget them. Normally, C. MCHC
MCV = 90 = 450 / 5 = (PCV × 10)
= 45 × 10 / 5 RBC
D. mean cell diameter
MCH = 30 = 150 / 5 = Hb × 10
= 15 × 10 / 5 RBC count 4. The amount of hemoglobin present in 100 ml of
MCHC = 33 = 1500 / 45 = Hb × 100 red blood cells is called:
= 15 ×100 /45 PCV A. MCH
B. MCHC
And then one can use these formula to plug in C. Hb index
values at hand. D. Price Jones index

MCHC = MCH/MCV; 5. MCHC does not usually exceed:


15 g of Hb is present in 100 ml of blood A. 30 g/dL
15 g of Hb is present in 45 ml of PRBC B. 33 g/dL
30 g of Hb is present in 90 ml of PRBC C. 37 g/dL
33 g of Hb is present in 100 ml of PRBC D. 25 g/dL

MCHC is the amount of Hb present in 100 ml of 6. Primary hemostasis refers to cessation of


packed red blood cells. bleeding due to:
A. formation of a definitive clot
RBC indices are useful in the differential B. clot retraction
diagnosis of anemias. Anemias may be classified C. formation of a temporary platelet plug.
as macrocytic, microcytic or normocytic based on
MCV, and normochromic or hypochromic based 7. Platelet aggregation is stimulated by:
A. thromboxane A2
52
E.S.Prakash. MCQs in Medical Physiology, May 2013
B. fibrinogen D. vitamin K
C. ADP
D. thrombin 15. Mutations in which of the following have been
E. epinephrine implicated in the pathogenesis of hypercoagulable
states?
8. Platelet aggregation is inhibited by: A. Protein C
A. ADP B. Protein S
B. 5-HT C. Factor V
C. PG I2 D. Antithrombin III
D. TX A2
16. Factor V Leiden:
9. Which of the following clotting factors is not A. is a mutated form of factor IX
vitamin K dependent? B. is inactivated by protein C
A. Factor II C. is present in a large subset of patients with
B. Factor V venous thromboembolism
C. Factor VII
D. Factor IX 17. Which condition(s) is / are characterized by an
E. Factor X increase in both bleeding time and clotting time?
A. Afibrinogenemia
10. The extrinsic pathway is triggered by the B. Hypoprothrombinemia
release of: C. Hemophilia A
A. factor VII D. von Willebrand’s disease
B. tissue factor
C. tissue factor pathway inhibitor 18. Red cell antigens A and B are chemically:
D. contact factor A. phospholipids
B. glycosphingolipids
11. The enzyme that ultimately lyses fibrin is: C. glycopeptides
A. plasminogen D. polypeptides
B. TPA
C. urokinase 19. Red blood cell antigens A and B are also
D. plasmin present in:
A. saliva
12. The extrinsic pathway is inhibited by: B. semen
A. tissue factor C. amniotic fluid
B. thromboplastin D. pancreas
C. tissue factor pathway inhibitor (TFPI)
D. contact factor 20. The red blood cells of a person with the
Bombay blood group do not have
13. The test that screens the extrinsic pathway is: A. GLUT
A. prothrombin time (PT) B. H substance
B. activated partial thromboplastin time (aPTT) C. spectrin
C. thrombin time D. ankyrin
D. urea solubility test
E. clot lysis time 21. CO is formed as an end product of:
A. heme metabolism
14. Prolongation of prothrombin time does not B. arginine metabolism
occur when there is a deficiency of only: C. oxidation of acetoacetate
A. factor VIII
B. factor IX 22. Heme is converted to bilirubin mainly in the:
C. factor VII A. kidneys
53
E.S.Prakash. MCQs in Medical Physiology, May 2013
B. liver D. macrocytic anemia
C. spleen
D. bone marrow 30. When a serum sample is electrophoresed,
which one of the following bands is absent?
23. The protein that binds extracorpuscular A. Albumin
hemoglobin is: B. α1 globulin
A. hemin C. α2 globulin
B. haptoglobin D. Fibrinogen
C. hemopexin E. γ-globulin
D. haptopexin
31. Erythropoietin is produced by:
24. Which of the following is not synthesized in A. interstitial cells surrounding peritubular
the liver? capillaries in the renal cortex;
A. IgG B. endothelial cells in peritubular capillaries in
B. α2 macroglobulin the renal cortex
C. Albumin C. perivenous hepatocytes
D. Angiotensinogen D. Kupffer cells of liver

25. Which of the following plasma proteins are 32. Hereditary spherocytosis occurs due to
protease inhibitors? mutations in genes coding for:
A. α1 antitrypsin A. spectrin and ankyrin
B. Transferrin B. Na-K ATPase
C. C-reactive protein C. glucose 6 phosphate dehydrogenase
D. Antithrombin III D. pyruvate kinase

26. Which of the following is a negative acute 33. The average half-life of neutrophils in the
phase reactant? circulation is:
A. Albumin A. 6 hours
B. C-reactive protein B. 5 days
C. α2 macroglobulin C. 2 weeks
D. Transferrin D. 1 month

27. ESR is increased in: 34. Most of the iron in the body is present in:
A. anemia A. hemoglobin
B. hypofibrinogenemia B. myoglobin
C. spherocytosis C. ferritin
D. each of the above D. transferrin

28. The protein content of lymph draining from Answers: Circulating Body Fluids
the _______ is highest. 1B 2C 3C 4B 5C
A. choroid plexus 6C 7all 8C 9B 10B
B. skeletal muscle 11D 12C 13A 14AB 15all
C. liver 16C 17ABD 18B 19all 20B
D. gastrointestinal tract 21A 22C 23B 24A 25AD
26A 27A 28C 29AC 30D
29. Osmotic fragility of red blood cells is 31ABC 32A 33A 34A
decreased in:
A. sickle cell anemia Answer Explanations:
B. hereditary spherocytosis 3. MCHC is the most reliable index because it
C. microcytic hypochromic anemia does not depend on the RBC count. This is
54
E.S.Prakash. MCQs in Medical Physiology, May 2013
because the estimation of RBC count is more error C have all been implicated in hypercoagulable
prone than the estimation of Hb or PCV. states.

8. ADP, TXA2, thrombin, serotonin, and 16. Factor V Leiden, a mutated form of Factor V,
epinephrine stimulate platelet aggregation. resists inactivation by protein C and is present in a
Prostacyclin inhibits platelet aggregation. Aspirin large subset of patients with venous
therapy lowers TXA2/PGI2. thromboembolism.

9. Vitamin K dependent clotting factors are 17. Fibrinogen is essential for platelet
factors II, VII, IX and X and Protein C and aggregation. Also fibrinogen is a clotting factor.
Protein S. Protein C and protein S inhibit clotting. Von Willebrand’s factor mediates the attachment
of platelets to subendothelial collagen. Also vWD
10. The extrinsic pathway is called so because it is binds factor VIII and prolongs its half-life in the
triggered by a factor extrinsic to plasma (tissue circulation. When there is a deficiency of vWF,
thromboplastin). Simply, it is triggered by “tissue more factor VIII spills over into the urine. Thus,
injury”. The extrinsic pathway is also the fastest bleeding time and clotting time are both increased
limb of the clotting cascade. Normally, PT = 12 – in hypofibrinogenemia and VWD. Thrombin is a
14 seconds. potent stimulator of platelet aggregation. At least,
theoretically, prothrombin deficiency would be
11. Fibrinolytic system: expected to delay platelet aggregation.
Plasminogen to plasmin
Enzyme: Tissue plasminogen activator (TPA) 19. If red cell antigens are also present in exocrine
Fibrin to fibrin degradation products secretions, such individuals are called secretors. In
Enzyme: Plasmin (fibrinolysin) some, they are not secreted. The significance of
this is unknown.
12. TFPI is tissue factor pathway inhibitor.
20. H substance is absent in persons with the
13. Prothrombin time screens the extrinsic limb Bombay phenotype. Normally, if an individual
and the final common pathway of the clotting has H substance, his blood group would be O. An
cascade. APTT screens the intrinsic limb of the individual lacking H substance will have anti H
clotting cascade. antibodies in plasma and can receive transfusions
only from a person with the Bombay blood group.
Test Assessing Factors
Bleeding time Platelets and 22. Heme oxygenase activity is highest in the
vessel wall spleen. Second, the spleen is much more sensitive
Clotting time Intrinsic & to red cell injury. [Wintrobe’s Clinical
Extrinsic Hematology, Lee GR et al, 10th ed, Lippincott
pathway Williams and Wilkins, volume 1, p 280.]
Prothrombin Extrinsic VII, X,
time pathway & Prothrombin,
common Fibrinogen
23. Haptoglobin binds extracorpuscular
pathway hemoglobin. Hemopexin binds heme. Haptoglobin
Activated Intrinsic VIII, IX, X, binds extracorpuscular hemoglobin and prevents it
partial pathway & XI, XII from being filtered and excreted by the kidney.
thromboplastin common Serum haptoglobin levels are reduced in
time pathway hemolytic anemias.

15. Loss-of-function mutations in protein C, 26. Albumin is a negative acute phase reactant;
protein S, antithrombin III and mutations in factor i.e., its synthesis is reduced during the acute phase
V resulting in resistance to inactivation to protein response.

55
E.S.Prakash. MCQs in Medical Physiology, May 2013
27. Presumably, the increase in ESR in anemia 8. Cardiovascular Physiology
has little to do with rouleaux formation; ESR may
be high because both PCV and viscosity are low List of acronyms that are used in this section:
which in turn reduce the resistance to HR – heart rate
sedimentation of cells. By the same reasoning, SBP – systolic blood pressure
ESR is reduced in polycythemia since the DBP – diastolic blood pressure
viscosity (and therefore the resistance to PP – pulse pressure
sedimentation) is high. However, if blood is more MAP – mean arterial pressure
viscous because of an increase in plasma levels of RPP – rate-pressure product
globulins and fibrinogen, ESR is higher since BPM – beats per minute
these proteins facilitate rouleaux formation. RR – RR interval
Q – cardiac output
29, 30. Osmotic fragility increases when the RBC TPR – total peripheral resistance
cytoskeleton is abnormal, as in hereditary TBF – tissue blood flow
spherocytosis.
Normal range: 0.5 - 0.3% NaCl Electrical properties of the heart
Increased in: hereditary spherocytosis 1. The heart continues to beat even after all nerves
Decreased in: microcytic anemia & sickle cell to it are sectioned. This property is called:
anemia. A. excitability
Sickle cells and microcytes show greater B. conductivity
resistance to osmotic lysis by virtue of their C. automaticity
smaller size and normal cytoskeleton. D. contractility

30. There is little fibrinogen in serum if at all. 2. Prepotentials are normally absent from:
A. P cells in the SA node
31. See p. 459, Ch 24, WFG, 2005. B. AV nodal cells
C. Purkinje fibers
32. Hereditary spherocytosis occurs due to D. working myocardial cells
mutations in cytoskeletal proteins spectrin,
ankyrin, Band 3. Defects in ankyrin are reported 3. What is the effect of vagal stimulation on the
to be commoner. membrane potential of the SA node?
A. It increases an inward calcium current
34. 70% of iron in the body is present in Hb. B. It increases the slope of the prepotential
Under abnormal circumstances, a large amount of C. It activates a hyperpolarizing potassium
iron may be present in hemosiderin. current
D. It increases intracellular cAMP
***************************************
4. Intrinsic heart rate can be determined by:
A. vagotomy
B. administration of atropine
C. beta adrenergic receptor blockade
D. IV administration of atropine and atenolol

5. The ST segment is elevated in acute myocardial


infarction because of:
A. flow of current into the infarct during diastole
B. TP segment depression
C. late depolarization of infarct
D. early repolarization of infarct

56
E.S.Prakash. MCQs in Medical Physiology, May 2013
6. Hypocalcemia is associated with QT 13. The maximum pressure rise in the ventricle
prolongation because: occurs during:
A. it is invariably associated with bundle branch A. ejection
block B. isovolumetric contraction
B. it increases ventricular activation time C. protodiastole
C. it lengthens the duration of ventricular D. diastasis
repolarization
D. it accelerates opening of potassium channels 14. During the cardiac cycle, aortic valve closes at
the end of:
7. AH interval is normally: A. isovolumetric systole
A. 30–50 ms B. rapid ejection
B. 60–125 ms C. diastasis
C. 100–200 ms D. protodiastole
D. 80–120 ms
15. The cardiac output of a 50 year old man at rest
8. The ability of the AV node to generate its own is 6 L / min; mean HR is 75 BPM. Left ventricular
impulses when the sinus node is “sick” is due to: end-diastolic volume (LVEDV) is 120 ml. What is
A. a constant phase 4 membrane potential the mean ejection fraction?
B. slow calcium entry during phase zero A. 35 %
C. spontaneous diastolic depolarization B. 50 %
D. the absence of prepotentials C. 66 %
D. 75 %
9. The AV node does not conduct more than:
A. 180 16. During exercise, a man consumes 2 liters of
B. 230 oxygen per minute. His arterial O2 content is 200
C. 280 ml/L and the oxygen concentration of mixed
D. 330 impulses / minute venous blood is 120 ml/L. His cardiac output is:
A. 16 L/min
Cardiac cycle; systolic time intervals B. 25 L/min
10. Which of the following need to be recorded in C. 32 L/min
order to determine systolic time intervals? D. 40 L/min
A. ECG, ECHO, PCG
B. ECG, PCG and carotid artery pulse (CAP) 17. Venous return is transiently increased during:
C. ECHO, CAP and PCG A. strain phase of Valsalva maneuver
D. ECG, CAP and apexcardiogram B. positive end-expiratory pressure
C. intravenous bolus of frusemide
11. Which of the following is not essential to D. deep inspiration
determine QS2, left ventricular ejection time
(LVET) and pre-ejection period (PEP)? 18. If blood [Hb A] = 15 g/dL and Hb is fully
A. Pulse transducer saturated with oxygen, average stroke volume =
B. Electrocardiograph 70 ml, average HR = 72/min, calculate whole-
C. Phonocardiogram body oxygen delivery; i.e., the amount of oxygen
D. Swan-Ganz catheter delivered to the tissues per minute assuming Hb is
fully saturated with oxygen.
12. The dicrotic notch is absent from: A. 1 L/min
A. radial arterial pulse tracing B. 0.5 L/min
B. pulmonary arterial pulse tracing C. 2 L/min
C. aortic pulse tracing D. 2.5 L/min
D. none of the above
The Microcirculation; Dynamics of blood flow
57
E.S.Prakash. MCQs in Medical Physiology, May 2013
19. Windkessel vessels are represented by the: C. skin
A. aorta D. brain
B. internal jugular v.
C. arterioles 27. Rank the hydraulic conductivity of capillaries
D. muscular arteries in various parts of the body.
A. GIT > glomerulus in kidney > lung > brain
20. The stopcocks of the circulation are: B. Glomerulus in kidney > brain > skin
A. arterioles C. GI mucosa > heart > brain
B. capillaries D. GI mucosa > circumventricular organs > lung
C. valves
D. venules Biophysics of the circulation:
28. Turbulence is almost always present when
21. Across which site in the circulation is the Reynolds number is more than:
pressure drop maximum? A. 2000
A. Arterioles B. 2500
B. Venules C. 3000
C. Capillaries
D. Aortic valve 29. The average arm-to-tongue circulation time is:
A. 5 seconds
22. Which of the following structures are not B. 10 seconds
innervated? C. 15 seconds
A. Arterioles D. 20 seconds
B. Postcapillary venules
C. AV anastomoses 30. Thin walled capillaries do not burst when
D. Precapillary sphincters intracapillary pressure is increased because:
A. they lack smooth muscle cells
23. At any time, the greatest fraction of blood B. the blood flow rate is less
volume is present in the: C. they have a small radius
A. heart D. capillary hematocrit is less than whole-body
B. arteries hematocrit.
C. veins
D. capillaries 31. That capillaries can withstand high internal
pressures without bursting is explained using:
24. What fraction of total blood volume is present A. Bernoulli’s principle
in the capillaries at any given time? B. Laplace’s law
A. 5% C. Poiseuille Hagen law
B. 20% D. Fahraeus-Lindquist effect
C. 15%
D. 1% 33. Bernoulli’s principle could be applied to
explain why:
25. The term “capacitance vessels” is applied to: A. intravesical pressure does not change with
A. pulmonary capillaries increase in intravesical volume
B. thoroughfare channels B. mean pressure in the femoral artery is greater
C. shunts than that in the brachial artery
D. veins and venules C. coronary blood flow is compromised in aortic
stenosis
26. Hydraulic conductivity of capillaries is highest D. tissue blood flow ceases below critical closing
in: pressure
A. glomeruli
B. intestinal villi Regulation of cardiovascular function:
58
E.S.Prakash. MCQs in Medical Physiology, May 2013
34. Quantitatively, the most important means of 3. The effect of head-up tilt would be to unload
increasing flow to an actively metabolizing tissue arterial baroreceptors in the carotid sinus.
is: 4. Herring’s nerve terminates in the nucleus
A. increasing cardiac output tractus solitarius.
B. increasing peripheral resistance 5. Activity in the arterial baroreceptors reflexly
C. increasing blood pressure excites vagal outflow to the heart and inhibits
D. decreasing local vascular resistance sympathetic outflow to resistance vessels.
6. The reflex effect of arterial baroreceptor
35. Which one of the following is not a loading is a decrease in HR and MAP.
vasodilator metabolite? 7. The reflex response to baroreceptor unloading
A. Adenosine is a decrease in sympathetic outflow to the
B. Potassium ions heart and blood vessels.
C. Endothelin-1
D. Hydrogen ions 41. Intravenous injection of norepinephrine to a
E. ADP normotensive healthy adult human leads to:
A. an increase in BP & HR
36. What is the chemical identity of endothelium- B. a decrease in BP & HR
derived relaxing factor (EDRF)? C. an increase in HR & decrease in BP
A. Nitrous oxide D. an increase in BP & decrease in HR
B. Nitric oxide
C. Potassium 42. The first reflex response to an increase in
D. Carbon monoxide arterial pressure produced by intravenous
injection of phenylephrine is:
37. Which one of the following does not have A. increase in RR interval
vasodilator actions? B. increase in heart rate
A. NO C. decrease in cardiac output
B. CO D. decrease in total peripheral resistance
C. Potassium
D. Angiotensin III 43. Which of the following maneuvers evokes an
increase in vagal discharge to the heart?
38. Blood flow to exercising skeletal muscle is A. IV infusion of phenylephrine
increased by all of the following except: B. Carotid massage
A. K+ C. Pressure on the eyeball
B. norepinephrine D. Irrigation of the ear canals
C. ↑ in PCO2 in muscle
D. adenosine 44. On rising from the supine position:
A. central blood volume increases
39. The most potent vasoconstrictor is: B. heart rate decreases
A. endothelin 1 C. central venous pressure decreases
B. angiotensin II D. discharge from arterial baroreceptors
C. norepinephrine decreases
D. vasopressin E. stroke volume decreases by about 40%

40. Check whether the following statements about 45. Marey’s law states that:
the arterial baroreflex mechanism are true or false. A. when BP increases, HR decreases
1. Baroreceptors are free nerve endings B. when BP decreases, HR increases
responsive to stretch. C. when BP decreases, HR decreases
2. Application of suction force with a neck D. when BP increases, HR increases
chamber loads the arterial baroreceptors.

59
E.S.Prakash. MCQs in Medical Physiology, May 2013
46. An increase in whole body oxygen demand is B. cardiac output decreases
met chiefly by: C. blood pressure decreases
A. increasing cardiac output D. heart rate increases
B. increasing oxygen content of arterial blood E. sympathetic outflow to blood vessels ↑
C. increasing oxygen extraction from arterial F. TPR gradually increases
blood
D. increasing blood pressure 53. Atrial natriuretic peptide is metabolized by:
A. dicarboxypeptidase
47. During severe exercise, a well-trained athlete B. kininase II
may be able to achieve a cardiac output of: C. neutral endopeptidase
A. 15 liters D. kininase I
B. 25 liters
C. 35 liters 54. Cardiac output increases during:
D. 45 liters A. Valsalva maneuver
B. positive pressure ventilation
48. Vasomotor ischemia triggers an increase in C. negative ‘g’
sympathetic outflow increasing BP and thereby D. head-up tilt
facilitating restoration of cerebral blood flow.
This is called: 55. Which of the following is least likely
A. Bainbridge reflex following prolonged space missions?
B. the CNS ischemic pressor response A. Cardiac hypertrophy
C. Head’s paradoxical reflex B. Postural hypotension
D. Marey’s reflex C. Weight loss
D. Motion sickness
49. Heart rate is slowed by:
A. deep inspiration Regional circulation:
B. Bainbridge reflex 56. The arteriovenous O2 concentration difference
C. increased intracranial tension is highest across the:
D. carotid massage A. brain
B. heart
50. In a healthy normotensive individual at rest, C. kidneys
heart rate is typically increased by: D. liver
A. deep expiration
B. fear 57. The mechanism that regulates cerebral blood
C. anger flow during cerebral compression is the:
D. IV infusion of phenylephrine A. CNS ischemic response
B. Cushing’s reflex
51. Which of the following varies from beat-to- C. Bezold-Jarisch reflex
beat? D. Bainbridge reflex
A. RR interval
B. Venous return 58. Activity in the noradrenergic nerves to the
C. Stroke volume heart causes / has:
D. Cardiac output A. coronary vasoconstriction
E. Blood pressure B. coronary vasodilation
C. no effect on blood flow
52. During the strain phase of the Valsalva
maneuver (forced expiration with the glottis open 59. If the noradrenergic nerves to the heart are
and maintaining an expiratory pressure of 40 mm stimulated after giving a β-blocker, then what
Hg for 15 seconds): would be the effect on coronary blood flow?
A. venous return decreases A. Coronary vasodilation
60
E.S.Prakash. MCQs in Medical Physiology, May 2013
B. Coronary vasoconstriction 51all 52all 53C 54C 55A
C. No change 56B 57AB 58B 59B 60B
D. Unpredictable 61B 62C 63A 64all
40: statement 7 is false; others are true
60. Capillaries empty when mechanically
stimulated. This is called the: Answer Explanations:
A. axon reflex 1. The automaticity is attributed to the presence of
B. white reaction pacemaker cells that demonstrate spontaneous
C. flare response depolarization in the absence of extrinsic
D. red reaction innervation. See below.

61. The wheal in the triple response is due to: 2. Note that the terms prepotential, pacemaker
A. contraction of precapillary sphincters potential and spontaneous diastolic depolarization
B. increased capillary permeability are often used interchangeably. They all mean the
C. axon reflex same. Normally prepotentials are present only in
D. decreased absorption of fluid the SA node and AV node which contain
pacemaker cells (P cells).
62. Which of the following physiologic responses
has a neural basis? However, in abnormal situations (e.g. hypoxemia)
A. Red reaction other regions of the heart (e.g. a ventricular focus)
B. White reaction exhibit prepotentials. This state is one of
C. Flare “increased automaticity”, and premature
D. Reactive hyperemia ventricular or atrial depolarizations are
manifestations of increased automaticity.
63. Of the following vascular beds, autoregulation 3. Stimulation of the right vagus nerve, which
of tissue blood flow is least prominent in the: predominantly innervates the SA node, decreases
A. skin the firing rate of the SA node. The effect of
B. heart acetylcholine on pacemaker cells in the SA node
C. brain is to activate a hyperpolarizing potassium current.
D. kidneys
4. Intrinsic heart rate (IHR) is the rate at which the
64. When determining BP with a heart will beat when completely denervated. This
sphygmomanometer, a spuriously high value (of is determined, in humans, by intravenous
either SBP or DBP) may be recorded when: administration of a standard dose of atropine and
A. there is an auscultatory gap atenolol. In healthy humans, IHR is about 100-
B. the cuff is smaller than preferable 110/min. The magnitude of IHR reflects the
C. the person is obese automaticity of the SA node. The fact that resting
D. the arm is not placed at the level of the heart heart rate in healthy humans is around 70 beats
per minute indicates that the effects of cardiac
Answers: vagal tone at rest are greater than the effect of
1C 2CD 3C 4D 5all sympathetic outflow to the heart. Patients with
6C 7B 8C 9B 10B transplanted hearts have higher resting heart rates
11D 12D 13B 14D 15C closer to IHR because of cardiac denervation.
16B 17D 18A 19A 20A Also, because of denervation, they are less able to
21A 22D 23C 24A 25D increase their heart rate in response to exercise.
26A 27C 28C 29C 30C
31B 33C 34D 35C 5. The infarct is deprived of blood supply and
36B 37D 38B 39A 40 consequently its RMP becomes less negative.
41D 42A 43all 44CDE 45AB What do you expect the effect of ATP depletion to
46A 47C 48B 49D 50C be on RMP? RMP will become less and less
61
E.S.Prakash. MCQs in Medical Physiology, May 2013
negative. The other reason why RMP becomes QS2 is the time duration from the beginning of
less negative is that potassium is lost from cells as the Q wave to the first high frequency component
a result of injury. Thus, the infarcted zone loses its of the aortic component A2 of the second heart
surface positivity and consequently it is negative sound. Note that this is an electromechanical
with respect to surrounding normally polarized interval; QS2 is called electromechanical systole.
tissue (surface positive, inside negative). It is remarkably constant.
Therefore, during diastole, extracellularly, current
flows into the infarct. This depresses the baseline, Left ventricular ejection time (LVET) is the
i.e., the TP segment is depressed. However, the period from the upstroke of the carotid artery
arrangement in ECG recorders is such that TP pulse to the dicrotic notch (an oscillation on the
segment depression is recorded as ST segment falling wave of the carotid artery pulse). The
elevation. dicrotic notch in the carotid artery pulse tracing
represents the closure of the aortic valve. Note
Secondly, the infarct depolarizes late with respect that ejection is a part of systole.
to surrounding normal tissue probably due to
decrease in conduction velocity in the infarcted Pre-ejection period (PEP) is calculated as QS2 –
tissue; the effect of this late depolarization is to LVET. It denotes the time taken for electrical
cause ST segment elevation. excitation of left ventricle, excitation-contraction
coupling and isovolumetric ventricular
Finally, ischemic myocardium repolarizes faster contraction.
due to accelerated opening of K channels. The
effect of early repolarization is also ST segment If the duration of ventricular excitation and
elevation. Thus, myocardial infarction is excitation-contraction coupling can be assumed to
characterized by ST segment elevation in leads be constant, then, PEP can be taken to reflect the
facing the infarct. See p 563-4, Ch 28, WFG, duration of isovolumetric ventricular contraction
2005. (IVVC).

7. The AH interval, from the A wave to the start PEP, which reflects the duration of isovolumetric
of the H spike is normally 60-125 ms. contraction, is prolonged in heart failure. This is
also associated with a decline in left ventricular
His Bundle Electrogram: ejection time. Thus, PEP/LVET ratio is a sensitive
Interval Physiologic Time index of left ventricular systolic performance. STI
correlate (ms) cannot be used for diagnosis of specific heart
PA interval Atrial 30 conditions. Of late, STI have been superseded by
activation time echocardiography.
AH AV nodal delay 60-125
interval
13. See Fig 29-2 on p.566, Ch 29, WFG, 2005
HV His Bundle - 35-50
interval ventricle 15. Cardiac output = SV × HR
HR = 75/min; cardiac output = 6 L/min.
The AH interval represents AV nodal conduction Therefore, average SV = 80 ml.
time. See p. 554, Ch 28, WFG, 2005. Average LVEDV = 120 ml.
Therefore, ejection fraction = SV/LVEDV = 0.66
8. The AV node, like the SA node, exhibits = 66%.
prepotentials but its rate of discharge is much less
compared to the SA node. 16. The Fick’s principle states that the amount of
a substance (X) consumed by an organ per unit
9. See p. 557, Ch 28, WFG, 2005. time (A) = A-V conc. difference of X across that
organ (circuit) × blood flow through that organ (or
10, 11, 12. Systolic time intervals (STI) circuit)
62
E.S.Prakash. MCQs in Medical Physiology, May 2013
that contribute to the reduction in coronary blood
Thus, blood flow (Q) = A / A-V conc. diff flow in aortic stenosis.
In this example, O2 consumption = 2000 ml/min.
AV O2 difference = 80 ml/L. Cardiac output = 34. Tissue blood flow = BP / local vascular
2000/80 = 25 L/min. resistance. Quantitatively, changes in local
vascular resistance achieved mainly through local
18. Cardiac output = 5 L/min; Blood Hb = 15 autoregulatory mechanisms contribute the most to
g/dL. Oxygen carrying capacity of blood is regulating tissue blood flow.
approximately 20 ml/dL i.e. 200 ml/L.
39. See p. 599, Ch 31, WFG, 2005
Whole body oxygen delivery = cardiac output ×
oxygen content of arterial blood. 41. Norepinephrine has a greater affinity for alpha
adrenergic receptors than beta adrenergic
Therefore about 1L of oxygen will be delivered to receptors. So when it is administered
the tissues per minute. intravenously to a healthy individual with normal
BP, it elicits a rise in total peripheral resistance
23. At any time, 54% of blood volume is because of alpha adrenergic receptor mediated
contained in the veins and venules & vena cava. vasoconstriction. The rise in BP elicits a
baroreflex mediated lowering of HR that
26. See Table 30-2, p. 579, Ch 30, WFG, 2005. overwhelms the direct cardioacceleratory effect of
norepinephrine.
30. For a cylindrical structure, transmural pressure
(P) = T/r (Laplace’s law) where T is wall tension 43. Phenylephrine is an alpha adrenergic agonist,
and r is radius so it raises TPR and BP and elicits a baroreflex
mediated lowering of HR. Carotid massage
Though capillaries are thin walled, they have a mechanically activates the vagus nerve contained
smaller radius and consequently need to develop within the carotid sheath increasing cardiac vagal
less tension in order to withstand a given outflow.
distending (transmural) pressure.
44. Central venous pressure decreases upon rising
The statement in the question should not mislead due to a decrease in venous return. A decrease in
you into believing that capillaries can withstand a arterial pressure, which occurs on rising from the
pressure of 100 mm Hg – they will definitely supine position, also decreases the stretch of
rupture at that pressure. Rather if we compared a arterial baroreceptors (i.e., arterial baroreceptors
larger vessel and a capillary distended by are said to be ‘unloaded’) and the firing rate in the
physiologic pressures (say 40 mmHg), one still carotid sinus nerves decreases. This reflexly
wonders how the thin walled capillary is able to inhibits vagal outflow to the heart and increases
withstand it. That is because it also has a smaller sympathetic outflow from the brain resulting in an
radius. increase in heart rate and a rise in total peripheral
resistance.
33. Bernoulli’s principle: The greater the velocity
of flow in a vessel, the lower the lateral pressure 45. Marey’s law states that HR (the dependent
distending its walls. variable) is inversely related to BP (the
independent variable) and that the converse is not
Note coronary arteries originate virtually at a right true.
angle from the aorta, above the aortic valve. Thus,
in aortic stenosis, the lateral pressure at the level Marey’s law is only a restatement of negative
of the origin of coronary arteries is reduced, and feedback regulation of BP by the arterial
this in turn reduces coronary blood flow. baroreflex; i.e., when BP increases, baroreflex–
However, this is only one of many mechanisms mediated lowering of HR occurs. When BP
63
E.S.Prakash. MCQs in Medical Physiology, May 2013
decreases, HR reflexly increases. This is a Reflex Cushing’s CNS ischemic
mechanism to rapidly regulate and maintain BP reflex pressor
within a normal range. response
Stimulus Raised Vasomotor
There are several exceptions to this: intracranial ischemia due
tension to any cause
causing (example,
1. During exercise, BP and HR both increase. vasomotor hypotension)
The goal here is to allow cardiac output to ischemia as a
increase and deliver more oxygen to actively result of
metabolizing tissue, and it is logical to raise compression
BP rather than maintain it at resting levels. of brain cells
Response Increased Increased
2. Vasomotor ischemia occurring in the face of sympathetic sympathetic
hypotension triggers an increase in outflow outflow
sympathetic outflow (CNS ischemic response) Effect Increase in Increase in BP
leading to an increase in BP but this is not BP. If the & HR toward
accompanied by HR lowering. individual had normal
normal BP at
baseline, his
3. Sometimes, when MAP is less than 50 mm BP would
Hg, activation of arterial chemoreceptors leads become higher
to a vagally mediated bradycardia. HR change If BP Increases
increases (since when
46. At rest, blood flow through the systemic above normal MAP is too
circulation (i.e., cardiac output) = 5 L/min (say from 120 low the
Assuming that functional Hb A concentration is mm Hg to 140 arterial
15 g/dL and PaO2 is 100 mm Hg and that Hb is mm Hg), then baroreflex
fully saturated with oxygen, a baroreflex mechanism
Oxygen content of arterial blood = 200 ml/L mediated does not
Whole body oxygen delivery = 1 L/min bradycardia operate)
would occur
Whole body oxygen consumption = 250 ml/min
Significance A mechanism A mechanism
Whole body oxygen extraction = 0.25 to eventually to eventually
This is sometimes called ‘oxygen utilization maintain maintain
coefficient (OUC)’. cerebral blood cerebral blood
Theoretically, oxygen extraction can increase up flow flow
to 1. Thus, an increase in oxygen extraction is one
mechanism of fulfilling an increase in oxygen 49. Note that raised intracranial tension (ICT) is
demand. not necessarily associated with bradycardia. An
However, in trained athletes, cardiac output can individual with raised ICT may be hypotensive
be increased 7 times from its resting value. due to blood loss. In this instance, hypotension is
usually associated with tachycardia.
47. See Table 29-3, p. 572, Ch 29, WFG, 2005.
Rather, if raised ICT leads to hypertension, this
48. Vasomotor ischemia stimulates sympathetic usually elicits a baroreflex mediated lowering of
outflow. This leads to an increase in BP that in heart rate.
turn serves to restore cerebral blood flow. This is
the CNS ischemic pressor response. 52. Valsalva maneuver:
Maneuver Forced expiration against a
Cushing’s reflex vs. CNS ischemic response closed glottis
Reflex Cushing’s CNS ischemic Stimulus Decrease in BP due to a
reflex pressor decrease in venous return and
response consequently stroke volume
64
E.S.Prakash. MCQs in Medical Physiology, May 2013
(SV) blood flow. If that cannot happen, myocardial
Response Immediate increase in HR and ischemia would result with predictable
a more gradual increase in consequences.
TPR
Goal To maintain BP At rest, oxygen extraction ratio in the heart
Immediately Venous return suddenly = (A–V O2 difference / arterial O2 content) × 100
after the increases, SV increases, TPR
= (114 / 200) × 100 = 0.55
maneuver is already high. Thus BP =
SV × HR × TPR ‘overshoots’
This is based on data in Table 32-1, p. 612, WFG,
Response The BP overshoot triggers a 2005.
baroreflex mediated lowering
of HR 58, 59. Activity in the noradrenergic nerves to the
Significance BP maintained heart increases myocardial oxygen demand since
it would increase the force as well as rate of
cardiac contraction. However, this normally
54. During negative ‘g’, i.e. acceleration in the results in coronary vasodilation caused by
long axis of the body from foot to head, there is a products of metabolism. The pressor effect of
headward shift of body fluids and central blood norepinephrine on alpha-adrenergic receptors in
volume, cardiac output and blood pressure the coronary arteries is not manifest therefore.
increase. When intrathoracic pressure is raised as However, the direct effect of norepinephrine on
occurs during the Valsalva maneuver, venous segments of isolated coronary arteries is
return reduces and cardiac output decreases. vasoconstriction. On the other hand, the increase
Similarly, during positive pressure ventilation, the in myocardial oxygen demand (and consumption)
increase in ITP reduces venous return and during exercise is attenuated in individuals taking
decreases cardiac output. Passive head-up tilt is beta-blockers because the norepinephrine
associated with a decreased cardiac output since mediated rise in heart rate and contractility are
venous return is reduced. also diminished, and consequently their ability to
exercise is diminished. The learning point here is
55. Cardiac atrophy rather than hypertrophy that the coronary circulation is capable of
occurs following prolonged space missions since excellent autoregulation.
the load on the heart is reduced. This is because
skeletal muscles are not regularly used in 60-63. The triple response:
microgravity conditions. Postural hypotension Response Mechanism
occurs upon return to earth (where the effects of Wheal production Increased capillary
earth’s gravity are again fully manifest) because and venular
of autonomic as well as physical deconditioning. permeability
Motion sickness in space occurs due to conflicting Red reaction Venodilation
Flare Axon reflex
neural inputs from visual cues and vestibular
system and diminished input from muscle The white reaction is the response to a benign
proprioceptors. stimulus. It occurs due to contraction of
precapillary sphincters.
56. If the arteriovenous concentration difference
of a substance (example, oxygen) across a The triple response is evoked by a noxious
vascular bed is high, it means that the vascular stimulus.
bed extracts a large fraction of this substance.
Oxygen extraction ratio across the heart is 0.5-0.7 With the exception of the flare which is mediated
at baseline, higher compared to other tissues. by an axon reflex, the triple response, indeed, is
Cardiac venous oxygen tension is low and little an example of autoregulation.
additional oxygen can be extracted from the blood
in the coronaries, so increases in myocardial However, we might say that autoregulation of
oxygen consumption require increases in coronary tissue blood flow is least prominent in the skin;
65
E.S.Prakash. MCQs in Medical Physiology, May 2013
this is because, cutaneous blood flow is mainly An increase in stroke volume for a given preload
regulated by neural signals from the and afterload is due to an increase in myocardial
hypothalamus; this is logical since skin serves as contractility. This is sometimes called
the interface across which heat exchange is ‘homometric regulation’ (meaning same length).
regulated to maintain body temperature. In contrast, heterometric regulation refers to
changes in stroke volume brought about by
64. All four options are correct. When there is an changes in end diastolic volume.
auscultatory gap and the palpatory method is not
used, systolic pressure will be underestimated. The best index of myocardial contractility is the
When there is an auscultatory gap and you use the rate of pressure rise (dP/dt) in the ventricle during
palpatory method, you will get a correct estimate isovolumetric contraction. However, clinically,
of systolic pressure (say 210 mm Hg). Sounds will the ejection fraction is used as a surrogate of
cease at some point (say 160 mm Hg). However, myocardial contractility.
if the cuff is not deflated any further (because
sounds disappeared at 160), one may not realize Myocardial contractility is markedly enhanced by
that there is an auscultatory gap with the result sympathetic stimulation, digoxin and depressed by
that diastolic pressure is overestimated. myocardial ischemia, acidosis.
Something like 210/160 mm Hg will be recorded
when the actual pressure is 210/90 mm Hg. Inotropic effects: the term refers to the effect of a
_________________________________ stimulus on myocardial contractility.
Supplementary Material:
What is BP determined by?
Effect of load on muscle contraction: BP = cardiac output × total peripheral resistance
Within physiologic limits, the energy of cardiac
contraction is directly proportional to preload. Blood pressure indexes:
This is the Frank-Starling law of the heart. Systolic pressure (SBP): the highest pressure in
the arteries during the cardiac cycle.
Preload: The load on a muscle before it contracts.
In the ventricle, the preload (end-diastolic fiber Diastolic pressure (DBP): the lowest pressure in
length) varies directly with the end-diastolic the arteries during the cardiac cycle.
volume.
Pulse pressure: SBP – DBP
Afterload: The load which contracting muscle
has to overcome before it shortens. The velocity Mean arterial pressure = diastolic pressure + 1/3
of shortening varies inversely with afterload. For (pulse pressure). However, this applies only when
the left ventricle, the afterload is the total systemic HR is in the 60-90 BPM range. MAP is actually
vascular resistance. determined by integrating the arterial pressure
curve.
Cardiac output: The volume of blood ejected by
the left or the right ventricle per minute. MAP is also equal to (SP + 2DP)/3

Cardiac output = stroke volume × heart rate Rate-pressure product (RPP): It is calculated as
the product of SBP and HR and generally divided
Cardiac index is the cardiac output per square by 100 to get a smaller number. If SBP is 120 mm
meter of body surface area. Hg and HR is 80 BPM, then RPP is 96 mm Hg
BPM 10-2
Regulation of stroke volume:
Stroke volume is influenced by preload, afterload Physiologic correlates of the blood pressure
and myocardial contractility. indexes:

66
E.S.Prakash. MCQs in Medical Physiology, May 2013
Diastolic pressure reflects the total resistance two subtypes of receptors of
offered to peripheral run off of blood (specifically, which is adapting and the other
the resistance offered by the arterioles). nonadapting.
Response to an An increase in BP leads to a
When blood is ejected into the arteries, arterial increase in BP decrease in cardiac output and
blood pressure increases from about 80 mm Hg to TPR through cardiac vagal
excitation and inhibition of
about 120 mm Hg. sympathetic outflow
Response to a Sympathetic outflow from the
Ejection of blood produces an increment in decrease in BP medulla is “disinhibited”; and
arterial blood pressure; this is called pulse vagal outflow to the heart is
pressure. disinhibited.
Response time: It takes only about 1 second for a
Pulse pressure depends upon two factors: change in BP to result in a
Volume of blood ejected (stroke volume) and compensatory change in HR; it
arterial compliance. If arteries are thick and rigid takes a little longer (about 10
(and thus less compliant), a given stroke volume seconds) for changes in TPR to
produces a greater rise in pulse pressure. occur following a primary change
in BP
It is often stated that pulse pressure = SBP– DBP. Isotonic versus isometric exercise:
However, conceptually, systolic pressure = Isotonic (dynamic) Isometric (static)
diastolic pressure + pulse pressure exercise; example, exercise; example,
running sustained isometric
RPP is a useful index because it reflects handgrip
myocardial O2 demand. Greater the HR, greater BP increases mainly due BP increases primarily
the myocardial oxygen demand; to generate a to an increase in cardiac due to an increase in TPR
greater systolic pressure, greater wall tension output
needs to develop (according to Laplace’s law) and TPR may fall in severe Cardiac output may
this also increases oxygen consumption. exercise because of reduce or not change
accumulation of significantly from
vasodilator metabolites baseline as a result of the
The arterial baroreflex mechanism for
baroreflex mechanism.
regulating blood pressure:
Inputs Mean arterial pressure, pulse ***************************************
pressure
Receptor Called “arterial baroreceptors”
Location of Adventitia of the carotid sinus
receptors and aortic arch
Receptor type Free nerve endings
Adequate Stretch (both tonic as well as
stimulus phasic response)
Afferent Via IX (from the carotid sinuses)
and X nerves (from the aortic
arch)
Termination of NTS in medulla
first order
neurons
Receptor MAP between 70 and 110 mm Hg
operating range
Receptor Increase in static (mean arterial
characteristics pressure) as well as phasic stretch
(pulse pressure) increase firing
rate; decrease in stretch decreases
firing rate. There are presumably
67
E.S.Prakash. MCQs in Medical Physiology, May 2013
9. Pulmonary Physiology Capacity Volume
(ml)
Respiratory minute volume and alveolar Functional residual RV + ERV 2300
ventilation: capacity (FRC)
Inspiratory capacity VT + IRV 3700
Respiratory minute volume = VT × respiratory (IC)
Vital capacity VT + IRV + 4800
rate
ERV
TLC RV + ERV + 6000
Alveolar ventilation = VT + IRV
(VT – anatomical dead space) × respiratory rate
Note that the term expiratory capacity is not used.
Anatomical dead space is roughly equal to body Unless otherwise stated, vital capacity is the
weight in pounds. maximum amount of air that can be expelled after
a forced inspiration. If measured during
If VT = 500 ml and respiratory rate = 12 per inspiration, it is specifically called inspiratory
minute, then respiratory minute volume = 6 liters vital capacity.

If the anatomical dead space = 150 ml, then Testing mechanics of respiration:
alveolar ventilation = (500 – 150) × 12 = 4.2 What are the factors affecting vital capacity?
liters/minute. Note that only alveolar ventilation 1. Strength of muscles of inspiration
could contribute to gas exchange. 2. Lung compliance
3. Strength of muscles of expiration
Lung volumes and capacities: 4. Airways resistance
Each lung volume is an independent fraction of
total lung capacity (TLC). Lung capacities are Evaluating airways resistance:
combinations of lung volumes. Values given Examine the relationship between air flow,
below are for a healthy young adult male airways resistance and airway pressure before you
weighing 80 kg with a body surface area of about consider evaluating airways resistance.
1.8 m2. Values in females are about 10% lower.
Note that lung volumes and capacities are greatly Air flow = transairway pressure / airway
influenced by height, weight, body surface area. resistance;

Tidal volume VT 500 ml Transairway pressure = intrathoracic pressure –


Inspiratory reserve volume (IRV) 3200 ml pressure at the mouth (which is zero mm Hg with
Expiratory reserve volume (ERV) 1100 ml reference to atmospheric pressure)
Residual volume (RV) 1200 ml
FEV1, the volume of air expelled during the first
Tidal volume is about 5–7 ml/kg body weight; second of a forced vital capacity maneuver, varies
directly with expiratory pressure and inversely
By definition, the reserve volumes are recruited with airways resistance. It is normally at least
with effort. 80% of FVC; FEV1/FVC is > 0.8.
FRC is the resting lung volume or the volume of Note that if vital capacity is reduced, FEV1 will
air present in the lungs at the end of a tidal be reduced, still FEV1/FVC may be normal.
expiration. It is the lung volume at which the
inward elastic recoil of lung parenchyma is For example, in restrictive lung disease, FEV1 is
balanced by the natural tendency of chest wall to reduced because vital capacity is reduced;
recoil outward. however, FEV1/FVC is normal because airways
resistance is normal.
Lung capacities in a healthy adult male
weighing 70 kg:
68
E.S.Prakash. MCQs in Medical Physiology, May 2013
Thus, FEV1/FVC is reduced when airways capacity
resistance is increased. Weakness of muscles of Myasthenia gravis,
respiration poliomyelitis
Peak expiratory flow rate (PEFR) is the highest Lung compliance “Restrictive” lung
flow rate during forced expiration (vital capacity reduced disease
maneuver). Note that when flows are reduced, Increased airways Major airway
resistance obstruction; bronchial
flow rates are reduced. Thus, for example, the
asthma
peak flow rate during tidal expiration is much
lower than that during forced expiration. In summary,
Normally PEFR is about 12 liters/second or 720 FEV1 FVC FEV1/FVC PEFR
liters/minute (values are 10-15% lower in age OLD* ↓ ↓ < 80% ↓
matched females). RLD* ↓ ↓ Normal or ↓
> 80%
PEFR is dependent upon vital capacity. If vital *OLD & RLD are obstructive & restrictive lung
capacity is reduced, PEFR is also reduced. Thus, disease respectively.
it does not make sense to use PEFR as an index of
airways resistance in a patient with restrictive “Air trapping” occurs whenever there is
lung disease. expiratory flow limitation. Since flow is driven by
pressure and opposed by resistance, air trapping is
So, PEFR must be used as an index of airways usually a consequence of an increase in airways
resistance only in patients known to have resistance. Air trapping occurs in chronic
obstructive airways disease (low FEV1/FVC) obstructive pulmonary disease. As a result of this,
because FEV1/FVC is normalized for flow total lung capacity increases, however the vital
whereas PEFR is not. capacity is subnormal; in other words, patients
with obstructive lung disease breathe at a higher
Forced expiratory time: this is a useful bedside FRC compared to normal subjects. The fraction of
index of airways resistance. Auscultate the trachea FRC that increases is the residual volume.
during forced expiration, if expiratory sounds are
heard for longer than 4 seconds, airways Below, there are questions here and there that
resistance is increased. have multiple correct answers.

Maximum mid-expiratory flow rate Questions:


(MMEFR): The first 25% of expired air comes 1. The volume of air that moves in or out during
mainly from the major airways; MMEFR is the each tidal breath is called:
highest rate at which the middle 50% of VC is
expelled; it is also called forced expiratory flow 2. The amount of air present in the lungs at the
rate (FEF 25 – 75). This reflects the resistance of the end of a tidal breath is:
small airways that are narrowed in bronchial A. residual volume
asthma. B. expiratory reserve volume
C. functional residual capacity
Using vital capacity as an index of pulmonary
function: Vital capacity is always reduced in lung 3. The largest lung volume is:
disease; however, total lung capacity may be A. residual volume
increased or decreased depending on the cause. B. inspiratory reserve volume
However, vital capacity is not a sensitive index of C. expiratory residual volume
pulmonary function. For example, a person can D. tidal volume
have a subnormal FEV1/FVC and yet his vital
capacity could be normal. 4. By spirometry, one cannot determine:
A. vital capacity
Mechanism of Disease B. functional residual capacity
reduction of vital C. residual volume
69
E.S.Prakash. MCQs in Medical Physiology, May 2013
D. total lung capacity A. respiratory minute volume
B. minute ventilation
5. Functional residual capacity can be determined C. maximal voluntary ventilation
using: D. vital capacity
A. spirometer
B. computerized spirometry 13. In a healthy adult male, maximum voluntary
C. helium dilution technique ventilation is about:
D. whole body plethysmography A. 60-80 L/min
B. 125-170 L/min
6. A person breathes into and from a spirometer C. 200-220 L/min
(volume 12 liters) containing 10% helium gas D. 90-100 L/min
mixture. After equilibration, helium concentration
of expired gas was found to be 6.7%. His vital 14. Which is the major muscle of inspiration?
capacity is 4.2 liters. What is his residual volume?
A. 1000 ml 15. Which muscle does not contract during forced
B. 1200 ml expiration?
C. 1500 ml A. Internal intercostals
D. 1800 ml B. External intercostals
C. Transversus abdominus
7. In a healthy individual with a total lung volume D. Rectus abdominis
of 6 liters, the amount of oxygen present in the
lungs at the end of a normal expiration is about: 16. The normal compliance of the human lungs
A. 400 ml and chest wall is:
B. 1000 ml A. 0.1 L/cm H2O
C. 100 ml B. 0.2 L/cm H2O
D. 250 ml C. 0.3 L/cm H2O
D. 0.4 L/cm H2O
8. The maximum amount of gas that can be
exhaled after a full inspiration is called: 17. Resting lung volume would be lesser than
A. expiratory reserve volume FRC if it were not for the presence of:
B. vital capacity A. surface tension forces
C. total lung capacity B. surfactant
D. functional residual capacity C. elastic recoil forces

9. Vital capacity correlates inversely with body 18. In which pathologic condition is lung
surface area. True / false? compliance increased?

10. Vital capacity is greater in the upright posture 19. Normally, during quiet breathing, intrapleural
compared to supine posture. True / false? pressure is negative during inspiration as well as
expiration. True/False?
11. Residual volume is normally about ____% of
total lung capacity. 20. The major constituent of surfactant is:
A. 10 A. neutral lipid
B. 20 B. dipalmitoyl phosphatidyl choline
C. 30 C. phosphatidylglycerol
D. 40 D. surfactant proteins A & D

12. The largest volume of gas that can be moved 21. Intrapleural pressure is positive during:
into and out of the lungs in 1 minute by voluntary A. deep inspiration
effort is called: B. tidal expiration
70
E.S.Prakash. MCQs in Medical Physiology, May 2013
C. forced expiration A. vital capacity
D. tidal inspiration B. FEV1
C. FEV1/FVC
22. Which of the following produce D. PEFR
bronchodilation? E. MMEFR
A. VIP
B. Epinephrine 30. In chronic obstructive lung disease:
C. Leukotriene C4 A. FRC increases
D. PAF B. TLC increases
C. VC decreases
23. Bronchodilation is mediated by: D. RV increases
A. α1 receptors
B. α2 receptors 31. Which of the following statements is correct?
C. M2 receptors A. Normally, ventilation is greater at the apex
D. β2 adrenoceptors than at the lung bases
B. In the upright position, lower lung zones are
24. Bronchial tone is highest at about: more compliant compared to upper lung zones
A. 6 AM C. In the upright position, blood flow is greater at
B. 10 AM the lung bases than at the apices
C. 2 PM D. Positive intrapleural pressures during forced
D. 6 PM expiration prevent airways from closing.

25. Which of the following produces 32. Anatomic dead space is determined using:
bronchoconstriction? A. whole body plethysmography
A. VIP B. spirometry
B. PAF C. single-breath nitrogen curve
C. LTB4 D. single-breath carbon monoxide method
D. Epinephrine
E. Nitric oxide 33. The pattern of ventilation in lungs is assessed
by:
26. Which of the following is least likely to A. Kety method
produce bronchoconstriction? B. inhalation of radioactive xenon
A. NO C. angiography
B. Sulfur dioxide D. single breath technique
C. Cool air
D. Exercise 34. Under basal conditions, the amount of oxygen
consumed per minute (in a healthy adult weighing
27. The FEV1/FVC ratio is normally greater than: 70 kg) is about:
A. 0.8 A. 100 ml
B. 0.7 B. 250 ml
C. 0.9 C. 350 ml
D. 0.5 D. 500 ml

28. Peak expiratory flow rate (PEFR) is reduced 35. Under basal conditions, the respiratory
in: quotient (i.e., the volume of CO2 produced /
A. restrictive lung disease volume of oxygen consumed) is about:
B. bronchial asthma A. 1
B. 0.8
29. The most sensitive index of small airways C. 0.7
resistance in a patient with bronchial asthma is: D. 1.2
71
E.S.Prakash. MCQs in Medical Physiology, May 2013
36. Under basal conditions, respiratory exchange 43. In the upright position, the V/Q ratio at the
ratio is normally: lung apex is about:
A. 0.7 A. 1
B. 0.8 B. 0.7
C. 0.9 C. 0.5
D. 1.0 D. 3

37. Under basal conditions, in a person on a 44. Arterial blood contains more oxygen than
mixed diet, respiratory quotient is about: carbon dioxide. True / false?
A. 0.7
B. 0.8 45. Venous blood is normally 40% saturated with
C. 0.9 oxygen. True / false?
D. 1.0
46. The sigmoid shape of the oxygen-hemoglobin
38. Alveolar oxygen tension (PAO2) is influenced dissociation curve is due to:
by: A. allosteric effects
A. barometric pressure B. steric effects of oxygen binding on the
B. fraction of oxygen in inspired air (FiO2) configuration of Hb
C. PACO2 C. the effects of 2,3–BPG on oxygen affinity of
D. respiratory exchange ratio Hb

39. At rest, the normal diffusing capacity of the 47. The major buffer of carbondioxide in blood is:
lungs for carbon monoxide is about: A. bicarbonate
A. 25 ml/min/mm Hg B. albumin
B. 50 ml/min/mm Hg C. hemoglobin
C. 75 ml/min/mm Hg
D. 100 ml/min/mm Hg 48. If blood [Hb] were 15 g/dL, how much
oxygen is contained in arterial blood?
40. The mean systemic arterial pressure and the A. Approximately 20 ml/dL
mean pulmonary artery pressure are respectively B. Approximately 15 ml/dL
90 and 15 mm Hg. Can you tell the ratio of C. Data inadequate
systemic and pulmonary vascular resistances?
A. 1 49. If Hb were completely absent, how much
B. 10 plasma would be required to transport oxygen
C. 6 required for basal metabolism?
D. Data inadequate A. 10 liters
B. 27 liters
41. Ventilation-perfusion ratio (whole lung) is C. 52 liters
normally: D. 84 liters
A. 1
B. 0.8 50. Which of the following shifts the
C. 0.6 oxyhemoglobin dissociation curve to the right?
D. 1.2 A. CO2
B. 2, 3-BPG
42. In the upright position, V/Q ratio is highest in C. Increase in tissue temperature
the: D. Decrease in tissue pH
A. lung apices
B. middle zone
C. lung bases
72
E.S.Prakash. MCQs in Medical Physiology, May 2013
51. Suppose you administer 100% oxygen at 4 58. The major stimulus for spontaneous
atmospheres, then how much oxygen would be respiration is:
transported in 100 ml of plasma? A. CO2
A. 0.3 ml B. O2
B. 3 ml C. lactate
C. 6 ml D. HCO3
D. 9 ml
59. Central chemoreceptors are chiefly stimulated
52. The decrease in oxygen affinity of Hb when by:
the pH of blood in tissues falls is: A. an increase in CSF [H+]
A. Bohr effect B. a fall in PaO2
B. Haldane effect C. a decrease in PaCO2
C. Hawthorne effect D. all of the above
D. Hamburger effect
60. The neurons pacing spontaneous breathing are
53. Which of the following shifts the located in:
oxyhemoglobin dissociation curve to the left? A. pneumotaxic center
A. CO B. apneustic center
B. Increase in [2,3 BPG] in RBC C. pre-Bottzinger complex
C. Increase in tissue temperature D. dorsal motor nucleus of vagus
D. Decrease in tissue pH
61. Systemic arterial chemoreceptors (commonly
54. The commonest cause of arterial hypoxemia called peripheral chemoreceptors) are mainly
is: stimulated by:
A. type II respiratory failure A. a decline in PaO2
B. lactic acidosis B. an increase in [H+] of arterial plasma
C. V/Q mismatch C. a decrease in blood flow through them
D. anemia D. an increase in PaCO2

55. PaO2 is / may be normal in hypoxia due to: 62. The plasma/CSF ratio of proteins is:
A. hypoventilation A. 3
B. diffusion limitation B. 10
C. severe circulatory shock C. 20
D. anemia D. 300
E. cyanide poisoning
63. Forced hyperventilation to exhaustion may
56. Administration of O2 rich gas mixtures result in:
improves tissue oxygenation most in: A. a decrease in serum ionized calcium
A. hypoxic hypoxia B. an increase in plasma protein level
B. anemic hypoxia C. an increase in blood pH
C. hypokinetic hypoxia
D. histotoxic hypoxia 64. Hypocapnia is usually due to:
A. hypoxemia
57. CO2 is chiefly transported in blood: B. alkalosis
A. as bicarbonate C. decreased CO2 production
B. bound to hemoglobin D. alveolar hyperventilation
C. as dissolved CO2
D. in combination with plasma proteins 65. Normally, PaCO2 chiefly depends on:
A. total lung capacity
B. PaO2
73
E.S.Prakash. MCQs in Medical Physiology, May 2013
C. alveolar ventilation B. 21%
D. rate of CO2 production C. 28%
D. 35%
66. Caution needs to be exercised in administering
O2 to patients with: 73. Regarding physiologic changes during
A. ARDS moderate isotonic exercise in a healthy young
B. metabolic acidosis adult, which of the following are true and which
C. type I respiratory failure are false?
D. type II respiratory failure 1. PaO2 declines
2. PaCO2 increases
67. Which of the following data suggests type II 3. pH of arterial blood decreases (eventually
respiratory failure? (pH, PaCO2 and plasma during intense exercise)
[HCO3] given below in that order). 4. A-V O2 difference increases
A. 7.4, 60 mm Hg, 30 mmol/L 5. SvO2 increases
B. 7.2, 80 mm Hg, 30 mmol/L 6. Plasma [K+] increases
C. 7.33, 46 mm Hg, 20 mmol/L 7. The impulse rate from carotid bodies
increases.
68. J receptors are located in: 8. During maximal exercise, O2 consumption
A. pulmonary interstitium may reach up to 4 L /min, about 16 times
B. alveoli basal oxygen consumption.
C. bronchial mucosa 9. VO2 max (maximum oxygen consumption
D. roof of the fourth ventricle during intense exercise) is increased by
training.
69. Stimulation of J receptors results in: 10. Heart rate may remain elevated for as long as
A. apnea 1 hour after severe exercise.
B. tachypnea
C. hyperpnea Are chemoreceptors stimulated in anemia or
D. apnea followed by tachypnea not?
For the sake of clarity, let us ask a question like
70. An increase in the duration of expiration this:
produced by lung inflation is due to: Chemoreceptors are not stimulated in:
A. Hering-Breuer deflation reflex A. anemia
B. pulmonary chemoreflex B. anemic hypoxia
C. Bezold-Jarisch reflex
D. Hering-Breuer inflation reflex Note the difference between anemia, which
simply means that the concentration of
71. Which of the following statements regarding hemoglobin is lower than normal, and anemic
Cheyne-Stokes respiration are correct? hypoxia - a consequence of anemia associated
A. It is characterized by periods of hyperpnea with anerobic metabolism and acidosis.
punctuated by apnea.
B. It is attributed to increased sensitivity of the The effect of acidosis is stimulation of the
respiratory center to CO2. peripheral chemoreceptors (i.e., those in the
C. It may occur if lung-to-brain circulation time carotid bodies) in anemic hypoxia. In an anemic
is prolonged. individual who is at rest, and whose arterial pH is
D. It may occur in severe congestive heart failure normal, peripheral chemoreceptors may not be as
stimulated.
72. During mouth-to-mouth resuscitation, what is
the oxygen concentration of the resuscitating gas “The additional increase in ventilation (during
mixture? exercise) produced by the acidosis is dependent
A. 16%
74
E.S.Prakash. MCQs in Medical Physiology, May 2013
upon the carotid bodies and does not occur if they 56A 57A 58A 59A 60C
are removed”. See p 682, Ch 37, WFG, 2005. 61ABC 62D 63AC 64D 65C
66D 67B 68A 69D 70D
Now take this question: Systemic arterial 71all 72A
chemoreceptors are stimulated in (check all 1 Tidal volume; 14 Diaphragm; 18 Emphysema
correct answers):
A. anemic hypoxia 73. Physiologic changes during exercise:
B. hypoxic hypoxia 1F 2F 3T 4T 5F
C. hypokinetic hypoxia 6T 7T 8T 9T 10T
D. histotoxic hypoxia
Answer Explanations:
All are correct. The question is whether peripheral 5. FRC (the volume of air remaining in the lungs
chemoreceptors are stimulated or not in various at the end of a tidal breath) cannot be determined
types of hypoxia. It is not how they are by spirometry.
stimulated. There are several factors besides a low
PaO2 that could stimulate peripheral FRC is determined by:
chemoreceptors. These are: 1. helium dilution technique
2. whole body plethysmography
A decrease in blood flow through the
chemoreceptors as occurs in circulatory shock 6. Principle: Indicator-dilution principle
(stagnant hypoxia) can stimulate them. Indicator: Helium. This is used because it has a
very low molar mass (low density), that it does
Inhibition of oxygen utilization in chemoreceptor not dissolve in blood when it is inhaled at 1
cells (by injection of cyanide) also activates atmosphere. Initially, the helium gas is present
chemoreceptors. only in the spirometer. The person breathes from
as well as into the spirometer. The circuit is
Whenever hypoxia is accompanied by acidosis closed. After a few breaths, the helium
(for example in an anemic individual doing concentration in the spirometer will reduce
exercise), acidosis activates the receptors and because it would have also got diluted in lung
increases minute ventilation. Injection of K+ has volume.
been shown to activate them; this may partly
account for exercise induced changes in Initially,
ventilation. C1: Helium concentration in spirometer = 10%
V1: Spirometer volume = 12 liters
Thus, in anemic hypoxia, arterial chemoreceptors C2: Helium concentration in spirometer (= lungs)
are not stimulated by hypoxia, but rather by the = 6.7%
ensuing acidosis. V2: Volume of distribution is now total lung
capacity + spirometer volume.
Answers to Pulmonary Physiology: Very little helium actually dissolves in blood. So
2C 3B 4BCD 5CD it can be neglected.
6D 7A 8B 9F 10T Applying law of conservation of mass,
11B 12C 13B 15B C1V1 = C2 (V1+V2)
16B 17B 19T 20B Substituting, we get,
21C 22AB 23D 24A 25BC 10 × 12 = 6.7 (12 + TLC)
26A 27A 28AB 29E 30all TLC = [10 × 12 / 6.7] – 12 = 18 – 12 = 6L
31BC 32C 33B 34B 35B VC = 4.2 L
36B 37B 38all 39A 40C So RV = 6 – 4.2 = 1.8 L = 1800 ml.
41B 42A 43D 44F 45F
46B 47C 48C 49D 50all 7. FRC = 2300 ml
51D 52A 53A 54C 55DE
75
E.S.Prakash. MCQs in Medical Physiology, May 2013
Because alveolar oxygen is diluted with CO2, the Forced expiratory flow rate (FEF 25-75) is an
fraction of oxygen in alveolar gas is only about integrated calculation of the average flow rate
16%. during exhalation of the interquartile range (25th -
Thus, the amount of oxygen present in lungs at the 75th percent) of vital capacity.
end of a tidal expiration = (16 / 100) × 2300 = 370
ml 31. Option B – upper lung zones are less
compliant because of their higher volume at
10. Intrathoracic blood volume increases by about baseline. Option D - Positive intrapleural
400 ml in the supine position compared to pressures during forced expiration tend to cause
standing; the increase in pulmonary blood volume compression of small bronchi (dynamic airway
is greater in patients with left ventricular systolic compression).
failure and consequently they may be breathless
when lying supine. 32. What is a shunt; what is dead space?
Shunt: Perfusion of unventilated alveoli results in
16. Lung compliance is the change in lung shunting of deoxygenated blood across the lungs
volume for a given change in pressure. to the heart, i.e. a right-to-left shunt.
Compliance = ∆V / ∆P Dead space: Ventilation of unperfused alveoli is
The normal compliance of human lungs and chest effectively an extension of anatomic dead space
wall is about 0.2 L/cm H2O. Compliance is since it cannot contribute to gas exchange.
reduced in restrictive lung disease.
35-37. At rest, a healthy adult weighing 70 kg
22. VIP is released by nonadrenergic uses about 250 ml of oxygen per minute and 200
noncholinergic neurons. VIP relaxes bronchial ml of CO2 is produced.
smooth muscle. VIP deficiency has been
implicated in asthma. Respiratory exchange ratio (RER) is the ratio of
the volume of CO2 to O2 exchanged across the
Leukotrienes are potent bronchoconstrictors. lungs per minute. At rest, it is normally 200 / 250
Zileuton, a lipoxygenase inhibitor inhibits the = 0.8
synthesis of leukotrienes. It is used in the
management of asthma. Zafirlukast is a Respiratory exchange ratio reflects the average
leukotriene receptor blocker. respiratory quotient (RQ) when gas exchange
across lungs is normal.
25. Inhaled nitric oxide has been shown to
produce bronchodilation as well as a decrease in RQ is different for different energy substrates.
pulmonary artery pressure. Substance P, platelet
activating factor (PAF) and leukotrienes are Carbohydrate 1
bronchoconstrictors. Protein 0.8
Fat 0.7
28. PEFR is reduced when vital capacity is Average RQ 0.8
reduced, and so is reduced in both obstructive and
restrictive lung disease. It is used therefore to The average RQ depends upon the metabolic
monitor airways resistance only in individuals state.
who are known to have asthma.
38. The alveolar gas equation:
29. Maximum mid-expiratory flow rate Alveolar oxygen tension PAO2
(MMEFR) is the peak flow rate during = [(PB-PH20) × FiO2] – [(PAC02)/R]
expiration. It may not be the same as PEFR.
MMEFR is effort independent. It reflects the PB is barometric pressure (760 mm Hg at mean
resistance of small airways affected in asthma. sea level);
PH20 is the pressure of water vapor at body
temperature (it is 47 mm Hg);
76
E.S.Prakash. MCQs in Medical Physiology, May 2013
FiO2 is the percentage of oxygen in inspired air; (P is mean arterial pressure and R is vascular
and PACO2 is the CO2 tension in alveolar gas resistance)
(normally about 40 mm Hg). It is equal to PaCO2 Ps / Pp = Rs/Rp (s and p refer to systemic and
because CO2 is highly soluble and readily pulmonary vascular beds respectively)
equilibrates across the alveolocapillary Ps / Pp = 90 / 15 = 6
membrane. So you can substitute PaCO2 for Ratio of systemic and pulmonary vascular
PACO2. resistance = 6
Thus, the pulmonary circulation is a low
R is respiratory exchange ratio (as defined above, resistance, low pressure system.
see answer explanation for Questions 35-37); it is
assumed to be 0.8 when the alveolar gas equation At rest, systolic pulmonary artery pressure is
is used for solving common clinical problems. about 25 mm Hg, diastolic pulmonary artery
pressure averages 10 mm Hg, mean pulmonary
Substituting, we get, artery pressure is about 15 mm Hg, and
PAO2 = [(760 – 47) × 0.21] – [40/0.8] pulmonary capillary pressure averages about 10
= 150 – 50 = 100 mm Hg mm Hg.

39. When alveolar ventilation is held constant, the 41-43. Ventilation–perfusion ratio is the ratio of
amount of CO2 removed is limited by pulmonary alveolar ventilation to pulmonary blood flow.
blood flow. However, CO2 accumulation is For example, at rest, alveolar ventilation = 12 ×
invariably due to a decrease in alveolar (500 – 150) = 4.2 L/min;
ventilation. Assuming VT = 500 ml, VD = 150 ml;
At rest, pulmonary blood flow = 5 L/min;
When alveolar oxygen tension is constant, Thus, whole lung V/Q ratio = 0.84;
normally (i.e., at rest), oxygen exchange is limited In the lung apices, V/Q approaches infinity;
by perfusion because enough time is available for In the lung bases, V/Q is lower than 1.
equilibration across the alveolocapillary
membrane. However, during exercise, when blood It must however be noted that alveolar ventilation
flow rates are higher, the time available for is much higher at the lung bases than the apices
equilibration is lesser and oxygen exchange is because the lower lung zones are less distended at
increasingly limited by diffusion (diffusion- the start of inspiration and therefore more
limited). Thus, impairment in diffusion capacity compliant. As a result of regional differences in
of the lungs for oxygen causes problems intrapleural pressure, upper lung zones are already
especially during exercise. in a more expanded position at the start of
inspiration.
Testing diffusion capacity of the lungs:
The “single breath technique”: The subject Lung zones:
inhales a mixture containing 0.01% CO PA = Alveolar pressure;
We know that diffusion rate J = (PA–Pc) × DA/x Pa = Pressure at the arterial end of the pulmonary
In a nonsmoker, Pc CO = 0 mm Hg and PA is a capillary;
constant; thus the diffusion rate depends upon Pv = Pulmonary venous pressure;
DA/x Pi = Pulmonary interstitial pressure.
DLCO = 25 ml/min/mm Hg
DLO2 = 25 ml/min/mm Hg Zone Definition
Since CO exchange is diffusion limited, it is used Zone 1 PA > Pa > Pv
Zone 2 Pa > PA > Pv
for assessing diffusion capacity. Zone 3 Pa > Pv > PA
Zone 4 Pa > Pi > PA
40. Since pulmonary and systemic circulations are
in series, the flows are identical Zone 1: above the heart; arterial pressure is lower
Cardiac output = P / R; and may be lower than alveolar pressure if the
77
E.S.Prakash. MCQs in Medical Physiology, May 2013
alveoli are well expanded. Flow may be minimal. H + HCO3 = CO2 (acid);
V/Q approaches infinity. Hb is the major buffer of CO2

Zone 2: pulmonary arterial pressure is greater 48. Factors that affect oxygen delivery to the
than alveolar pressure and flow is determined by tissues include the following (the oxygen
arterial – alveolar pressure difference; however, cascade):
during inspiration when alveolar pressure • Alveolar ventilation
becomes more negative, blood flows from the • Pulmonary blood flow
arteries into the pulmonary veins. This is called • Ventilation / perfusion balance
the ‘waterfall effect’. • Diffusion capacity of lungs for oxygen
• Cardiac output
Zone 3: flow is continuous and is driven by • Hb concentration of arterial blood
pulmonary arterial – venous pressure gradient.
• Affinity of Hb for oxygen
• Blood flow to each tissue
Zone 4: it occurs in pulmonary edema (always
abnormal). When pulmonary interstitial pressure
49. Basal oxygen requirement at rest = 250
is > than alveolar pressure, alveolar collapse
ml/min
(atelectasis) would result.
O2 carrying capacity of 1L of plasma = 3 ml
Amount of plasma that would be required to
When a Swan Ganz catheter is wedged in the
transport 250 ml of oxygen = 83 liters!
pulmonary capillary to estimate left ventricular
This problem illustrates the importance of Hb.
end diastolic pressure (LVEDP) in a patient who
is on a ventilator with positive end expiratory
51. O2 solubility in plasma is limited, hence Hb
pressure (PEEP) added, the measured pressure
required for oxygen transport.
would reflect alveolar pressure rather than
O2 solubility in plasma = 0.003 ml O2/100 ml
LVEDP; thus, the catheter must be placed in Zone
plasma/ mm Hg PO2.
3 conditions (perhaps by momentarily
Greater the PaO2, greater the dissolved oxygen
discontinuing PEEP) to reliably estimate LVEDP
(Henry’s Law)
Calculate the expected PaO2 using the alveolar
Regulation of pulmonary vascular resistance:
gas equation:
PAO2 = [(4 × 760) – 47] × 1 – (40/0.8)
Block ventilation to a lung zone
Expected PaO2 is approximately 3000 mm Hg.
Thus, about 9 ml of oxygen will be transported in
Perfusion of that lung zone decreases
100 ml of plasma.
That is, in 4 liters of plasma 360 ml of oxygen can
Hypoxic pulmonary vasoconstriction serves to
be transported. Let us say the requirement is 250
shunt blood to well ventilated lung zones and
ml O2/min. This meets the demand; this is the
optimize oxygen uptake in the lungs. However,
rationale for use of hyperbaric oxygenation.
chronic alveolar hypoxia leads to a chronic
However, it is resorted to only when the oxygen
increase in pulmonary vascular resistance and
transport capacity of blood cannot be raised
pulmonary hypertension.
otherwise because oxygen toxicity is also a
significant concern.
44. It contains more CO2 since CO2 is much more
soluble than O2 in blood.
52. The presence of higher levels of CO2 and H+
in the capillaries of metabolically active tissue
45. It is 75% saturated with oxygen
promotes the release of O2 from hemoglobin. This
is the Bohr effect.
47. HCO3 cannot buffer protons formed from
CO2;
53. CO is a competitive inhibitor of oxygen
CO2 + H2O = H+ + HCO3
binding of hemoglobin. Exposure for 1 hour to a
78
E.S.Prakash. MCQs in Medical Physiology, May 2013
CO concentration of 0.1% in inspired air leads to hypocapnia (PaCO2 < 35 mm Hg) is almost
the occupancy by CO of about 50% of heme sites always due to hyperventilation, i.e. minute
in Hb, a proportion that is frequently fatal. Apart ventilation in excess of that required to maintain
from being a competitive inhibitor of oxygen arterial plasma pH at 7.4
binding to Hb, CO also inhibits the dissociation of
oxyhemoglobin (prevents unloading of oxygen at 66. In chronically hypercapnic patients, CO2
the tissues). That is, it shifts the ODC to the left. depresses rather than stimulates respiration and
respiration is driven by hypoxia. Administration
54. Ventilation – perfusion imbalance is the of 100% oxygen may remove the hypoxic drive
commonest cause of hypoxemia. Of course, the and breathing may stop. Such a patient will
hypoxemia in ARDS is also due to ventilation- require mechanical ventilation to bring down CO2
perfusion match; V/Q mismatch is an umbrella a level which would stimulate respiration.
term that illuminates pathophysiology and is the
single best answer for this question. 70. The Hering Breuer inflation reflex is an
increase in the duration of duration of expiration
55. Note the difference between the terms produced by steady lung inflation and the Hering
hypoxemia (decreased arterial PO2) and hypoxia Breuer deflation reflex is a decrease in the
(inadequate tissue oxygenation). While the PaO2 duration of expiration caused by marked deflation
may be normal in carbon monoxide poisoning, of the lung.
circulatory shock, cyanide poisoning and anemia,
the tissues are yet hypoxic. Carbon monoxide Nonchemical influences on respiration
inhibits the dissociation of O2 from HbO2. Reflex Stimulus Response
Hering Lung Increase in
58. CO2 in plasma quickly equilibrates across the Breuer inflation the duration
blood brain barrier. There it is quickly hydrated to inflation of
form H2CO3. Carbonic acid is unstable and reflex* expiration
dissociates at physiologic pH to form H and
Hering Lung Increase in
HCO3. The protein content of CSF is extremely
Breuer deflation the duration
low that it cannot buffer H ions formed from deflation of
carbon dioxide. Thus H ions in CSF stimulate reflex* inspiration
central chemoreceptor neurons which in turn drive
the respiratory neurons. Central chemoreceptors Head’s Lung Further
are chiefly stimulated by an increase in PaCO2 i.e. paradoxical inflation inflation
an increase in [H+] of CSF. reflex€
*Hering Breuer reflexes are probably initiated by
61. In arterial plasma, CO2 gets buffered by Hb considerable changes in lung volume (deep
and plasma proteins. Therefore, it is less effective inspiration or deep expiration).

stimulating receptors in carotid and aortic bodies. It may play a role in lung expansion in
This would not happen in the CSF because there newborns.
is very little protein there. Note that the ratio of
the concentration of protein in plasma and CSF is 71. Periodic breathing may occur in:
normally about 300. • Severe congestive heart failure
• Uremia
62. Plasma protein concentration = 6000 mg/dL • Sleep
CSF protein concentration = 20 mg/dL • Brain stem disease
• A normal individual following a bout of
64. If minute ventilation reduces, then PaCO2 voluntary hyperventilation.
increases (hypercapnia). Hypercapnia is a PaCO2
> 45 mm Hg. Hypercapnia is almost always due 73. PaCO2 changes little because alveolar
to inadequate alveolar ventilation. Conversely, ventilation also increases.
79
E.S.Prakash. MCQs in Medical Physiology, May 2013
The solubility of oxygen in plasma at body
The impulse rate from the carotid bodies increases temperature is 0.003 ml / mm Hg PO2 / 100 ml of
possibly because of an increase in body blood;
temperature, plasma [K] and [H] formed from Assuming PO2 is 100 mm Hg, the amount of
lactic acid. oxygen dissolved in blood / plasma ≈ 0.3 ml / dL
However, CO2 is about 20 times more soluble in
Plasma K increases because K is released from blood compared to O2 and there is much more
actively metabolizing skeletal muscle. CO2 in arterial blood compared to oxygen (see the
tables below).
pH of arterial blood is initially maintained but
eventually decreases and that is when one Normal PO2 and PCO2 levels in blood:
fatigues. A substantial increase in hydrogen ion PO2 Arterial 100 mm Hg
concentration of arterial blood will accelerate the PO2 Venous 40 mm Hg
onset of fatigue. PCO2 Arterial 40 mm Hg
PCO2 Venous 46 mm Hg
Arteriovenous O2 difference increases because Gas content of blood (in ml).
oxygen extraction by tissues increases. Arterial Venous
Dissolved Combined Dissolved Combined
Saturation of mixed venous blood with oxygen O2 0.3 19.5 0.12 15
decreases because oxygen extraction by CO2 2.5 45 3 50
metabolically active tissue increases.
Oxygen transport capacity of blood:
During maximal exercise, O2 consumption may Assuming all Hb is Hb A, PaO2 = 100 mm Hg,
increase to as high as 3-4 liters/min, i.e. 12-16 and that Hb is fully saturated with O2, then:
times the oxygen consumption at rest. Content of oxygen in arterial blood (CaO2) = 19.8
ml/dL of blood; of this, about 0.3 ml is dissolved
Heart rate may remain elevated for as long as 1 in plasma and the rest is in combination with Hb
hour after severe exercise because the increase in A.
sympathetic nerve activity also persists. This is
due to the oxygen debt incurred during exercise. Hypoxia: The term is often used to denote
Until the debt is cleared, minute ventilation and inadequate oxygenation of tissues (tissue
cardiac output will be higher compared to hypoxia).
preexercise levels.
Hypoxemia: a reduction in oxygen tension of
*************************************** arterial blood below 80 mm Hg
Supplement: Gas Transport
Classification of hypoxemia (based on
Oxygen loading in the lungs: severity):
the reaction of Hb with oxygen Grade PaO2
The T (tense) configuration reduces the affinity of Mild 61–80 mm Hg
Hb for O2; the R (relaxed) configuration increases Moderate 41–60 mm Hg
the affinity for O2 Severe (fulminant) < 40 mm Hg
Hb4 + O2 = Hb4O2
Hb4O2 + O2 = Hb4O4 Classification of causes of tissue hypoxia:
Hb4O4 + O2 = Hb4O6 Type PaO2 Cause Treatment
Hb4O6 + O2 = Hb4O8 Hypoxic ↓ High Administer
hypoxia altitude; oxygen;
These are some facts that you must remember: defects in address cause
gas
exchange
Anemic Normal Anemia; Raise oxygen
80
E.S.Prakash. MCQs in Medical Physiology, May 2013
hypoxia CO carrying Transport of CO2 in 100 ml arterial blood:
poisoning capacity of Arterial Venous
plasma / Dissolved 2.5 ml 3 ml
blood as Bicarbonate 45 ml 47 ml
appropriate;
address cause
Carbamino 2.5 ml 3.5 ml
Hypokin Normal Myocardia Address compounds
etic or ↓ l hypoxia cause;
hypoxia, due to How much acid do the lungs excrete per day?
alt. coronary 200 ml of CO2 per minute amounts to 288 liters of
stagnant ischemia; CO2 per day, which is equivalent to 12500 meq of
hypoxia, circulatory H+ per day. The kidneys normally excrete about
ischemic shock 50 mEq of H ions per day.
hypoxia
Functions of hemoglobin: Apart from
Histotox Normal Cyanide Address transporting oxygen, CO2 and buffering protons,
ic poisoning cause;
Hb also binds nitric oxide (a vasodilator); this
hypoxia hyperbaric
oxygenation may help match oxygen delivery with increases in
tissue blood flow.
Myoglobin (Mb): an oxygen storage protein; also ***************************************
a heme protein; single subunit; so there is no
quaternary structure; so it does not bind 2,3 BPG;
it is abundant in red muscle; 1 mol of Mb binds
one mol of O2; its P50 is about 1 mm Hg (much
lesser compared to HbA). Neuroglobin may have
a similar function in brain.

Transport of CO2
1. CO2 from tissues diffuses into RBC
2. It is hydrated by the enzyme carbonic
anhydrase (CA) present in RBC:
CO2 + H2O = H2CO3
The hydration of CO2 in plasma is much
slower because of the absence of CA in
plasma.
3. H2CO3 dissociates into H+ and HCO3
4. H+ is buffered by Hb; HCO3 enters plasma in
exchange for chloride which enters RBC; this
is called chloride shift or Hamburger
phenomenon (it is mediated by the Cl-HCO3
exchanger)

What happens in the lungs?


The high PO2 in alveolar gas causes Hb to give up
CO2 and take up oxygen.

Haldane effect: The high concentration of O2 in


the alveolar capillaries unloads H+ and CO2 from
hemoglobin; i.e., oxygen dissociation curve of Hb
shifts to the left.

81
E.S.Prakash. MCQs in Medical Physiology, May 2013
10. Renal and Acid-Base Physiology B. GFR / RPF
C. RPF × GFR
Abbreviations used here:
ADH – antidiuretic hormone 5. What fraction of renal plasma flow is normally
ANG II – angiotensin II filtered in the glomerular capillaries?
CCD – cortical collecting duct A. 0.1
MCD – medullary collecting duct B. 0.2
MD – macula densa C. 0.3
DCT – distal convoluted tubule D. 0.4
GFR – glomerular filtration rate
MI – medullary interstitium 6. When mean arterial pressure is held constant,
PT – proximal tubule selective constriction of glomerular efferent
PCT – proximal convoluted tubule arterioles:
PST – proximal straight tubule A. increases GFR
RBF – renal blood flow B. raises filtration fraction
RPF – renal plasma flow C. decreases renal plasma flow
TAL – thick ascending limb
TDL – thin descending limb 7. In humans, what percentage of nephrons has
OMCD – outer medullary collecting duct long loops of Henle?
IMCD – inner medullary collecting duct A. 5
FENa – fractional excretion of sodium B. 15
C. 25
Instructions: In each of the following questions, D. 85
check all correct answers.
8. Juxtamedullary nephrons constitute about
1. The ‘gold standard’ for estimation of ______ % of the total nephrons.
glomerular filtration rate is the clearance of A. 10
A. inulin B. 15
B. creatinine C. 25
C. urea D. 40
D. mannitol
E. glucose 9. The macula densa is located in the:
F. para-amino hippuric acid A. juxtaglomerular cells
B. extraglomerular mesangium
2. In clinical practice, the urinary clearance of C. beginning of early distal tubule
which substance is most frequently used to D. peritubular capillaries
estimate GFR?
A. Inulin 10. The ‘brush border’ is most extensive in cells
B. Creatinine of the:
C. Urea A. PCT
D. Mannitol B. loop of Henle
C. DCT
3. Which of the following substances should not D. collecting duct
be used to measure GFR?
A. Inulin 11. Which of the following increases renal
B. Creatinine cortical blood flow?
C. Phenol red A. Prostaglandin E2
B. Norepinephrine
4. The term filtration fraction is used to refer to: C. Angiotensin II
A. RPF / GFR D. Vasopressin
82
E.S.Prakash. MCQs in Medical Physiology, May 2013
9. It causes mesangial cells to contract;
12. The PAH clearance of a 56-year-old male with 10. Normally, it serves to maintain GFR;
sustained elevations of blood urea nitrogen, 11. It increases Na and H2O reabsorption by a
creatinine and potassium was estimated to be 100 direct action on PT
ml/min. His PCV is 20. His renal blood flow is 12. It plays an important role in cardiac
approximately: remodeling after a myocardial infarction;
A. 25 ml/min 13. Angiotensin III has 100% aldosterone-
B. 125 ml/min stimulating activity;
C. 250 ml/min 14. Angiotensin III is a more potent pressor than
D. 600 ml/min angiotensin II;
15. Angiotensin converting enzyme is a dipeptidyl
13. Autoregulation of GFR is achieved through carboxypeptidase.
changes in: 16. ACE is the only enzyme responsible for
A. renal perfusion pressure angiotensin II synthesis.
B. renal blood flow
C. glomerular afferent arteriolar resistance 16. Which of the following stimulates the release
D. glomerular efferent arteriolar resistance of aldosterone from the adrenal cortex?
A. Renin
14. In a healthy adult human with a resting BP B. Angiotensin III
averaging about 110/70 mm Hg who is not on any C. ↓ plasma [K+]
medication, the earliest MAP at which renal D. ANP
autoregulation of GFR is overwhelmed is when
mean arterial pressure falls below: 17. In the nephron, glucose reabsorption occurs
A. 30 mm Hg mainly in the:
B. 50 mm Hg A. proximal tubule
C. 70 mm Hg B. loop of Henle
D. 90 mm Hg C. distal convoluted tubule
D. collecting duct
15. The rate-limiting step in the biosynthesis of
angiotensin II is the synthesis of: 18. The following data were obtained on a patient:
A. renin Creatinine clearance = 125 ml/min; plasma
B. angiotensinogen [glucose] = 400 mg/dL. If the tubular maximum
C. angiotensin I for glucose were 350 mg/min, what would the
D. angiotensin II glucose excretion rate be?
A. 50 mg/min
All about angiotensin II: B. 100 mg/min
Which of the following statements regarding C. 150 mg/min
angiotensin II are true and which are false? D. 200 mg/min
1. It is a much more potent vasopressor
compared to norepinephrine; 19. Reabsorption of water in the PT:
2. It stimulates aldosterone release from adrenal A. is chiefly driven by osmotic gradients
cortex; B. occurs through water channels in apical
3. It inhibits ADH release; membranes of tubular epithelial cells
4. It has a dipsogenic effect; C. is facilitated by vasopressin
5. It reduces Na reabsorption in PT; D. all of the above statements are correct
6. It facilitates norepinephrine release from
postganglionic sympathetic neurons; 20. Principal cells (P cells) in the cortical
7. It has a greater constrictor effect upon collecting duct of the nephron are concerned with:
glomerular afferent arterioles; A. secretion of protons
8. It reduces renal plasma flow; B. potassium reabsorption
83
E.S.Prakash. MCQs in Medical Physiology, May 2013
C. ADH-stimulated water reabsorption D. collecting ducts
D. reabsorption of sodium and chloride E. loop of Henle

21. Which ion is both secreted and absorbed by 28. In a euvolemic normotensive healthy adult
the nephron? with, ADH secretion is most likely to be
A. Na stimulated by:
B. K A. intake of 500 ml water
C. Cl B. intake of 30 ml of 10% ethanol
D. Ca C. 2.5 mg of enalapril
D. ingestion of a 400 meal consisting exclusively
22. The retention of ‘free water’ by ADH in the of simple sugars
collecting ducts is mediated mainly by which
aquaporin in the luminal membrane? 29. Which one of the following inhibits ADH
A. AQP-1 secretion?
B. AQP-2 A. Hyperosmolality
C. AQP-3 B. Exercise
D. AQP-4 C. Angiotensin II
D. Ethanol
23. The major humoral regulator of the
concentration of sodium in plasma is: 30. Water deprivation fails to produce an increase
A. aldosterone in urine osmolality in:
B. vasopressin (ADH) A. neurogenic diabetes insipidus
C. renin B. nephrogenic diabetes insipidus
D. angiotensin II
31. The tubular fluid at the end of the PT is ____
24. Plasma potassium concentration is chiefly with respect to plasma
regulated by: A. isotonic
A. aldosterone B. hypotonic
B. vasopressin (ADH) C. hypertonic
C. renin
D. ANP 32. Which is the ‘concentrating segment’ of the
nephron?
25. In the presence of vasopressin, most of the A. Descending limb of loop of Henle
water filtered by the kidneys is reabsorbed in the: B. Thin ascending limb of loop of Henle
A. proximal tubule C. Thick ascending limb of loop of Henle
B. thin descending limb D. Collecting ducts
C. thick ascending limb
D. collecting ducts 33. Which portion of the tubule is least
permeable to water?
26. In the absence of vasopressin, most of the A. Proximal tubule
filtered water is reabsorbed in the: B. Thin descending limb of LOH
A. PT C. Thick ascending limb of LOH
B. loop of Henle D. Cortical collecting duct
C. DT
D. collecting ducts 34. The ‘diluting segment’ of the nephron is:
A. proximal tubule
27. Potassium secretion occurs mainly in: B. descending limb of loop of Henle
A. PT C. ascending limb of loop of Henle
B. early distal tubule D. cortical collecting duct
C. late distal tubule
84
E.S.Prakash. MCQs in Medical Physiology, May 2013
35. ‘Free water’ is formed in the: C. aquaporins in PT
A. descending limb of loop of Henle D. carbonic anhydrase in PT
B. thin ascending limb of loop of Henle
C. thick ascending limb of loop of Henle 42. Vasa recta receive blood from the:
D. collecting duct A. afferent arteriole of cortical nephron
B. afferent arteriole of juxtamedullary nephron
36. Na-K-2Cl transporter in the apical membrane C. efferent arteriole of cortical nephron
of the thick ascending limb of the loop of Henle is D. efferent arteriole of juxtamedullary nephron
an example of:
A. primary active transport 43. Hypertonicity in the inner medulla is
B. secondary active transport maintained because:
C. passive transport A. vasa recta receive 50% of renal blood flow
D. countertransport B. of the low blood flow rate through vasa recta
C. NaCl passively moves into the medullary
37. What fraction of filtered water is reabsorbed interstitium from the thin ascending limb
in the loop of Henle?
A. 15% 44. The system for the formation of concentrated
B. 25% or dilute urine does not include the:
C. 35% A. proximal convoluted tubule
D. 5% B. loop of Henle
C. collecting ducts
38. What fraction of filtered Na is reabsorbed in D. vasa recta
the loop of Henle? E. medullary interstitium
A. 15%
B. 25% 45. ‘Free water’ formation in the nephron is
C. 35% essential for production of:
D. 5% A. dilute urine
B. concentrated urine
39. The fluid entering the early distal tubule is
(choose the best answer): 46. What contributes most to the osmolality of the
A. always isotonic medullary interstitium? Urea / NaCl?
B. always hypotonic
C. isotonic / hypotonic but not hypertonic 47. Which segment of the nephron is permeable to
D. isotonic / hypertonic but not hypotonic urea?
E. hypertonic when urine is concentrated A. Descending limb of loop of Henle
F. hypotonic when urine is dilute B. Thin ascending limb of loop of Henle
C. Thick ascending limb of loop of Henle
40. Which of the following transport mechanisms
in the macula densa allows it to sense Na and Cl 48. The principal regulator of plasma osmolality
concentrations in the tubular fluid that flows past is:
this segment? A. plasma [Na]
A. Apical chloride channels B. antidiuretic hormone
B. 1Na-1K-2Cl transporter (NKCC2) C. aldosterone
C. Na-Cl cotransporter D. angiotensin II
D. Na-H exchanger
E. ROMK exchanger 49. Which is the fundamental mechanism
generating hypertonicity in the medullary
41. Loop diuretics (example, furosemide) inhibit: interstitium?
A. 1Na-1K-2Cl transport in the TAL A. Active transport of NaCl in TAL
B. Na reabsorption in the collecting duct B. ADH action in the collecting ducts
85
E.S.Prakash. MCQs in Medical Physiology, May 2013
C. Passive recirculation of NaCl in MI B. loop of Henle
D. Urea permeability of collecting ducts C. the aldosterone sensitive distal nephron
D. urinary bladder
Acid-Base Physiology
57. Normally, in urine, most of the H+ is tied up
50. In the nephron, the highest amount of H ions with:
is secreted into tubular lumen in the: A. bicarbonate
A. PCT B. phosphate
B. DCT C. ammonia
C. collecting ducts
D. loop of Henle 58. Normally, most of the titratable acidity of
urine is attributable to acid buffered by:
51. The principal site of ammoniagenesis in the A. bicarbonate
nephron is: B. phosphate
A. PT C. ammonia
B. Loop of Henle D. uric acid
C. Early distal tubule E. creatinine
D. Late distal tubule
E. Outer medullary collecting duct 59. Which of the following statements is / are
correct regarding the role of kidney in acid-base
52. The principal mechanism of H+ secretion in balance?
the proximal convoluted tubule is: A. The kidney can eliminate CO2 as free H
A. Na–H exchange B. Acid secretion is a passive process
B. H–ATPase C. Most acid secreted in the PCT acidifies urine
C. H–K ATPase D. Acid secretion in collecting ducts is facilitated
D. H–Ca exchange by aldosterone

53. The principal mechanism for acid secretion by 60. Type B intercalated cells in the collecting duct
type A intercalated cells in the nephron is: are concerned with:
A. H–ATPase A. secretion of protons
B. Na–H exchanger B. controlled bicarbonate secretion
C. H–Ca exchanger C. ADH-stimulated water reabsorption
D. H–K ATPase D. Na reabsorption

54. Normally, most of the filtered HCO3 is 61. The [H+] of arterial plasma is normally about:
reabsorbed from: A. 20 nmol/L
A. PT B. 30 nmol/L
B. distal tubule C. 40 nmol/L
C. cortical collecting duct D. 50 nmol/L
D. loop of Henle
62. Urinary ratio of NH4+ to titratable acid ranges
55. The most abundant buffer in proximal tubular from:
fluid is: A. 1–2.5
A. bicarbonate B. 2–5
B. phosphate C. 0.1–0.5
C. ammonia D. 5–7

56. The principle site of acidification of tubular 63. The Henderson-Hasselbalch equation states
fluid is: that when a buffer acid is half dissociated the pH
A. proximal convoluted tubule of the solution is equal to:
86
E.S.Prakash. MCQs in Medical Physiology, May 2013
A. pKa
B. 7.0 All about ANG II:
C. 2 pKa 1T 2T 3F 4T 5F
D. 0.5 pKa 6T 7F 8T 9T 10T
11T 12T 13T 14F 15T
64. A buffer is most effective when the pH of the 16F
solution is close to:
a. pKa Answer Explanations:
b. 2 pKa 1-3. Clearance is the volume of plasma that is
c. 0.5 pKa cleared of a substance per minute
Clearance of a substance X = UXV/PX
65. Quantitatively, the most important buffer of UX: concentration of substance in urine
carbonic acid in blood is: PX: concentration of substance in plasma
A. bicarbonate V: urine flow rate in ml/min
B. plasma protein
C. phosphate Inulin clearance gives the best estimate of GFR
D. hemoglobin since it is neither reabsorbed nor secreted by the
nephron; it is nontoxic, freely filtered and does
66. HCO3 cannot buffer protons formed from: not affect hemodynamics; however, estimation of
A. lactic acid inulin clearance is impractical. Endogenous
B. sulfuric acid creatinine clearance is slightly higher than inulin
C. carbonic acid clearance since some creatinine is also secreted by
the tubules. Despite this shortcoming, creatinine
67. In a healthy human with an arterial plasma pH clearance remains the most common clinical
of 7.38, and normal levels of electrolytes, the method to estimate GFR. These days, we calculate
anion gap reflects the plasma concentration of: creatinine clearance inputting age, sex, ethnicity,
A. HCO3 and serum creatinine, deploying regression
B. chloride equations such as the MDRD equation or the
C. protein anions Cockcroft Gault equation. This calculator is
D. phosphates available online at www.nephron.com

68. The anion gap may be elevated in: Phenol red has been used to estimate renal plasma
A. metabolic acidosis flow. Tubular cells secrete it.
B. metabolic alkalosis
12. Given PAH clearance = 100 ml/min
Answers: Renal Physiology Effective renal plasma flow (ERPF) = 100
1A 2B 3C 4B 5B ml/min.
6ABC 7B 8B 9C 10A Renal blood flow (RBF) = RPF × (100 / 100 –
11A 12B 13CD 14C 15D Hct). In this case PCV = 20; thus RBF = 125
16B 17A 18C 19AB 20CD ml/min.
21B 22B 23B 24A 25A True renal plasma flow actually equals
26A 27D 28D 29D 30AB ERPF/PAH extraction ratio. If this problem noted
31A 32A 33C 34C 35C that the PAH extraction ratio was 0.9, then RBF
36B 37A 38B 39B 40B would have been 138 ml/min.
41A 42D 43B 44A 45AB
46 47B 48B 49A 50A 13-14. Renal autoregulation defends changes in
51A 52A 53A 54A 55A GFR and renal blood flow in the face of changes
56C 57C 58B 59AD 60B in renal perfusion pressure. As the name suggests,
61 62 63 64 65 it is essentially independent of extrinsic
66 67 68 69 70 innervation, although it is influenced by renal
87
E.S.Prakash. MCQs in Medical Physiology, May 2013
sympathetic activity. The effect of intense Vasopressin mediates its antidiuretic effect by
stimulation of the renal sympathetic nerves is to increasing the number of AQP-2 channels in the
reduce GFR. luminal membrane of collecting duct epithelial
cells.
Renal autoregulation of GFR may be
overwhelmed when mean arterial pressure falls 23. Changes in water excretion are a commoner
below 70 mm Hg. The lower limit of this cause of changes in plasma Na concentration
autoregulatory change depends on an individual’s rather than changes in Na excretion. ADH, being
typical resting mean arterial pressure. the hormone that regulates water excretion, is an
important determinant of plasma Na concentration
When MAP falls rapidly, the reflex increase in and osmolality. For example, diabetes insipidus,
sympathetic outflow to the kidneys causes afferent which is caused by ADH deficiency, is
arteriolar constriction, decreases GFR and RPF characterized by excretion of large volumes of
and acute renal failure may ensue. dilute urine, and plasma [Na] increases.

15. Synthesis and secretion of renin by 24. Aldosterone is the hormone regulating
juxtaglomerular cells is regulated by renal blood secretion of potassium in the collecting ducts, and
flow as well as by sympathetic innervation of JG hyperaldosteronism is typically associated with
cells. hypokalemia. However, some patients with
primary hyperaldosteronism are normokalemic.
16. The renin-angiotensin-aldosterone axis is a
salt and water conserving and blood pressure 25. 60-70% of the filtered water is reabsorbed in
raising system that is set in motion by stimuli such the proximal tubule. This does not depend upon
as hypovolemia, hypotension, a decrease in renal vasopressin. Water reabsorption in the collecting
blood flow, and a decrease in GFR. ducts requires vasopressin facilitated aquaporin 2
molecules on the luminal membrane of collecting
Ang II stimulates the release of ADH by an action duct epithelial cells. When vasopressin is absent,
on the subfornical organ (SFO) and the organum 24-hr urine volume may increase to as high as
vasculosum of lamina terminalis (OVLT). It 12% of filtered water load, about 22 liters for a
stimulates Na reabsorption, and proton secretion GFR of 180 liters per day.
in the PCT. The glomerular efferent arterioles are
more susceptible to the constrictor effects of 30. Normally, water deprivation (for say 7 hr)
angiotensin II compared to afferent arterioles. produces a significant increase in ADH secretion
Angiotensin III has only 40% of the pressor Consequently, urine becomes more concentrated
activity of angiotensin II but it has 100% and its osmolality exceeds 700 mOsm/kg H2O. In
aldosterone stimulating activity. There is evidence neurogenic diabetes insipidus, urine does not
that there are enzymes other than ACE that become concentrated because of a deficiency of
convert angiotensin I to angiotensin II although ADH. In nephrogenic diabetes insipidus, urine
their quantitative importance is not established. does not become concentrated because the
kidneys do not respond to ADH. This is usually
18. Filtered load = plasma [glucose] × GFR due to a defect in V2 receptors or ADH
Amount of glucose filtered per minute = 500 mg responsive aquaporins (AQP-2) in the collecting
Tmax for glucose is given to be 350 mg/min ducts. After administration of a vasopressin
Thus, glucose excretion rate analog, the osmolality of urine increases
= filtered load – Tmax for glucose significantly in neurogenic diabetes insipidus
= 500 – 350 = 150 mg/min. whereas urine remains dilute in nephrogenic
diabetes insipidus.
22. Rapid diffusion of water across cells depends
upon the presence of water channels called 32. Luminal fluid becomes more and more
aquaporins. There are several types of aquaporins. concentrated in the thin descending limb because
88
E.S.Prakash. MCQs in Medical Physiology, May 2013
this portion of the nephron is relatively 58. The amount of alkali required to titrate acidic
impermeable to solute and freely permeable to urine to the pH of arterial plasma is called
water. Water exits the descending limb into the titratable acidity; normally, most of this is due to
hypertonic medullary interstitium. Thus protons buffered by phosphate. However, in
osmolality of tubular fluid is greatest at the tip of patients with chronic renal failure, protons
the loops of Henle. buffered by creatinine and uric acid constitute
about 20% of titratable acid.
34. The ascending limb of the loop of Henle is
impermeable to water and is the diluting segment The ammonia-ammonium buffer base system has
of the nephron. The fluid emerging from the a pK of 9.0. Thus, at a pH of 7, the ratio of NH3
ascending limb of the loop of Henle is always to NH4 is 1: 100, and it does not contribute to
hypotonic whether the urine excreted is dilute or titratable acidity. To calculate total acid excretion
concentrated. in the urine, titratable acidity and urinary NH4
concentrations should be summed.
41. The thick ascending limb of the loop of Henle
is impermeable to water. It is permeable only to 60. Type A intercalated cells secrete protons (and
solute. 25% of the filtered Na is recovered in the conserve bicarbonate); acid-secretion by
loop of Henle. That is why loop diuretics are very intercalated cells is stimulated by aldosterone.
efficacious. They inhibit the 1Na-1K-2Cl Proton secretion is also affected by transepithelial
symporter and increase solute delivery to distal voltage; a lumen negative transepithelial voltage
portions of the nephron. The unabsorbed solutes favors secretion of protons and potassium ions in
in the tubular lumen retain their water equivalent the collecting duct. Type B intercalated cells have
and a considerable diuresis results because the been observed in the context of metabolic
reabsorptive capacity of the collecting duct is alkalosis. They express pendrin, a Cl-HCO3
overwhelmed. exchanger, on the luminal membrane, and secrete
bicarbonate in exchange for luminal chloride.
46. Both NaCl and urea contribute substantially
(and to a similar extent) to the tonicity of the 61-70. See Supplement on acid-base physiology
medullary interstitium. below.

56. Tubular fluid is mainly acidified in the distal ************************************************


portions of nephron (distal convoluted tubule and Supplement on acid-base physiology
collecting ducts). Since bicarbonate has been pH = - log [H] in moles/liter
completely reabsorbed by the time fluid reaches Suppose [H] = 100 nM, pH = - log [10-7]
= - 7 [-1] = 7
this portion of the tubule, secreted H acidifies the
tubular fluid. The concentration of phosphate pH Hydrogen ion concentration
increases however and it buffers some of the corresponding to this pH
secreted H. This allows more H to be secreted 7 100 nM
without acidifying the urine beneath a limiting 6 1000 nM
pH. 5 10000 nM
4 100000 nM
56. The highest amount of acid is secreted in the
proximal tubule; however, this contributes mainly H ion concentration is expressed in logarithmic as
to reabsorption of filtered bicarbonate rather than well as linear scale. It is expressed on a log scale
acidification of tubular fluid. In the collecting so that the values become convenient to handle
ducts, the protons secreted acidify tubular fluid. (because hydrogen ion concentration is of the
57. Normally, the ratio of NH4 to titratable acid in order of nanomoles). However, it must be noted
urine is 1 – 2.5. that a unit change in pH say from 6 to 7 represents
a ten-fold change in H ion concentration. The
table above is meant to emphasize this.
89
E.S.Prakash. MCQs in Medical Physiology, May 2013
pH of various body fluids: In contrast, bicarbonate cannot buffer protons
pH [H] Compartment / formed from carbonic acid (see the reaction
concentration condition
in nanomoles
sequence below).
per liter
1 100 million Gastric juice CO2 + H2O = H2CO3 = H + HCO3
6.9 120 ICF (muscle);
arterial plasma in Thus, the protons formed from carbondioxide can
life threatening be buffered only by hemoglobin or plasma
acidosis
7 100 Pure water proteins.
7.1 80 Inside RBC
7.33 50 Cerebrospinal The Henderson-Hasselbalch equation may be
fluid modified to read thus:
7.4 40 Normal arterial
plasma [H+] in nM = 24 × PaCO2 (mm Hg)
7.7 20 Severe alkalosis
8 10 Pancreatic juice [HCO3-] mM
This is the modified Henderson’s equation.
The Henderson-Hasselbalch equation reads thus: Substitute normal values of PaCO2 and plasma
pH = pKa + log [salt] / [acid] HCO3 and you can easily remember the equation.
For example, consider the H2CO3 –HCO3 system Normally, [H+] nmol/L = 40 nmol. This is on the
The acid is H2CO3 left side of the equation.
The conjugate base is HCO3
40 = 24 (a constant) × 40 / 24
[H2CO3] [H+] + [HCO3-]

pH = pK + log [HCO3-] / [H2CO3] On the right side, 40 = normal PaCO2 and 24 =


normal plasma HCO3. The constant 24 has been
If buffer acid were half dissociated, i.e., the derived taking into consideration the solubility
concentration of acid and the conjugate base are coefficient of carbon dioxide at body temperature,
equal, then, [salt] = [acid], and the pH would be so that PaCO2 can be substituted in the equation
equal to pK. A buffer system works best when the instead of carbonic acid.
pH of the solution is close to the pK value.
Anion gap is calculated as:
Buffers in 1 L arterial plasma: {[Na+] + [K+]} – {[Cl-] + [HCO-3]}
Buffer Concentration in
plasma (mM)
All of them are expressed in mmol/L, and the
HCO3 24
Hb 9 anion gap normally varies from 8-16 mmol/L
Protein 15
Total buffer 48 Some authors omit [K+] from the above equation.
base That is OK, but if this is done, then the normal
value will be lower by about 4 mmol/L.
What does HCO3 in ECF buffer?
HCO3 in ECF combines with H liberated by the Here is an easy way to understand the anion gap
formation of fixed acids (lactic acid, ketoacids). • In plasma, the sum of charges of positive ions
Lactic acid → Lactate + H is equal to the sum of negative charges
H is buffered by HCO3 to form H2CO3 • Plasma is thus electroneutral
H2CO3 is unstable. • Some ions in plasma are routinely measured
It dissociates to form CO2 + H2O. (measured ions)
CO2 is volatile. The lungs excrete CO2. • Some are not (unmeasured ions)
The hydrogen ions formed from lactic acid are • What are the major cations in plasma?
eliminated as carbon dioxide.
90
E.S.Prakash. MCQs in Medical Physiology, May 2013
• They are Na and K
• What are the major anions in plasma? Stress Incontinence (SI) – a significant increase
• They are chloride and HCO3 in intra-abdominal pressure (called ‘stress’ here)
• Measured cations are Na and K triggers the emptying.
• Measured anions are Cl and HCO3
• Unmeasured cations are Ca and Mg Urge Incontinence (UI) – the urge to micturate is
perceived, but one is unable to voluntarily inhibit
• Normally, the unmeasured anions are
phosphate, sulfate and protein anions. it at bladder volumes an age-matched normal
individual can hold.
• The concentration of unmeasured cations is
small that it can be ignored
Some individuals have both SI and UI.
• In summary, [measured cations] – [measured
anions] = [unmeasured anions]
A weakness of the pelvic floor (in females) and
the external urethral sphincter (in both males and
Anion gap = {[Na] + [K]} – {[Cl] + [HCO3]} =
females) due to any cause (example, denervation)
[unmeasured anions]
is a basic abnormality underlying both stress and
urge incontinence.
When does the anion gap increase and why?
In metabolic acidosis (for example, due to lactic
Urge incontinence is a bit more problematic
acidosis), pH decreases. The H ions are buffered
because “detrusor instability” (sometimes
by HCO3. The concentration of lactate (an
known as an over-reactive bladder is believed to
unmeasured anion) increases. Therefore, anion
additionally contribute (Cecil 24e, pp. 110). The
gap increases. The increase in anion gap reflects
over-reactive bladder is often ‘idiopathic’ but may
an increase in the concentration of one or more
be due to an upper motor neuron type lesion (see
unmeasured anions in plasma. This happens in
under Chronic Spinal Cord Injury below). Note
metabolic acidosis. The anion gap may also be
however that an over-reactive bladder is not
increased in metabolic alkalosis; one can plug
necessarily one with good contractility; i.e. it may
some chloride, Na and bicarbonate values in and
not empty completely. Detrusor hyperactivity
try. However, in practice, the anion gap is used
may also be associated with a bladder calculus,
mainly in the differential diagnosis of metabolic
urethral obstruction.
acidosis.
Overflow incontinence – A distended bladder
Examples of high anion gap metabolic acidoses contracts and empties in an individual with no
and the unmeasured anions: voluntary control of micturition (see below).
Cause of Unmeasured
metabolic anion(s)
acidosis Acute phase of spinal cord injury (spinal shock):
Lactic acidosis Lactate There is abrupt loss of tonic facilitation of
Ethylene glycol Oxalate parasympathetic motor neurons from supraspinal
poisoning influences. Urinary retention is due to lack of
Ketoacidosis Acetoacetate, beta- neural drive for detrusor contraction but is
hydroxybutyrate additionally likely to uninhibited contraction of
Acute renal Multiple anions the internal urethral sphincter; the bladder
failure; end-stage include phosphate distends (overfills) and empties reflexly (at which
renal disease the point the IUS relaxes), and no voluntary
control is present (“overflow incontinence”);
Disorders of Micturition – Summary nonetheless the contractions of the detrusor are
Definition for Urinary Frequency: > 8 voids per not as forceful and emptying may be incomplete;
day (Cecil 24e, pp. 111) with risk for UTI. Bladder catheterization is
necessary.
Urinary incontinence – inability to voluntarily
restrain bladder emptying;
91
E.S.Prakash. MCQs in Medical Physiology, May 2013
Chronic spinal cord injury
Spinal reflexes below the level of the transection With a lesion in this region, the voiding reflex per
are “released” from the inhibitory influences of se is normal, but there is no voluntary control. The
supraspinal centers. individual may or may not be aware of the
incontinence; commonly called “frontal-type
The basic sacral reflex arc for micturition incontinence”
becomes hyperactive; i.e., the threshold for the
micturition reflex is a lower bladder volume The term “Neurogenic bladder” is a nonspecific
(hyperactive voiding reflex). Marked by bladder term – it may refer to flaccid bladder, or a
wall hypertrophy; reduced bladder capacity; hyperreflexic bladder, and therefore it should be
increased urinary frequency [collectively called used only with further qualification.
Spastic Neurogenic Bladder]. Furthermore,
neural influences on detrusor and the sphincter References: Ganong’s Review of Medical
may not be coordinated in the absence of Physiology (2005); Cecil’s Textbook of Medicine
supraspinal influences (detrusor-sphincter dys- 24e (Ch 25 Incontinence)
synergia).
****************************************
Deafferentation: Sacral afferent roots sectioned
experimentally; equivalent – tabes dorsalis
affecting sacral sensory roots. Here, the brain is
slow to perceive bladder filling – therefore, the
bladder in the long run becomes thin walled,
distended; it becomes hypotonic because of loss
of afferents in the sacral micturition reflex arc.
Contractions of detrusor that occur are an intrinsic
response of smooth muscle to stretch.

Denervation (deafferentation + injury to


efferent nerves innervating the bladder) as with
cauda equine tumors compressing on filum
terminale. In this case, information about bladder
distension does not promptly reach the CNS.
Neither is the efferent nerve supply intact. Thus,
the bladder is effectively “decentralized”

The bladder becomes flaccid (flaccid, areflexic


bladder). The muscle becomes active gradually
with contraction waves expelling dribbles of urine
out of the urethra. Since the spinal reflex arc
itself is interrupted, incontinence of the ‘overflow
type’ occurs here.

Lesion of the parasagittal medial frontal


micturition center, as can happen with
parasagittal meningiomas, hydrocephalus, or with
a stroke of the frontal cortex.

The medial frontal micturition centers in the


cortex are presumably the origins of descending
pathways that activates the voiding reflex.
92
E.S.Prakash. MCQs in Medical Physiology, May 2013
11. Critical Care Physiology A. 150 ml
B. 190 ml
Instructions: In each of the following questions, C. 230 ml
select all correct answers. Each question may D. 270 ml
have one or more correct responses or none at
all. 6. An (acute) increase in intracranial tension does
not occur as a result of:
1. An 8-year-old girl is being treated with A. breath holding in full inspiration
mechanical ventilation and other disease-specific B. ventilation with 20 cm H2O PEEP
therapy for acute respiratory distress syndrome. C. Valsalva maneuver
She is being administered 60% oxygen. Her D. decrease in PaCO2
PaCO2 and PaO2 were found to be 20 and 100 mm
Hg respectively. Which of the following 7. A 60 yr old man is on mechanical ventilation
statements is correct? for management of acute pulmonary edema and
A. Gas exchange in the lungs is normal renal function is normal.
B. There is a defect in pulmonary gas exchange
C. She has alkalosis Results of arterial blood gas analysis are: pH =
7.5, [HCO3] = 24 mmol/L; PaCO2 = 30 mm Hg,
2. An eight-year old boy was put on a ventilator PaO2 = 50 mm Hg. O2 saturation of Hb is 84%.
after he developed acute respiratory failure. FiO2
was 60%, and PaCO2 was found to be 60 mm Hg. Ventilator settings are as follows: tidal volume =
What is the expected O2 tension of arterial blood? 1000 ml, respiratory frequency = 12 breaths per
A. 340 mm Hg min, and FiO2=1.
B. 290 mm Hg
C. 240 mm Hg Which of the following statements is correct with
D. 190 mm Hg regard to management?
A. Minute ventilation should be increased
3. During mechanical ventilation, the addition of B. PEEP should be instituted
positive end-expiratory pressure (PEEP): C. Calcium gluconate must be given
A. is helpful in the management of ARDS D. IV infusion of 20% mannitol is required
B. may decrease cardiac output
C. is associated with increased ADH secretion 8. Does any one of the following physiological
D. allows FiO2 to be reduced parameters in itself indicate that arterial blood
oxygenation in a critically ill patient is
4. A 60-yr old man is being mechanically satisfactory? If so, which one?
ventilated after he developed respiratory arrest. A. (A–a) O2 gradient = 10 mm Hg
The ventilator settings are as follows: VT = 500 B. PaO2 = 95 mm Hg
ml; respiratory rate = 12/min; FiO2 = 60%; and C. O2 saturation of Hb > 90%
PEEP = 5 cm H2O. His arterial [H+] is 20 nM; D. Blood [Hb] = 12 g/dL
arterial plasma HCO3 is 24 mM; and PaO2 is 200
mm Hg. Which of the following statements is 9. In ventilated patients, oxygenation of arterial
correct? blood should be assessed by monitoring:
A. FiO2 will need to be increased A. (A–a) O2 gradient
B. Intracranial tension is elevated B. PaO2
C. Minute ventilation needs to be decreased C. blood hemoglobin concentration
D. PEEP could be increased D. O2 saturation of Hb

5. A 70-kg man is breathing through a tube that 10. The organ that excretes the greatest amount of
has a radius of 5 mm and a length of 1 m. The acid is:
dead space is approximately A. lungs
93
E.S.Prakash. MCQs in Medical Physiology, May 2013
B. kidneys Actual PaO2 = 100 mm Hg;
C. skin (A–a) O2 difference = 290 mm Hg
D. GIT This represents a considerable defect in gas
exchange. In fact, this is the reason why this girl
11. The most common acid-base disturbance in was ventilated in the first place; acute respiratory
patients on mechanical ventilators is: distress syndrome is characterized by hypoxemia
A. metabolic acidosis and normocapnia or hypocapnia (type I
B. metabolic alkalosis respiratory failure). Regarding option C, it is
C. respiratory acidosis impossible to tell whether she has alkalosis
D. respiratory alkalosis because plasma pH is not known.

12. Sodium bicarbonate should not be 2. Use the alveolar gas equation and first
administered to a patient with: determine the oxygen tension of alveolar gas
A. arterial blood pH of 7.1 PAO2.
B. serum [HCO3] of 12 mM PAO2 = [(PB-PH20) × FiO2] – [(PACO2) / R].
C. a PaO2 of 50 mm Hg Here PB = 760 mm Hg, PH20 = 47 mm Hg, FiO2 =
D. a PaCO2 of 68 mm Hg 0.6, R = 0.8].
Since PaCO2 = 60 mm Hg, PACO2 will be about
13. Hyperbaric oxygen therapy is useful in the the same, since CO2 rapidly equilibrates across the
management of (tick all that apply): alveolar capillaries. It is much more soluble than
A. cyanide poisoning oxygen.
B. carbon monoxide poisoning
C. diabetic leg ulcers Substituting, we get, alveolar PO2 = 350 mm Hg.
D. gas gangrene Normally, the (A-a) O2 gradient is less than 10
E. decompression sickness mm Hg. So we would expect arterial PaO2 to be
about 340 mm Hg.
14. Which of the following is not required to
calculate total dead space using Bohr’s equation? Thus, it is clear that alveolar PO2 is much less
A. End-tidal CO2 than PO2 of inspired air. This is because it gets
B. PaO2 saturated with water vapor and then diluted with
C. PaCO2 carbon dioxide in the alveolar gas.
D. Tidal volume
The reason for subtracting PACO2 /R is that
Answers: Critical Care Physiology alveolar PO2 gets diluted by alveolar CO2 which is
1B 2A 3all 4CD 5C equal to arterial CO2 (since CO2 is highly
6D 7B 8 None 9 All 10A soluble). This is divided by R, the respiratory
11D 12D 13 All 14B exchange ratio; i.e., the ratio of the volumes of
CO2 to O2 exchanged across the lungs per unit
Answer Explanations: time. R is for practical purposes assumed to be
1. The approach: 0.8; i.e., 200 ml CO2 / 250 ml O2. Let us say an
• What is the PAO2? equal volume of CO2 and O2 were exchanged;
• What is the expected PaO2? then the denominator would be 1. Since only a
• Is there a difference between expected PaO2 lesser volume of CO2 is exchanged (i.e. 200 ml)
and actual PaO2? and more O2 is taken up (i.e. 250 ml), alveolar
• Alveolar–arterial O2 gradient does not oxygen gets even more diluted by CO2. This is
normally exceed10 mm Hg given by R; i.e. VCO2 /VO2).

Solution: Using the alveolar gas equation, we get: 3. In patients who are on mechanical ventilators,
Expected PAO2 = 400 mm Hg; intrathoracic pressure (ITP) becomes positive
Expected PaO2 = 390 mm Hg; during inspiration, and when PEEP is added,
94
E.S.Prakash. MCQs in Medical Physiology, May 2013
average intrathoracic pressure becomes even more PAO2 = (760 – 47) 0.6 – (20/0.8) = 400 mm Hg
positive. The beneficial effect of PEEP is believed His PaO2 is expected to be at least 380 mm Hg
to stem from the recruitment of previously whereas his arterial PaO2 is only 200 mm Hg. The
atelectatic alveoli and consequently an increase in (A-a) O2 gradient = 180 mm Hg is quiet high and
the surface area and time available for reflects poor gas exchange (a right-to-left shunt).
oxygenation; furthermore, it keeps fluid from
entering the pulmonary interstitium. However, the Increasing FiO2 alone does not ameliorate
hemodynamic effect of raised ITP would be to hypoxemia due to a shunt because the oxygen
reduce venous return and consequently cardiac administered would not equilibrate with the shunt.
output. Note that it is possible that the beneficial In this case, the underlying disease process has to
effect of PEEP on oxygenation might be offset by be addressed.
a low cardiac output. To prevent this from
happening, fluids must be administered, and Increasing PEEP in this patient would certainly
cardiac output has to be monitored. improve oxygenation if the poor gas exchange,
manifest as high (A– a) DO2, were due to a
One or more of the following means could generalized process such as ARDS or cardiogenic
improve oxygenation: pulmonary edema. If that happens, FiO2 could
• Increasing minute ventilation but this has the then be reduced. However, cardiac output has to
effect of removing CO2; further, there is a risk be monitored.
of ‘volutrauma’ with higher tidal volumes.
• Increasing FiO2 but there is a limit to the time In this patient, it is however important to reduce
for which oxygen rich gas could be minute ventilation since it will be difficult to wean
administered; i.e., there is a risk of lung him at a later stage if his PaCO2 is as low as 20
damage due to high FiO2. mm Hg. CO2 is the major stimulus for respiration.
• Administering PEEP; however, this comes If the PaCO2 is too low, the drive for spontaneous
with two risks: one, of barotrauma due to respiration itself is removed.
overdistended lung zones; and two, reduction
in cardiac output due to a decrease in venous In ventilated patients, it is not uncommon to vary
return. The risk of barotrauma and consequent minute ventilation so that PaCO2 is around 45 mm
pneumothorax is significant because even in Hg (i.e. at the upper end of the normal range)
patients with lung disease, not all lung zones though this represents a trade off from the
are affected, and some might be even normal. possibility of better oxygenation. This is called
Lung zones that are normal will be permissive hypercapnia. This is usually
overdistended by the applied PEEP and may accompanied by hypoxemia (also called
rupture. permissive hypoxemia). This is an attempt to
preserve the drive for spontaneous respiration.
4. [H+] = 24 [PaCO2] / [HCO3] This makes it more likely that, as soon as the
Here [H+] = 20 nM, and [HCO3] = 24 mM, so underlying disease process is fully addressed, the
PaCO2 = 20 mm Hg patient could be weaned to start breathing on his
or her own.
An increase in PaCO2 raises intracranial tension
(ICT) by causing cerebral vasodilation. In this 5. Radius = 0.5 cm
case, ICT cannot be high since [H+] is quite low; Length = 1 m = 100 cm
i.e. it is only 20 nmol/L (normal [H+] of arterial Volume of the tube
plasma is 40 nmol/L). Also [HCO3] is normal, so = Π × (radius squared) × length
the patient has respiratory alkalosis. This is = (22/7) × (0.5)2 × 100
because of hyperventilation. = 80 ml

Use the alveolar gas equation and determine his This adds to the normal anatomical dead space of
PaO2. about 150 ml (in a 70 kg man) to make it 230 ml.
95
E.S.Prakash. MCQs in Medical Physiology, May 2013
• Conclusion: D (A-a) O2 only tells us the
6. PaCO2 will gradually rise during breath holding efficiency of oxygen exchange
and it will raise intracranial tension (ICT) by
causing cerebral vasodilation. Intrathoracic Case 2: An anemic patient with a PaO2 of 95 mm
pressure is increased during PEEP and Valsalva Hg performing mild-moderate exercise
maneuver and this will reduce venous return from • PaO2 = 95 mm Hg
the skull and raise ICT. A decrease in PaCO2 per • Hb = 7.5 g/dL
se will produce cerebral vasoconstriction and this • Oxygen carrying capacity is reduced by half
in itself cannot cause ICT to increase. • Significant exercise intolerance due to anemic
hypoxia
7. This patient, who has acute pulmonary edema,
has arterial hypoxemia despite administration of Case 3: A patient who is on a ventilator and has a
100% O2. The expected PaO2 can be calculated to PaO2 of 95 mm Hg
be about 650 mm Hg but his arterial PaO2 is only • Patient is given 60% oxygen
50 mm Hg. This represents a formidable defect in
• His PaCO2 is 40 mm Hg
oxygen exchange. In this situation, institution of
• Then, expected PaO2 is 400 mm Hg
PEEP would improve oxygenation by inflating
atelectatic alveoli. Secondly, PEEP would reduce • If actual PaO2 is 95 mm Hg then, this
represents a defect in gas exchange.
the gradient for filtration of fluid across the
pulmonary capillaries. FiO2 can subsequently be
Case 4: A PaO2 of 90 mm Hg may occur even in
tailored down to an acceptable value; this is
the absence of Hb!
important since prolonged administration of high
concentrations of O2 (especially > 24 h) may
Case 5: A person can have a blood [Hb] of 3 g/dL
result in oxygen toxicity. This patient has
and yet the O2 saturation of Hb may be > 90%;
respiratory alkalosis, a consequence of
hyperventilation. Note that CO2 elimination is
The properties of Hb are such that it is 90%
hardly of any concern even in the face of severe
saturated with O2 even if PaO2 is only 60 mm Hg.
impairment of oxygen exchange. This is because
While this is a safety characteristic, a decline in
CO2 is much more soluble. Patients with
PaO2 below 60 mm Hg is associated with a steady
respiratory alkalosis may develop hypocalcemia
decline in O2 saturation of Hb (please examine the
but in this instance the alkalosis can be readily
oxygen dissociation curve for adult hemoglobin).
corrected by reducing minute ventilation. The
Further, if O2 saturation of Hb is 90% but blood
patient also does not require mannitol since there
Hb is only 3 g/dL, then he will definitely be
is no evidence from the data provided that ICT is
hypoxemic!
raised. In fact, PaCO2 is lower than normal.
Case 6: Neither does a blood [Hb] of 12 g/dL by
8. None of these parameters taken alone will give
itself guarantee adequate oxygenation.
us this information.
An individual might have a blood [Hb] of 12 g/dL
To illustrate, consider the following situations:
and still not be able to transport enough oxygen to
A (A-a) O2 gradient of 10 mm Hg only tells us
the tissues if the hemoglobin itself is not pure. For
that diffusion capacity of the lungs for oxygen is
example, with even small amounts of carboxy
normal.
hemoglobin, the oxygen dissociation curve is
shifted to the left. Simple methods of blood Hb
Case 1: a healthy individual at high altitude
estimation cannot distinguish between normal and
• D (Alveolar – arterial) O2 gradient = 10 mm
abnormal hemoglobin derivatives.
Hg
• His PAO2 = 60 mm Hg 9. All of these parameters should be monitored.
• His PaO2 = 50 mm Hg Furthermore, one should know if hemoglobin
• Problem: moderate hypoxemia itself is normal Hb A or not.
96
E.S.Prakash. MCQs in Medical Physiology, May 2013
14. Bohr’s equation for determining total dead
12. Injected NaHCO3 dissociates to give Na and space: PECO2 × VT = PaCO2 (VT–VD)
HCO3. The HCO3 buffers H ions in ECF and If PECO2 = 28 mm Hg, PaCO2 = 40 mm Hg, and
becomes H2CO3 VT = 500 ml, then, dead space = 150 ml

H2CO3 will dissociate to give CO2 and H2O. End-tidal carbondioxide is measured by
Normally, the lungs eliminate CO2 readily. capnometry. Note that PaCO2 represents gas
However, if the PaCO2 is already as high as 68 equilibrated with perfused alveoli. If there is
mm Hg, this means that minute ventilation is significant V/Q mismatch, PaCO2 will be higher
already inadequate and such a patient would than PACO2.
require mechanical ventilation since he very likely
has respiratory acidosis (type II respiratory VD is an index of dead space ventilation; VD/VT is
failure). The administration of NaHCO3 in this used as yet another index of dead space
instance will aggravate acidosis. So get the CO2 ventilation.
out down to a level that will make him breathe.
Intubate and ventilate! Sodium bicarbonate is ****************************************
appropriate only for the management of metabolic
acidosis – even in metabolic acidosis there are
specific clinical guidelines for its use. It depends
on what the cause of metabolic acidosis and if that
is reversible, the pH and other factors.

13. The principle of hyperbaric oxygenation has


been discussed in the chapter on pulmonary
physiology. 100% oxygen is administered at
pressures exceeding atmospheric (usually 2–3
atmospheres) for brief periods of time. This
greatly increases arterial oxygen tension if
pulmonary gas exchange is normal.

Hyperbaric oxygenation is generally resorted to


only when all other means of increasing oxygen
carrying capacity of arterial blood have failed.

High concentrations of oxygen might serve to


displace cyanide from cytochrome oxidase. In
carbon monoxide poisoning, high concentrations
of oxygen help overcome competitive inhibition
by carbon monoxide. Diabetic leg ulcers due to
obliterative peripheral vascular disease might
benefit from an increase in the content of
dissolved oxygen content of blood. In gas
gangrene, the causative organisms are anerobic
and may be killed by high oxygen concentration.
In decompression sickness, the rationale is to
“recompress” as well as “oxygenate” and this is
achieved by hyperbaric oxygenation, and of
course, this is followed by slow decompression.

97
E.S.Prakash. MCQs in Medical Physiology, May 2013
SELF-SCORERS
Self-Scorer 1: General and cellular basis
of medical physiology
Select the single best answer unless otherwise
Fig A Fig B
instructed.
Which of the following statements is correct?
1. 2 liters of Ringer’s lactate is administered to a
A. B is transported by facilitated diffusion
12-year old boy with isotonic dehydration. What
B. Transport of B may be ATP dependent
will be the change in ICF volume?
C. There is a ‘transport maximum’ for A
A. No change
D. Substance A would move in one direction
B. Increases by 2 liters
only
C. Increase
D. Decreases by 0.5 liter
7. Which of the following mediates ion transport
between the cytosols of adjacent epithelial cells?
2. Cell volume and pressure is mainly dependent
A. Gap junctions
upon activity of:
B. Tight junctions
A. Na glucose cotransporter
C. Desmosomes
B. Na-K pump
D. Hemidesmosomes
C. glucose transporter
D. Na-Ca exchanger
8. Which of the following transport processes is
active? (select all correct answers)
3. Which of the following ions is not transported
A. Acidification of lysosomes
across the cell membrane by a primary active
B. Actin-myosin cross bridge cycling
transport mechanism?
C. Exocytosis
A. Na
D. Micropinocytosis
B. Cl
C. K
9. Which of the following transport processes is
D. Ca
mediated by a carrier protein in the plasma
membrane?
4. Which of the following substances can be used
A. Glucose uptake through SGLT-1
as a marker for the ECF compartment?
B. Na influx through Na channels
A. Nonmetabolizable sugars
C. Na-K ATPase
B. Glucose
D. Water fluxes through aquaporins
C. Radio-iodinated albumin
D. D2O
10. The volume of RBC placed in a NaCl solution
with an osmolality of 280 mOsm/Kg H2O is 100
5. Sodium-glucose cotransport in the intestine and
femtoliters. What will be the steady state volume
kidneys is an example of:
when they are placed in a NaCl solution with an
A. primary active transport
osmolality of 350 mOsm/Kg H2O?
B. secondary active transport
A. 120 fL
C. facilitated diffusion
B. 80 fL
D. passive transport
C. 50 fL
6. Figures A and B given below depict the D. Data inadequate
relationship between transport rates (in Y-axis, in
arbitrary units) and concentration gradients (in X- Answers to Self-Scorer 1:
axis, in arbitrary units) of two substances A and B General and cellular basis of medical
respectively. physiology
1C 2B 3B 4A 5B
98
E.S.Prakash. MCQs in Medical Physiology, May 2013
6B 7A 8all 9AC 10B 9. The term ‘carrier-mediated transport’
emphasizes the fact that the transported species
Notes: binds to a transport protein. Glucose attaches to
1. Ringer’s lactate is an isoosmotic solution; the GLUT molecule and a conformational change
however, as lactate is taken up by liver and other in GLUT shuttles glucose to the interior of the
cells and metabolized, the infusion of it adds a cell. The transport of Na and K by Na-K ATPase
definite amount of water to plasma that dilutes is also an example of carrier-mediated transport
plasma in the steady state. This will, in turn, cause because both ions bind to the transporter.
some water to enter cells. It is hard to tell what the
actual increase in ICF volume will be but one In contrast, aquaporins and ion channels are not
would expect ICF volume to increase especially carrier proteins; they are channels through which
when a large volume of Ringer’s lactate is water or the respective ions flow. The ions
administered. themselves do not bind to ion channels. This is an
example of diffusion facilitated by the presence of
2. By pumping 3 Na ions out of the cell for every proteins in the cell membrane (facilitated
2 K ions pumped into the cell, the Na-K ATPase diffusion). Some authors also call it “simple
effectively removes one osmole (and the diffusion”.
corresponding water equivalent) from the cell.
Thus, inhibition of Na-K pump would cause cells 10. The following mathematical relationship
to swell. applies to the situation described in the question.
Pi × Vi = Pf × Vf
3. Chloride transport across the cell membrane Pi and Pf are initial and final osmolalities and Vi
occurs by secondary active transport mechanisms and Vf are initial and final volumes respectively
or diffusion but not by a primary active transport 280 × 100 = 350 × Vf
mechanism. Vf = 80 femtoliters
Thus, there is a decrease in RBC volume in the
4. Examples of substances that distribute steady state when it is placed in hypertonic saline.
exclusively in the ECF include inulin, mannitol,
and sucrose. Isotonic solutions of inulin, mannitol **************************************
and sucrose can all be used to determine ECF Self-Scorer 2: Physiology of nerve and
volume. Radioiodinated albumin may be used for muscle cells
estimating plasma volume.
In each of the following questions, select the
5. The complete absorption of glucose from the single best response.
intestinal lumen (against a concentration gradient)
is driven by the electrochemical gradient of 1. Which one of the following increases
sodium. This is a classic example of secondary excitability of cardiac muscle?
active transport. A. Increase in ECF [K+] from 5 to 10 mM
B. Increase in ECF [K+] from 5 to 28 mM
6. B may be transported either by facilitated C. Decrease in ECF [K+] from 5 to 1.4 mM
diffusion or active transport. Both are like enzyme
catalyzed reactions and exhibit saturation kinetics. 2. The force of skeletal muscle contraction cannot
Diffusion rate is proportional to the concentration be increased by:
gradient when other factors affecting diffusion are A. increasing the frequency of activation of
held constant. motor units
B. increasing the number of motor units activated
8. All of them are examples of active processes. C. increasing the amplitude of action potentials in
There are proton translocating ATPases in motor neurons
lysosomes. D. recruiting larger motor units

99
E.S.Prakash. MCQs in Medical Physiology, May 2013
3. Which of the following is correct with regard C. nerve action potential recordings
to numbers of each of the following in the CNS? D. clinical examination of tendon jerks
A. neurons >> glia > synapses
B. glia >>> synapses >>> neurons Answers to self-scorer 2:
C. synapses >> glia > neurons Physiology of nerve and muscle cells
D. synapses >> neurons >> glia 1A 2C 3C 4C 5B
6C 7D 8A
4. Na is not included in the assessment of the
approximate value of the resting membrane Notes:
potential of nerve fibers because: 1. Using the Nernst equation (below) to determine
A. extracellular [Na] is higher than ICF [Na] the equilibrium potential of K, we get, equilibrium
B. Na-K pump extrudes 3 Na for 2 K pumped in potential for potassium
C. permeability of the resting membrane to Na is Ek = - 61 log [(K)i / (K)o]
low Case a: Ek ≈ RMP = - 70 mV
D. unexcited cells are equally permeable to Na Case b: Ek ≈ RMP = - 40 mV
and K Case c: Ek ≈ RMP = - 122 mV

5. Neuron C responds to stimulation of neuron A When RMP is – 70 mV, the muscle cell is closest
with a propagated action potential only when to the firing level, which is about – 55 mV. In
neuron B (which terminates only on neuron C) is contrast, when RMP is – 40 mV, it has crossed the
also simultaneously stimulated. Which of the firing level so it would be absolutely refractory or
following is correct? inexcitable. Hypokalemia makes the RMP more
A. Neuron C releases glycine negative, and because at RMP the heart would be
B. Neuron B releases an excitatory filling with blood, it may stop the heart in
neurotransmitter diastole. Similarly, intense stimulation of the
C. The synapse between A and C is axosomatic vagus nerve to the heart may stop the heart in
D. The neurotransmitter released by neuron A diastole because acetylcholine activates an
opens K channels in neuron C outwardly directed potassium current in SA node;
i.e. it hyperpolarizes the SA node and the AV
6. Which of the following nerve fibers is most node.
susceptible to hypoxia?
A. Somatic motor neurons 2. There is no “larger” action potential; rather,
B. Nociceptive afferents action potentials are all or none.
C. Preganglionic autonomic neurons
D. Fibers transmitting touch sensation 3. The ratio of synapses to neurons in the brain is
about 1000; glia outnumber neurons by about 50
7. The term ‘size principle’ refers to the fact that: times. See p. 51 in Ch 2 and p. 85 in Ch 4 in
A. neurons in small motor units conduct slowly WFG, 2005.
B. chronically denervated skeletal muscle
decreases in size due to reduced release of 4. The resting cell membrane is 50 times more
growth factors from motor neurons permeable to K than Na. When Na and Cl are
C. small motor units are recruited for more removed from the Goldman Hodgkin equation, it
forceful contractions becomes the much simpler Nernst equation.
D. fast motor units are recruited after slow motor
units 5. It is easy if you draw a schematic diagram. You
will see that both neurons A and B have to release
8. Recruitment and activation of motor units is excitatory neurotransmitters in order to fire C (i.e.,
investigated by: by spatial summation of EPSP). However,
A. electromyography repeated stimulation of either A or B may also
B. electroencepaholgraphy
100
E.S.Prakash. MCQs in Medical Physiology, May 2013
elicit an action potential in C by temporal
summation. 4. In which sensory transduction pathway does
hyperpolarization of the receptor cell activate
6. Type B fibers are most susceptible to hypoxia. transmission in the pathway?
A. Phototransduction
7. Slow motor units are activated by slowly B. Auditory signaling
conducting axons; fast motor units are innervated C. Gustation
by fast conducting axons. Slow units are recruited D. Olfactory signaling
first in most movements; fast motor units, which
are easily fatigable, are recruited for more 5. The muscle spindle is not innervated by:
powerful contractions. See p. 76, WFG, 2005. A. Aα fibers
B. Aγ fibers
8. In this technique, compound muscle action C. Ia fibers
potentials (CMAP) are recorded by placing D. II fibers
electrodes either on the surface of muscles or
within the muscle (when needle electrodes are 6. The muscle spindle remains capable of
used); it is much like ECG which represents the responding to stretch even the muscle is
compounded potentials generated by heart contracting. This is because of:
recorded by placing electrodes on the body A. presence of stretch receptors in the tendon
surface. CMAP is examined at rest, during mild B. co-activation of α- and γ-motor neurons
contraction, and forceful contractions. The pattern C. reciprocal innervation
of EMG during these maneuvers is useful to D. autogenic inhibition
distinguish between muscle weakness due to
myopathy and neuropathy. 7. Muller’s doctrine of specific nerve energies is
better known as:
************************************* A. Bell-Magendie law
Self-Scorer 3: Functions of the CNS B. labeled-line principle
C. Weber-Fechner law
1. Which of the following cells in the CNS is D. law of projection
primarily involved in the reuptake of excitatory
neurotransmitters released by neurons? 8. The receptor for autogenic inhibition (or the
A. Astroglial cell inverse stretch reflex) is:
B. Microglial cell A. hair cell
C. Oligodendroglial cell B. muscle spindle
D. Ependymal cell C. Golgi tendon organ
D. glomus cell
2. Which of the following effects is not produced
by stimulation of µ opioid receptor? 9. Proprioceptive information reaches
A. Analgesia consciousness through the (choose all correct
B. Constipation answers):
C. Miosis A. spinothalamic system
D. Diuresis B. dorsal column – medial lemniscal pathway
E. Sedation C. spinocerebellar pathway
F. Euphoria D. olivocerebellar pathway

3. The phantom limb phenomenon exemplifies: 10. If light reflex is present and the
A. Bell-Magendie law accommodation reflex absent, then the lesion is
B. the law of projection most likely in the:
C. Muller’s doctrine of specific nerve energies A. pretectal nucleus
D. Weber-Fechner law B. ganglion cells
101
E.S.Prakash. MCQs in Medical Physiology, May 2013
C. Edinger-Westphal nucleus D. 40 minutes
D. visual association cortex
18. Which of the following statements regarding
11. REM behavior disorder is primarily the autonomic nervous system is incorrect?
characterized by: A. Preganglionic sympathetic neurons are more
A. lack of REM sleep sensitive to hypoxia compared to
B. day-time somnolence postganglionic sympathetic neurons.
C. absence of hypotonia during REM sleep B. Postganglionic fibers are unmyelinated.
D. absence of PGO spikes during REM sleep C. Medulla is the highest center for integration of
autonomic reflexes.
12. Which sensory modality is transduced by free D. Stimulation of parasympathetic system usually
nerve endings? elicits discrete responses whereas stimulation
A. Vision of the sympathetic nervous system usually
B. Taste elicits widespread physiologic responses.
C. Smell
D. Sound 19. Normally, the total blood flow to the brain is
about:
13. The fifth taste modality is: A. 250 ml/min
A. umami B. 500 ml/min
B. imami C. 750 ml/min
C. himami D. 1200 ml/min
D. emami
E. mami 20. Normally, cerebral metabolic rate for oxygen
(CMRO2) is about:
14. Muscle contraction in response to maintained A. 2 ml/100 g/min
stretch is initiated by sensory inputs from: B. 3.5 ml/100 g/min
A. nuclear bag fibers C. 5 ml/100 g/min
B. nuclear chain fibers D. 7 ml/100 g/min
C. Golgi tendon organ
D. γ-motor neurons 21. Normally, the brain is perfused with what
fraction of resting cardiac output?
15. Rhodopsin has peak sensitivity to light at a A. 5%
wavelength of: B. 10%
A. 405 nm C. 15%
B. 505 nm D. 20%
C. 605 nm
D. 705 nm 22. In which sensory system does excitation of
receptor by an adequate stimulus result in
16. Which one of the following sensory hyperpolarization of receptor cells?
modalities does not have a separate neocortical A. Visual pathway
projection? B. Auditory pathway
A. Taste C. Taste pathway
B. Smell D. Olfactory signaling
C. Vision
D. Hearing 23. Which of the following has been suggested as
a possible mediator of analgesia produced by
17. Dark adaptation is nearly maximal in about: stimulation of the raphespinal pathway?
A. 1 minute A. Substance P
B. 5 minutes B. Glutamate
C. 20 minutes C. Serotonin
102
E.S.Prakash. MCQs in Medical Physiology, May 2013
D. GABA 1. Indeed, reduced reuptake of excitatory
neurotransmitters such as glutamate by astrocytes
24. Which area is uniquely concerned with color in ischemic zones is said to contribute to
vision? excitotoxicity in stroke.
A. V1
B. V3 2. Diuresis is mediated by an action on kappa
C. LO receptors. See Table 4-5 on p. 113, Ch 4, WFG,
D. V8 2005.

25. Which eye movements bring new objects of 3. When light strikes retina, conformational
interest onto the fovea? changes in rhodopsin activate phosphodiesterase,
A. Saccades decreasing cGMP, causing closure of Na channels
B. Convergence movements and hyperpolarization of receptor cells and
C. Smooth pursuit movements decreased release of neurotransmitter. This
D. Vestibular movements triggers response in other neural elements of the
retina. See Fig 8-19, p. 159 in WFG 22nd ed.
26. Kluver-Bucy animals exhibit:
A. hypersexuality 6. Since α- and γ-motor neurons are coactivated,
B. visual agnosia intrafusal fibers also contract whenever extrafusal
C. bizarre exploratory behaviour fibers contract. Thus, the central portion i.e. the
D. amnesia spindle is stretched and fires even while the
E. all of the above muscle is contracting; thus the spindle is capable
of responding to rate of change of stretch, as well
27. Symptoms of the Kluver-Bucy syndrome have as maintained stretch.
been shown to be reproduced by:
A. amygdalectomy 7. The same sensation is perceived no matter how
B. dorsal rhizotomy a sensory pathway is excited. Each pathway is
C. decerebration labeled to transmit a specific sensory modality.
D. lesions in the internal capsule This is the labeled line principle or Muller’s
doctrine of specific nerve energies.
28. In a majority of lefthanders, the left cerebral
hemisphere is the categorical hemisphere. 8. Autogenic inhibition is a term that is used
True/False. interchangeably with inverse stretch reflex. It
indicates that the relaxation that occurs when
29. The planum temporale is an area in the muscle is excessively stretched arises from a
superior temporal gyrus concerned with language- receptor located in the muscle itself. This receptor
related auditory processing. True/False. is the Golgi tendon organ.

30. Lumbar CSF pressure varies from 70-180 mm 9. Impulses in the spinothalamic tracts and the
CSF. True / False dorsal column-medial lemniscus pathways reach
consciousness. Impulses in the spinocerebellar
Answers to Self-Scorer 3: Functions of CNS pathways terminate in the cerebellar cortex and
1A 2D 3B 4A 5A therefore do not reach consciousness.
6B 7B 8C 9AB 10D
11C 12C 13A 14B 15B 10. This is what happens in cortical blindness.
16A 17C 18C 19C 20B
21C 22A 23C 24D 25A 11. In this condition, hypotonia does not occur
26E 27A 28T 29T 30T during REM sleep and the person may act out his
dreams. It is treated with benzodiazepines.
Explanations:
103
E.S.Prakash. MCQs in Medical Physiology, May 2013
14. The nuclear bag fiber is activated whenever Self-Scorer 4: Endocrinology and
the spindle is stretched; i.e. it detects a change in Reproduction
length of the muscle. The nuclear chain fiber, a
slowly adapting receptor, fires even during In each of the following questions, choose the
maintained stretch; i.e. it detects the absolute single best response.
length of the muscle. See p. 131, Ch 6, WFG,
2005. 1. JAK-STAT pathways mediate the effects of:
A. transducin
15. Steady state neurotransmitter release from rod B. aquaporins
cells occurs in the dark. Light causes activation of C. gusducins
cGMP - phosphodiesterase, reducing intracellular D. growth hormone
cGMP, this resulting in closure of Na channels,
hyperpolarization of rod cells and reduced release 2. Men castrated before puberty grow taller
of neurotransmitter from rod cells. because:
A. release of growth hormone is increased
17. Dark adaptation requires synthesis of more B. of androgen resistance
rhodopsin. C. estradiol level in the epiphyseal growth plate
takes longer to reach its threshold
18. The hypothalamus is the highest center for D. of insensitivity to IGF-1
integration of autonomic reflexes. Charles
Sherrington said, “the hypothalamus is the head 3. Patients with sexual precocity are apt to be
ganglion of the autonomic nervous system”. dwarf because of:
A. increased levels of androgens
20. Mass of the brain = 1400 g B. the estrogen surge during precocious puberty
Cerebral oxygen consumption per minute = 50 ml C. deficiency of androgen receptors
CMRO2 (i.e. oxygen consumption per minute per D. increased levels of thyroxine
100 g of brain tissue) = 3.5 ml/100 g/min
4. The satiety-producing hormone is:
21. Resting cardiac output = 5 L/min A. orexin
Cerebral blood flow = 750 ml/min (this is 15% of B. resistin
cardiac output) C. adiponectin
D. leptin
23. Serotonin may act presynaptically on the
‘gate’ in the dorsal horn of the spinal cord and 5. Hypophysectomy does not immediately affect
reduce the efficacy of transmission in the pain the secretion of:
pathway. Enkephalins are also implicated in this A. zona glomerulosa
mechanism. See p. 146, WFG, 2005. B. zona fasciculata
C. zona reticularis
25. Saccades are sudden jerky movements and
occur as gaze shifts from one object to the other. 6. Which hormone also upregulates its receptors
They are programmed in the frontal eye field (in in the adrenal cortex?
the cerebral cortex) and the superior colliculus. A. Epinephrine
B. Angiotensin II
26, 27. Kluver & Bucy observed the effects of C. Cortisol
bilateral temporal lobectomy. However, it is now D. Aldosterone
known that these features could be reproduced by
bilateral resection of the amygdalae. 7. In glucocorticoid-remediable aldosteronism:
A. angiotensin II has no effect on aldosterone
*************************************** synthesis

104
E.S.Prakash. MCQs in Medical Physiology, May 2013
B. angiotensin II stimulates whereas angiotensin 14. The “penis-at-12” syndrome occurs due to a
III inhibits aldosterone synthesis deficiency of:
C. aldosterone synthase is expressed in the zona A. 17α hydroxylase
fasciculata and exquisitely sensitive to B. testosterone 5α reductase
stimulation by ACTH. C. 11β hydroxysteroid dehydrogenase
D. administration of glucocorticoids increases D. CYP 450scc
ACTH levels
15. The most abundant androgen in the plasma of
8. What is true about development of a 44 XX, both males and females is
SRY+ zygote? A. androstenedione
A. Female internal and external genitalia develop B. dehydroepiandrosterone sulfate
B. Both ovaries and testes develop C. testosterone
C. Ovaries develop D. dihydrotestosterone
D. Testes develop
16. In humans, the hormone that is mainly
9. Which of the following zones in the adrenal secreted by the adrenal medulla is:
cortex does not normally express 17 alpha A. epinephrine
hydroxylase? B. norepinephrine
A. Zona glomerulosa C. dopamine
B. Zona fasciculata D. adrenomedullin
C. Zona reticularis
17. The major stimulator of PTH release from the
10. The blood-testis barrier is formed by tight parathyroid gland is:
junctions between: A. vitamin D
A. Leydig cells B. a fall in plasma ionized calcium
B. primary spermatocytes C. calcitonin
C. spermatogonia D. a fall in plasma phosphate concentration
D. Sertoli cells
18. In postnatal life, thyroid hormones do not
11. Human menopausal gonadotropin consists of: increase oxygen consumption in the:
A. ß-subunits of hCG A. heart
B. FSH & LH B. skeletal muscle
C. prolactin C. brain
D. recombinant GnRH D. liver

12. The hormone that is essential for the 19. Which of the following hormones is
formation of male external genitalia is: lactogenic?
A. testosterone A. Chorionic growth hormone
B. dihydrotestosterone B. HCG
C. Mullerian regression factor C. Oxytocin
D. SOX factor D. Relaxin

13. Testicular feminization syndrome is caused by 20. Which of the following inhibits lactation?
mutations in genes coding for: A. Prolactin
A. androgen-binding proteins B. Chorionic somatomammotropin
B. testosterone 5α reductase C. Estrogen and progesterone
C. androgen receptor D. Growth hormone
D. inhibin
21. Which of the following inhibits ovulation?
A. Prolactin
105
E.S.Prakash. MCQs in Medical Physiology, May 2013
B. Human menopausal gonadotropin • Leptin is a satiety signal from the adipocytes
C. Estrogens acting on the hypothalamus to inhibit food
D. Luteinizing hormone releasing hormone intake.
• It presumably signals the amount of fat to the
22. Which of the following hormonal CNS.
combinations in plasma is associated with normal • Chemically, it is a polypeptide. It is coded by
lactation? the ob gene.
A. Cortisol + prolactin + oxytocin • Leptin levels increase after a meal
B. Estrogen (E) + progesterone (P) + prolactin • Leptin receptors are coded by db gene.
C. E + P + insulin + cortisol + prolactin • Leptin deficiency as well as deficiency of
D. Prolactin + oxytocin + E + P functional leptin receptors have been
implicated in the pathogenesis of obesity.
23. During normal pregnancy, which hormone(s)
• Leptin increases sensitivity to insulin.
peak during the first trimester? (select all that
• Adiponectin is another hormone from
apply)
adipocytes that increases insulin sensitivity.
A. HCG
B. HCS • Significant weight loss (fat loss) is
accompanied by a reduction in serum leptin
C. Prolactin
level.
D. Estradiol
E. Estriol • When serum levels of leptin are abnormally
F. Relaxin low, GnRH secretion is eventually
downregulated. Low levels of leptin may in
Answers: Self-Scorer 4 - Endocrinology & part explain amenorrhea that occurs in athletes
Reproduction and individuals with thyrotoxicosis.
1D 2C 3B 4D 5A
5. Although ACTH is trophic to all three zones in
6B 7C 8D 9A 10D
the adrenal cortex, the major stimulators of
11B 12B 13BC 14B 15B
aldosterone synthesis is angiotensin II, and a rise
16A 17B 18C 19A 20C
in plasma [K]. In the long run, however, the size
21A 22A 23AF 24 25 of the zona glomerulosa would also decrease.
Answer Explanations: 7. The basic defect in glucocorticoid-remediable
1. Janus tyrosine kinases (JAK) are enzymes that aldosteronism (GRA; sometimes called
phosphorylate signal transducers and activators of dexamethasone remediable aldosteronism) is a
transcription (STAT). STAT proteins are hybrid aldosterone synthase gene that has a
transcription factors. Growth hormone, prolactin, promoter sensitive to stimulation by ACTH, and
and erythropoietin act via JAK-STAT pathways. this gene is expressed in the zona fasciculata.
(Note, normally, aldo synthase is expressed only
2. The testes is virtually the only source of circulating
testosterone and very little testosterone is synthesized by
in the ZG). The ZF is 5 times larger than the zona
the adrenal cortex. Thus, orchiectomy prior to puberty fasciculata, and therefore aldosterone levels are
will result in a steep fall in testosterone levels in males; high even with normal levels of ACTH. The
consequently the pubertal surge in androgens would be hypertension that occurs as a result of
deficient. Thus, the formation of estradiol in the hyperaldosteronism suppresses plasma renin
epiphyseal growth plates from circulating testosterone is activity. Hyperaldosteronism in this instance is
also greatly reduced. In males as well as females, closure remediable with glucocorticoid therapy (‘low dose
of epiphyseal plates occurs is mediated by estradiol
(acting via estrogen receptors) when its level in the
dexamethasone’); i.e., administered
growth plate exceeds a certain threshold level. This glucocortiocoids inhibit ACTH release and
happens following the pubertal growth spurt consequently reduce aldosterone synthesis.

4. All you need to know about leptin: 8. The sex determining region of the Y
chromosome dictates the formation of embryonic
106
E.S.Prakash. MCQs in Medical Physiology, May 2013
testes and inhibits the development of ovaries. • In T 5α- reductase deficiency, during puberty,
Mullerian inhibiting polypeptide from the testes surges in LH are greater because of the lack
inhibits the growth of Mullerian duct structures. of negative feedback effect of DHT on the
Testosterone induces the development of male pituitary and the hypothalamus; thus, induces
internal genitalia. synthesis of large amounts of T.
• Because of DHT deficiency, feedback
13. Testicular feminizing syndrome (by inhibition of LH release is lower than normal.
definition testes are present but external genitalia • ‘High levels’ of T via their effects on the
are female) androgen receptor (normal in this case) may
Genotype 46XY induce growth of the clitoris.
Gonad Testes
• However, despite high levels of T, labioscrotal
Testosterone Levels normal or
increased
fusion does not still occur because androgen
Internal genitalia Male or female receptors are downregulated at this site.
External genitalia Female • These individuals have been raised as females
Phenotype Female and they may prefer to change their gender
Defects Loss of function identity at the time of puberty; the
mutations in genes management of this condition is complex.
coding for either the
androgen receptor or
15. DHEA sulfate and androstenedione are
testosterone 5α
reductase (type 2) biologically very weak androgens if at all. They
function mainly as precursors for the synthesis of
This condition is an example of male testosterone in males and estrogens in females.
pseudohermaphroditism. Testosterone levels are However, when large amounts of adrenal
increased in this condition. When the basic defect androgens are present and depending on when the
is a mutated androgen receptor, there is resistance level of adrenal androgens is high, it can induce
to actions of testosterone and dihydrotestosterone virilization in a female fetus or in postpubertal
and the resulting clinical syndrome is called females.
androgen resistance syndrome. The resistance
can vary from mild (partial androgen 18. T4 and T3 increase the oxygen consumption
insensitivity) to severe (complete androgen of all metabolically active tissues. The exceptions
resistance). Both testosterone and DHT act upon are the adult brain, testes, uterus, lymph nodes,
the same androgen receptor. DHT is much more spleen and anterior pituitary.
potent and is essential for formation of male
external genitalia. 20. Estrogens stimulate breast growth but
antagonize the milk producing effect of prolactin;
Causes of male pseudohermaphroditism: lactation commences once the placenta is expelled
• Androgen resistance syndrome and there is an abrupt decline in circulating levels
• Testosterone 5-alpha reductase (type 2) of estrogens and progesterone.
deficiency
• Deficiency of steroidogenic acute regulatory 21. The best answer is prolactin. Prolactin
protein (STAR), rare inhibits the release of GnRH from the
hypothalamus, inhibits release of FSH and LH
• 17α- hydroxylase deficiency, rare
from the anterior pituitary and also inhibits the
actions of FSH and LH on the ovary. These
14. Pathogenesis of the “penis-at-12 syndrome”:
effects of prolactin constitute the mechanism of
• It occurs due to a deficiency of the enzyme
lactational amenorrhea. However, oral
testosterone 5α- reductase (type 2)
contraceptive pills which inhibit ovulation usually
• Testosterone (T) and DHT act on the same contain a combination of estrogen and
receptor progesterone, and the dose is pharmacologic.
• DHT is much more potent than T
107
E.S.Prakash. MCQs in Medical Physiology, May 2013
************************************** 6. In which of the following conditions is a long
Self-Scorer 5: Gastrointestinal acting analog of somatostatin of therapeutic
Physiology value?
A. Acute mesenteric ischemia
1. Normally, instillation of acid into the B. Bleeding from esophageal varices
duodenum to reduce pH in its lumen to 4: C. Cholelithiasis
A. stimulates gastrin release D. Pancreatic exocrine insufficiency
B. increases output of an enzyme rich pancreatic E. Pernicious anemia
juice
C. increases activity of Brunner’s glands 7. Which of the following inhibits gastric acid
D. relaxes the pyloric sphincter secretion by an action on the parietal cell?
A. Acetylcholine
2. Which of the following statements regarding B. Epinephrine
the actions of gastrointestinal hormones is C. Gastrin
incorrect? D. Histamine
A. Secretin inhibits gastric emptying E. Prostaglandin E
B. Gastrin stimulates histamine release in the
stomach 8. The ‘postprandial alkaline tide’ is abolished by:
C. Somatostatin reduces blood flow to the GIT A. antrectomy
D. GIP inhibits the release of insulin from B. gastrin
pancreas C. parietal cell vagotomy
D. total inhibition of gastric H+-K+ ATPase
3. The prokinetic effects of erythromycin are due E. truncal vagotomy
to its agonist actions on:
A. histamine receptors 9. Which pattern of motility hastens the transit of
B. motilin receptors chyme in the small intestine in the digestive state?
C. CCK receptors A. Mass action contraction
D. somatostatin receptors B. Migrating motor complex
C. Peristalsis
4. Which of the following mechanisms allows D. Segmentation contraction
complete absorption of glucose from the intestinal E. Tonic contraction
lumen into the cytosol of the enterocyte?
A. Simple diffusion through GLUT-5 10. Intestinal absorption of which of the following
B. Na-glucose cotransporter-1 (SGLT-1) does not directly utilize a Na+ gradient?
C. Na channels in enterocyte membrane A. Fructose
D. Paracellular uptake via gap junctions B. Galactose
C. Glucose
5. Which of the following strategies would most D. Phenylalanine
likely be therapeutic in an individual with E. Tyrosine
achalasia?
A. Injection of tetanus toxin into the lower 11. The release of gastrin from G cells in the
esophageal sphincter antrum of the stomach is inhibited by:
B. Surgical division of the lower esophageal A. activation of vagal efferent fibers to the
sphincter stomach
C. Administration of a muscarinic receptor B. circulating epinephrine
agonist C. blood-borne calcium
D. Administration of a nitric oxide synthase D. mechanical distention of the stomach
inhibitor E. somatostatin

108
E.S.Prakash. MCQs in Medical Physiology, May 2013
12. Which of the following muscles is subject to 2. Secretin inhibits gastric emptying by
control by voluntary as well as reflex contracting the pyloric sphincter. Gastrin from G
mechanisms? cells stimulates acid secretion by two means: one,
A. External anal sphincter it stimulates histamine release from ECL cells in
B. Lower esophageal sphincter the stomach; two, it directly acts on parietal cells
C. Pyloric sphincter to augment acid secretion. Somatostatin reduces
intestinal blood flow and secretion.
13. From a physiologic standpoint, the advantage
of a ‘parietal cell vagotomy’ over truncal Glucose-dependent insulinotropic polypeptide
vagotomy for treating duodenal ulcer is that in (GIP) was previously called gastric inhibitory
parietal cell vagotomy: peptide. However, now it is clear that the
A. a gastrojejunostomy is not required physiologic action of GIP is stimulation of insulin
B. only basal acid output is reduced secretion. Its inhibitory effects on gastric secretion
C. G cells are also denervated occur only at pharmacologic doses.
D. pyloric sphincter is also denervated
E. receptive relaxation is abolished 3. Erythromycin is a motilin receptor agonist. It is
used in the management of diabetic gastroparesis.
14. Your patient is a 30-year-old man with
recurrent right upper quadrant pain suggestive of 4. Glucose could enter enterocytes from the
biliary pain. There is no clinical evidence of liver intestinal lumen by diffusion through GLUT;
cell disease, gallstones in the gall bladder or the however, for complete absorption of glucose
bile duct, gastroesophageal reflux, peptic ulcer against a concentration gradient, the active
disease. However, mild and transient elevations in transport mechanism provided by the sodium-
serum transaminases were documented during an glucose cotransporter is required.
episode of ‘biliary pain’. Ultrasonography of the
hepatobiliary system following an intravenous 8. The H+- K+ ATPase in the parietal cell is the
infusion of cholecystokinin indicated that it final common pathway for gastric acid secretion
caused dilation of the common bile duct. Further, by the stomach. The postprandial alkaline tide
the pancreatic duct dilated during intravenous refers to a slight increase in pH of plasma
infusion of secretin. These findings are most following a meal. It reflects gastric acid secretion
consistent with the possibility of: by parietal cells since for every proton secreted by
A. defective pumping of bile by gall bladder the parietal cell, one bicarbonate ion enters the
B. spasm of the sphincter of Oddi blood stream. Though the liver, pancreas and the
C. a defect in secretion of bile by hepatocytes small intestine collectively secrete a large amount
D. a defect in processing of bile by of alkaline fluid into the GI lumen, this does not
cholangiocytes produce a postprandial acid tide in plasma
E. a defect in processing of pancreatic secretion because the stomach secretes about 2-2.5 liters of
by pancreatic duct epithelial cells gastric juice per day, and parietal cells concentrate
protons a million fold in gastric juice relative to
Answers: Self-Scorer 5 - Gastrointestinal plasma.
Physiology
1C 2D 3B 4B 5B 10. Fructose is not absorbed by an active transport
6B 7E 8D 9C 10A process. It enters enterocytes down its
11A 12A 13A 14B concentration gradient through GLUT-5.

1. The acidity of duodenum drives the release of 11. Options A-D are factors that stimulate gastrin
secretin thereby evoking alkaline secretions from release.
the liver, pancreas and intestine.
13. Option A is correct because the denervation in
a parietal cell vagotomy (PCV) involves the
109
E.S.Prakash. MCQs in Medical Physiology, May 2013
branches that supply the fundic area but not the
antrum. As a result antral motility is preserved 3. Conduction speed is slowest in the:
and a gastrojejunostomy (GJ) is not required. A. SA node
B. atrial pathways
PCV will reduce BAO as well as meal stimulated C. bundle of His
acid output. D. Purkinje system

G cell innervation remains intact in PCV. The 4. The propagation of repolarization from the
innervation of the pyloric sphincter is preserved in ventricular epicardium to endocardium is
PCV. represented by the
A. QRS complex
Receptive relaxation is lost after PCV because a B. QT interval
vagovagal reflex that requires intact efferent C. T wave
innervation of the fundus mediates it; but the loss D. TP period
of receptive relaxation after PCV is not the reason
why it is advantageous over a truncal vagotomy. 5. T wave inversion occurs when ventricular
repolarization occurs from:
14. Based on anatomic and physiologic principles, A. endocardium to epicardium
with the information provided, it can be deduced B. epicardium to endocardium
that there is increased resistance to flow of bile C. apex to base of the heart
and pancreatic juices into the duodenum following D. base to apex of the heart
administration of secretin and CCK. Spasm of the
sphincter of Oddi is the best explanation, as it 6. The most reliable index of AV nodal delay is:
regulates the flow of bile as well as pancreatic A. PR interval
juice into the duodenum. A diagnosis of sphincter B. PR segment
of Oddi dysfunction is established by sphincter of C. AH interval
Oddi manometry (performed usually during D. PA interval
endoscopic retrograde
cholangiopancreatography). Spasm of the SOD is 7. Cardiac muscle cannot be tetanized because of:
a potential cause of recurrent biliary pain A. accommodation
following cholecystectomy, and may underlie B. its slow rate of repolarization
pancreatitis that is otherwise unexplained. C. calcium influx during phase II
D. voltage inactivation of Na channels at
************************************** membrane potentials < 80 mV
Self-Scorer 6: Cardiovascular
Physiology 8. When you stand up, pressure falls in:
A. carotid arteries
1. What is the primary ionic basis of the B. brachial arteries
prepotential in the SA node? C. lower limb arteries
A. Ca influx through transient T Ca channels D. all arteries
B. Inwardly directed long-lasting Ca current
C. Outward Na current 9. If QRS deflection is highest and upright in lead
D. Potassium efflux through leak channels I and equiphasic (or null) in lead aVF, then mean
electrical axis of the QRS vector in the frontal
2. Normally, the impulse that excites the left plane is about:
ventricular myocardium originates in the: A. 30 degrees
A. SA node B. 0 degrees
B. Purkinje system C. +45 degrees
C. left bundle branch D. +90 degrees
D. ventricle
110
E.S.Prakash. MCQs in Medical Physiology, May 2013
10. What would be the change in blood flow to a D. A doubling of pressure in the veins of the foot
tissue if radius of the arterioles in that tissue is
doubled and perfusion pressure is halved? 17. The blood-brain barrier is formed by:
A. Increase 8 times A. tight junctions between vascular endothelial
B. Increase 16 times cells in the cerebral capillaries
C. Increase 4 times B. choroidal epithelial cells
D. Decreases 4 times C. ependymal cells
D. foot processes of oligodendroglia
11. Recombinant brain natriuretic peptide
(nesiritide) is currently used in the management Answers to Self-Scorer 6 - Cardiovascular
of: Physiology
A. bronchial asthma 1A 2A 3A 4C 5A
B. congestive heart failure 6C 7B 8AB 9B 10A
C. bilateral renal artery stenosis 11B 12C 13C 14B 15ABC
D. ACE inhibitor induced cough 16D 17A

12. Maximal oxygen consumption (VO2 max) in Answer Explanations:


healthy active men is about 1. See p 548, Ch 28, WFG, 2005.
A. 10 ml/kg/min
B. 20 ml/kg/min 2. Normally, it is the impulse that originates in the
C. 40 ml/kg/min SA node that excites the ventricular myocardium.
D. 80 ml/kg/min Such a rhythm is called sinus rhythm.

13. The ‘last ditch stand’ in defense of a falling 3. There are two regions in the heart where the
blood pressure is the: inhibitory effect of vagal stimulation on
A. arterial baroreflex mechanism conduction speed is profound. These are the SA
B. arterial chemoreflex mechanism and AV nodes. The conduction speed in the SA
C. CNS ischemic pressor response and AV nodes is about 0.05 m/s. See p. 549, Ch
D. Bainbridge reflex 28, WFG, 2005. Compared to the fast conducting
Purkinje system, this is about 80 times slower.
14. Which of the following does not occur when The conduction speed is slowed even further in
the body is immersed in water up to the neck? disease conditions such as SA node exit block and
A. Increase in central venous pressure AV nodal block.
B. Increase in plasma level of renin
4-5. Depolarization and repolarization are
15. Which of the following statements is correct? electrically opposite processes. Normally,
A. Pulse pressure is directly proportional to ventricular depolarization occurs from
stroke volume endocardium to epicardium, and repolarization
B. Pulse pressure is inversely proportional to occurs from epicardium to endocardium. This is
compliance of large arteries why the T wave is upright, i.e. in the same
C. Reflected arterial pulse waves normally serve direction as QRS. An inverted T wave indicates
to increase coronary perfusion during diastole that the direction of ventricular repolarization is
reversed, i.e. occuring from endocardium to
16. Which of the following is most likely to epicardium.
happen in a normal healthy adult seated in a
centrifuge and spun at a force equal to 2 times that 6. PA interval (approximately 30 ms) reflects the
of gravity? time taken for conduction across the atrial
A. Increase in intracranial tension pathways to the AV node. PR segment is a
B. Increase in BP combined index of AV nodal delay and
C. Increase in cardiopulmonary blood volume conduction in infranodal pathways. PR interval is
111
E.S.Prakash. MCQs in Medical Physiology, May 2013
a rather poor index of AV nodal delay. AH of a falling BP), contributes to restoring cerebral
interval, which is obtained by a His bundle blood flow.
electrogram specifically reflects AV nodal delay.
See p. 553-554, Ch 28, WFG, 2005. Reflex mechanism Mean arterial
pressure (mm Hg)
range in which it
7. The long duration of action potential (or the
operates
absolute refractory period) in the heart is due to Arterial baroreflex 70–110
slow repolarization. This is a safety features that Arterial chemoreflex 40–70
ensures that the ventricles relax to fill with blood CNS ischemic < 50
before contracting again. response

10. Tissue blood flow = BP / local vascular 14. When a person is immersed in water up to the
resistance. Blood flow is directly proportional to neck, this functions as an anti-G suit (anti G suits
the fourth power of radius. are filled with water or compressed air) – thereby
mitigating the effect of gravity. Thus, venous
11. ANP is a natriuretic hormone. It increases return and central blood volume increase. A rise
blood flow through the kidneys and glomerular in CVP due to an increase in central blood volume
filtration rate. ANP levels are increased in heart is accompanied by a reflex decrease in renin
failure. The action of ANP in heart failure is to release from the kidneys as long as arterial blood
increase Na and water excretion by the kidneys. pressure, renal blood flow and GFR are normal.
At first sight, this would appear to be a beneficial
effect; however, it must be noted that in heart 15. The stiffer the arteries, i.e., the less distensible
failure, effective arterial blood volume is they are, higher the pulse pressure for a given
diminished. Angiotensin II, aldosterone and the stroke volume. This is the basis of higher systolic
renal vasoconstrictive effects of elevated activity pressure in isolated systolic hypertension, the
in the renal sympathetic nerves counteract the commonest type of hypertension in the elderly.
natriuretic effects of ANP.
16. The effect of positive g forces, in an
12. VO2 max is the maximum amount of oxygen individual standing upright, would be to throw
that can be utilized during dynamic exercise; it blood into the lower part of the body.
can be increased by training. See p 635, Ch 33,
WFG, 2005. 17. While foot processes of astrocytes end on
cerebral capillaries and induce the formation of
13. Arterial BP is affected by numerous neural tight junctions, the actual anatomic basis of the
and humoral mechanisms. The arterial baroreflex blood brain barrier is the tight junctions between
mechanism buffers BP fluctuations when mean endothelial cells in cerebral capillaries. In the
arterial pressure (MAP) is in the 70-150 mm Hg circumventricular organs such as area postrema,
range. But when MAP is lower than 70 mm Hg, subfornical organ, this barrier is absent.
baroreceptors are maximally deactivated and the
arterial chemoreflex mechanism assumes greater ***************************************
importance in defending against a further fall in Self-Scorer 7: Pulmonary Physiology
BP.
Choose the single best answer
In severe hypotension such as when mean arterial
pressure is < 50 mm Hg), ischemia of the 1. The major area of airway resistance during
vasomotor center in the medulla triggers a breathing is located in:
powerful increase in sympathetic outflow and A. oropharynx
MAP. This mechanism, the CNS ischemic pressor B. trachea and large bronchi
response (called the “last ditch stand” in defense C. intermediate-sized bronchi
D. bronchioles < 2 mm in diameter
112
E.S.Prakash. MCQs in Medical Physiology, May 2013
9. Deep sea divers have been known to offer their
2. Pulmonary capillary wedge pressure must be mouths to fish! This is because:
estimated under: A. of low fat solubility of nitrogen
A. zone 1 conditions B. of nitrogen toxicity
B. zone 2 conditions C. helium impairs intellectual functions
C. zone 3 conditions D. of increased work of breathing
D. zone 4 conditions
10. Which of the following is used during deep
3. Cardiogenic pulmonary edema occurs when sea diving?
pulmonary capillary pressure exceeds: A. Compressed room air
A. 10 mm Hg B. 100% oxygen
B. 15 mm Hg C. A helium–oxygen mixture
C. 20 mm Hg D. 50% N2 and 50% oxygen
D. 25 mm Hg
11. In which of the following situations does
4. Apneusis occurs typically after: decompression sickness not occur?
A. damage to phrenic motor neurons A. During ascent in an unpressurized cabin of an
B. damage to pneumotaxic center airplane
C. lesions of pneumotaxic center and vagotomy B. At a depth of 100 ft in the ocean
D. denervation of peripheral chemoreceptors C. During rapid ascent from a deep-sea dive

5. Which one of the following has been recently 12. What is the treatment of choice for air
postulated to function as a sensor of oxygen levels embolism?
and a facilitator of oxygen transport in the brain? A. Continuous positive airway pressure
A. Myoglobin B. Gradual decompression
B. Neuroglobin C. Hyperbaric oxygenation
C. Nitric oxide D. Positive pressure ventilation
D. Cytochrome oxidase c
13. Which of the following combination of arterial
6. Life is impossible without pressurization at an blood gas results is most likely in a normal person
altitude greater than: after a month’s residence at 4000-meter altitude?
A. 12000 m Arterial pH, PaCO2 and HCO3 (mmol/L)
B. 14000 m respectively would be:
C. 17000 m A. 7.4. 40 mmHg, 24
D. 19000 m B. 7.36, 36 mm Hg, 30
C. 7.6, 20 mm Hg, 40
7. The altitude at which body fluids boil is about: D. 7.46, 26 mm Hg, 19
A. 8800 m
B. 12400 m 14. The amount of oxygen dissolved in plasma
C. 14500 m relates most closely with:
D. 19000 m A. FiO2
B. PAO2
8. Which of the following occurs as a C. PaO2
consequence of increased barometric pressure? D. Blood [Hb]
A. Rapture of the deep
B. Black out Answers: Self-Scorer 7 - Pulmonary Physiology
C. Bends 1C 2C 3D 4C 5B
D. Air embolism 6B 7D 8A 9B 10C
11B 12C 13D 14C

113
E.S.Prakash. MCQs in Medical Physiology, May 2013
Answer Explanations: this as ‘compression sickness’. In contrast,
1. This is because of the high airflow velocity in decompression sickness (the exact opposite of
these segments. Bronchioles less than 2 mm in compression sickness) occurs during rapid ascent
diameter represent less than 10% of total airway following a deep-sea dive or when an airplane
resistance. See p 282, NMS Physiology, Bullock J cabin loses pressure at high altitude. Gases
et al, 2001. dissolved in body fluids come out of solution
because of the reduction in ambient pressure,
3. Normally, pulmonary capillary hydrostatic forming bubbles and blocking blood flow to
pressure is about 10 mm Hg. It is considered tissues.
elevated when PCWP exceeds 18 mm Hg. The
osmotic pressure of plasma proteins is normally 14. By applying high pressure, solubility of gas
about 25 mmHg. (air emboli in this question) in plasma is
increased. At a constant temperature, solubility of
4. The effect of afferent vagal input from stretch a gas in plasma is directly proportional to partial
receptors in the lungs is to inhibit inspiration. The pressure of gas (Henry’s law).
pneumotaxic center in the pons serves to ‘switch’
from inspiration to expiration. Thus, after ***************************************
vagotomy alone, the depth of respiration would be Self-Scorer 8: Renal and Acid-Base
higher than normal. However, after damage to Physiology
both pneumotaxic center as well as vagotomy,
prolonged inspiratory spasms that resemble breath 1. Which of the following substances cannot be
holding (apneusis) occur. See p 672, Ch 36, WFG, used for measuring GFR?
2005. A. Glucose
B. Iothalamate
6. At an altitude of 14000 meters, consciousness is C. Inulin
lost despite administration of 100% oxygen. This
is because the barometric pressure at this altitude 2. If the glomerular capillary hydrostatic pressure,
is far too low to permit adequate oxygenation of osmotic pressure of plasma proteins, hydrostatic
arterial blood. See p. 689, Ch 37, WFG, 2005. pressure in the Bowman’s space and oncotic
pressure in the interstitium are respectively 40, 25,
9. The euphoria is attributed to increased 5, and 0 mm Hg respectively, what is the net
solubility of nitrogen in body fluids. This is why a pressure driving filtration of fluid into the
helium-oxygen mixture is used and while Bowman’s space?
increased solubility of helium is associated with A. 10 mm Hg
impairment of manual dexterity, intellectual B. 15 mm Hg
function is apparently not significantly impaired. C. 20 mm Hg
See p. 694-5, Ch 37, WFG, 2005. D. 25 mm Hg

10. If a diver breathes compressed air, the 3. In a normal adult human on an average diet,
increased partial pressure of nitrogen can result in and with an arterial plasma pH of 7.37, which of
a constellation of neurological symptoms known the following is least likely to be present in urine
as “rapture of the deep”. See p 694-5, Ch 37, obtained first thing in the morning?
WFG, 2005. A. Calcium
B. Urea
11. At a depth of 100 ft in the ocean, barometric C. Uric acid
pressure is about 4 atmospheres, four times D. HCO3
greater than at mean sea level. The problems that
occur are therefore due to increased solubility of 4. Normally the ratio of PaCO2 and plasma HCO3
gases in blood and depend on the gas mixture is:
used for breathing. You may want to remember A. 1.2
114
E.S.Prakash. MCQs in Medical Physiology, May 2013
B. 1.4 pressure gradient for filtration across the
C. 1.6 glomeruli is (40 + 0) – (25 + 5) = 10 mm Hg.
D. 1.8
4. Normally, PaCO2 = 40 mm Hg, and plasma
+
5. If PaCO2 is 80 mm Hg and arterial plasma [H ] HCO3 = 24 mM and PaCO2 / [HCO3] = 1.6. If this
is 80 nmol/L, then plasma [HCO3] is: were the case, then the [H+] of arterial plasma
A. 40 mmol/L would be equal to 40 nM (corresponding to an
B. 28 mmol/L arterial pH of 7.4) as predicted by the modified
C. 24 mmol/L Henderson equation.
D. 10 mmol/L
5,6. Using the modified Henderson equation, we
6. In the above situation (Question 5), what is the get, plasma HCO3 = 24 mM. This is
acid-base status? uncompensated respiratory acidosis because there
A. Compensated metabolic alkalosis is no compensatory increase in bicarbonate levels.
B. Uncompensated respiratory acidosis
C. Compensated respiratory acidosis 7. Plasma pH and total CO2 concentration are
D. Uncompensated metabolic acidosis measured, and plasma [HCO3] is calculated using
E. Normal acid-base status the Henderson-Hasselbalch equation.

7. In arterial blood gas analysis, which one of the 8. Base excess = Observed buffer base – normal
following is calculated? buffer base. Normal buffer base is approximately
A. Arterial pH 48 mM. If observed buffer base is 52 mM, then
B. Plasma bicarbonate base excess if 4 mM. Base excess is present in
C. PaCO2 metabolic alkalosis. The calculation of base
D. PaO2 excess does not offer any advantage over
assessment of serum bicarbonate if blood Hb and
8. Which of the following is not elevated in plasma protein concentrations are constant.
ethylene glycol poisoning?
A. Anion gap Serum osmolality is elevated in ethylene glycol
B. Base excess poisoning because of an increase in the serum
C. Osmolar gap level of oxalate; thus, the difference between
D. Serum osmolality measured and calculated osmolality (osmolar gap)
increases.
9. The severity of acidosis is related to:
A. anion gap The elevation of anion gap in this condition is
B. pH of arterial plasma primarily because of an increase in serum oxalate.
C. pH of ICF and plasma
D. plasma HCO3 9. It is important to understand that it is possible
that there might be an acid base disturbance
Answers: although the pH of arterial plasma may be 7.4
1A 2A 3D 4C 5C (‘normal’). For example, there might be
6B 7B 8B 9C intracellular acidosis before arterial pH drops. See
Balakrishnan et al. What is the ultimate goal in
Answer Explanations: acid-base regulation? Advances in Physiology
1. Glucose is normally not excreted by the Education.
nephron. So it cannot be used to measure GFR.
*************************************
2. Recall Starling’s equation describing fluid
movement across capillaries. In this instance, the

115
E.S.Prakash. MCQs in Medical Physiology, May 2013
A Blast from the Past D. Cl

I believe some of these questions appeared in past 6. Amongst plasma proteins, albumin makes the
PG entrance exams in some form but I have made greatest contribution to the colloid osmotic
several editorial changes to the stem and the pressure of plasma proteins because relative to
answer choices in many of them. Needless to say, most plasma proteins albumin:
it is good to use these questions to guide your A. is the most abundant plasma protein
preparation for future exams. B. has the longest half-life
C. has the lowest molar mass
Choose the single best answer D. exits the capillary fastest

1. A substance injected intravenously was found 7. The osmotic pressure of crystalloids is not
to be distributed through 30% of total body water included in the Starling’s equation describing
(TBW). It probably: fluid exchange across capillaries because:
A. did not pass through blood capillaries A. they were discovered after Starling described
B. was distributed evenly throughout body water the forces
C. did not enter cells B. crystalloids carry electric charges
D. was excluded from CSF C. crystalloids diffuse freely across the capillary
endothelium
2. What fraction of total body potassium is present D. the quantity of crystalloids in plasma is
in plasma? negligible compared to plasma proteins
A. 0.4%
B. 7.6% 8. A pole vaulter fell during pole vaulting and
C. 10.4% paralyzed his arm. Which of the following
D. 89.5% investigations is most useful to estimate likelihood
of recovery?
3. A solution contains 1 gram mole of magnesium A. Electromyography
sulfate per liter. Assuming full ionization, B. Muscle biopsy
calculate the osmotic pressure of the solution. (1 C. Strength-duration curve
mOsm/L exerts an osmotic pressure of 19.3 mm D. CPK levels
Hg)
A. 19.3 mm Hg 9. Synaptic conduction is mostly orthodromic
B. 3.86 mm Hg because:
C. 19300 mm Hg A. dendrites cannot be depolarized
D. 38600 mm Hg B. once repolarized, an area cannot be
E. 57900 mm Hg depolarized.
C. the strength of antidromic impulses is less.
4. If potassium chloride concentration of a D. chemical mediator is located only in the
solution is 39 mg %, then, what is the millimolar presynaptic terminal.
concentration of potassium in this solution?
A. 1 mM 10. The main excitatory neurotransmitter in the
B. 3.9 mM CNS is:
C. 10 mM A. glycine
D. 100 mM B. acetylcholine
C. aspartate
5. Emiocytosis requires an increase in the D. glutamate
intracellular concentration of:
A. Na 11. A 10ºC decrease in body temperature
B. K decreases cerebral metabolic rate (i.e., cerebral
C. Ca consumption of oxygen) by:
116
E.S.Prakash. MCQs in Medical Physiology, May 2013
A. 10% 18. Which combination of the following
B. 30% statements are correct regarding hypoxia?
C. 50% (choose all correct answers).
D. 70% A. When it is severe, causes stimulation of the
sympathetic nervous system
12. In spinal anesthesia, which of the following is B. It leads to the accumulation of hydrogen and
lost first? lactate ions in tissues and the circulation
A. Sympathetic tone C. It causes a decrease in cerebral blood flow
B. Sensation in the lower limbs D. Chronic hypoxia is typically associated with a
C. Motor function rightward shift of oxyhemoglobin dissociation
D. Proprioception curve

13. Feedforward inhibition occurs in the: 19. An anterolateral cordotomy relieving pain in
A. basal ganglia the right leg is effective because it interrupts the:
B. thalamus A. left dorsal column
C. cerebellum B. left ventral spinothalamic tract
D. cerebral cortex C. left lateral spinothalamic tract
D. right lateral spinothalamic tract
14. Which of the following changes occur in rod
cells when rhodopsin is activated by light? 20. A lesion of the ventrolateral part of the spinal
A. Increase in cGMP cord is likely to lead to loss (below the level of the
B. Deactivation of phosphodiesterase lesion) of:
C. Depolarization of rod cells A. pain sensation on the ipsilateral side
D. Decreased release of neurotransmitter B. proprioception on the contralateral side
C. pain sensation on the contralateral side
15. During the dark phase of the visual cycle, D. proprioception on the ipsilateral side
which form of vitamin A combines with opsin to
make rhodopsin? 21. Which of the following states does a
A. All trans retinaldehyde predominance of beta waveforms in an
B. All trans retinol electroencephalogram suggest?
C. 11-cis retinaldehyde A. Deep anesthesia
D. 11-cis retinol B. Surgical anesthesia
C. Light anesthesia, eyes closed, relaxed
16. In the visual system, the term blobs refers to: D. Awake, alert state
A. parvocellular pathway from the lateral
geniculate nucleus 22. Which of the following is least likely to
B. clusters of axons of ganglion cells in the retina aggravate insult in an injured brain?
C. clusters of amacrine cells in the retina A. Hypercapnia
D. clusters of cells in layers 2 and 3 of the visual B. Hypoxia
cortex that have high levels of cytochrome C. Hypotension
oxidase D. Hypothermia

17. Which of the following phenomena is closely 23. Cushing’s triad does not include:
associated with slow wave sleep? A. hypertension
A. Dreaming B. bradycardia
B. Atonia C. hypothermia
C. Sleep walking D. irregular respiration
D. Irregular heart rate
24. Cell bodies of orexigenic neurons are present
in:
117
E.S.Prakash. MCQs in Medical Physiology, May 2013
A. locus ceruleus
B. dorsal raphe nucleus 30. Adrenaline, noradrenaline and dopamine act
C. lateral hypothalamic area upon membrane receptors that span the
D. hippocampus membrane:
A. 2 times
25. Exposure to darkness leads to increased B. 5 times
melatonin secretion. It is brought about by: C. 7 times
A. decreasing the activity of suprachiasmatic D. 10 times
nucleus
B. increasing activity of serotonin N-acyl 31. ADH circulates in plasma primarily
transferase A. bound to neurophysin I
C. decreasing the hydroxy-indole-O-methyl B. bound to neurophysin II
transferase activity. C. bound to plasma albumin
D. blocking the release of norepinephrine from D. not bound to plasma proteins
sympathetic nerve terminals.
32. The following values were obtained from a
26. Stimulation of sympathetic nerves to the patient diagnosed to have diabetic ketoacidosis:
pineal gland: PaO2 = 90 mm Hg, PaCO2 = 30 mm Hg, plasma
A. increases synthesis of melanin [HCO3] = 10 mM, the pH of CSF in this patient
B. increases activity of serotonin N-acetyl would be about:
transferase A. 7.2
C. reduces melatonin synthesis B. 7.25
D. increases release of serotonin C. 7.33
D. 7.40
27. Which of the following statements regarding
the thermic effect of feeding is incorrect? 33. A 10 days old neonate is posted for pyloric
A. It is most prominent at the time of eating stenosis surgery. Serum calcium is 6 mg/dL
B. It is synonymous with specific dynamic action (normally 8.5 – 10.5 mg/dL). What information
of food would you like to have before you supplement
C. It is greater following a protein meal calcium to this neonate?
D. It is enhanced by sympathetic neural activity A. Blood glucose
B. Serum albumin
28. The first physiologic response to high C. Serum bilirubin
environmental temperature is: D. Oxygen saturation
A. sweating
B. cutaneous vasodilation 34. The positive feedback effect of estrogen in
C. decreased heat production inducing LH surge is associated with one of the
D. nonshivering thermogenesis following steroid hormone ratios in the peripheral
circulation:
29. A 38-year old gentleman reports decreased A. High estrogen: low progesterone
hearing in the right ear for the last 2 years. On B. Low estrogen: high progesterone
testing with a 512 Hz tuning fork, the Rinne’s test C. Low estrogen: low progesterone
(without masking) is negative on the right ear and D. High estrogen: high progesterone
positive on the left side. With the Weber’s test,
the tone is perceived as louder in the left ear. The 35. The major trophic hormone for Sertoli cells
most likely diagnosis is: and the major stimulator of aromatase activity of
A. conduction loss in the right ear Sertoli cells is
B. sensorineural hearing loss in the right ear A. inhibin
C. sensorineural hearing loss in the left ear B. LH
D. conduction loss in the left ear C. FSH
118
E.S.Prakash. MCQs in Medical Physiology, May 2013
D. Melatonin A. the involution of corpus luteum causes
estradiol and progesterone levels to fall
36. In a young female of reproductive age with dramatically.
regular menstrual cycles of 28 days, ovulation B. LH levels rise after ovulation.
occurs around the 14th day. When is the first polar C. estradiol levels are not involved in the LH
body extruded? surge phenomenon.
A. 24 h prior to ovulation D. estradiol inhibits the induction of the
B. At the time of ovulation progesterone receptor in the endometrium.
C. 48 h after the ovulation
D. At the time of fertilization 42. The conjugation of bilirubin with glucuronic
acid in the liver:
37. A baby girl presents with bilateral inguinal A. converts a hydrophilic compound to a
masses, thought to be hernias, but these were hydrophobic molecule
found to be testes in the inguinal canals. Which B. converts a hydrophobic molecule to a
karyotype would you expect to find in the child? hydrophilic molecule
A. 46 XX C. enables bilirubin to cross the cell membrane
B. 46 XY D. is increased during neonatal jaundice
C. 47 XXY
D. 47 XYY 43. Removal of the liver is fatal because:
A. blood urea rises
38. The enzyme needed for the conversion of B. jaundice develops
androstenedione to estrone in the granulosa cells C. clotting time is prolonged
of the ovarian follicle in the follicular phase of the D. progressive hypoglycemia occurs
ovarian cycle is:
A. cholesterol desmolase 44. CD-95 has a major role in:
B. CYP11B1 A. apoptosis
C. aromatase (CYP 19) B. cell necrosis
D. 17-beta hydroxysteroid dehydrogenase C. interferon activation
D. proteolysis
39. The biologically most active androgen is:
A. testosterone 45. Erythropoiesis is inhibited by:
B. dehydropepiandrosterone A. ACTH
C. androstenedione B. thyroxine
D. dihydrotestosterone C. estrogen
D. prolactin
40. The laboratory report shows values of
gonadotropins and ovarian hormones of the blood 46. Vagal stimulation (considering all viscera) is
sample taken on the 20th day of the menstrual least likely to:
cycle of a young woman. Whether her cycle was A. increase intestinal secretion
ovulatory or not may be validly assessed by B. contract smooth muscle of intestine
measuring serum levels of: C. cause bronchodilation
A. FSH D. cause an increase in heart rate
B. LH
C. estradiol 47. Saccades are
D. progesterone A. voluntary, slow eye movements
B. involuntary, slow eye movement
41. Apoptosis occurs as a result of changes in C. abrupt, involuntary, slow eye movements
hormone levels during the ovarian cycle. When D. abrupt, voluntary, rapid eye movements
the ovum is not fertilized, endometrial cells die
because:
119
E.S.Prakash. MCQs in Medical Physiology, May 2013
48. In an Rh-negative mother not previously 10.399780

sensitized by the Rh antigen, Rh incompatibility X beat


5.199890
does not usually have a serious consequence

Volts
ECG
during the first pregnancy because: 0.000000

A. antibodies are not able to cross placenta


B. the titer of IgG is low during the primary -5.199890

immune response 128.00000 130.00000

C. IgG is ineffective against fetal red cells seconds

D. massive hemolysis in the fetus is compensated A. X beat originated from an atrial focus
by increased erythropoiesis B. X beat reset the cardiac rhythm
C. Both heart sounds would have been present at
49. Although more than 400 blood groups have X beat
been identified, ABO blood group system is one D. The path of spread of excitation was normal.
of the most important in clinical medicine
because: 53. A dye ABC has been in use for the
A. it was the first blood group system to be measurement of blood volume and cardiac output.
discovered. This was rivaled by the introduction of XYZ,
B. it has four different groups viz A, B, AB and which crossed the capillaries. What difference
O would the use of XYZ make in this context?
C. ABO antigens are present in most body tissues A. No change in measured cardiac output &
and fluids. blood volume
D. ABO antibodies are invariably present in B. Increase in measured blood volume alone
plasma when the person’s RBC lacks the C. Increase in measured cardiac output alone
corresponding antigen. D. Cause an error in the measurement of both
50. A 55-year-old male accident victim in the ED 54. The most recent technique for noninvasive
urgently requires a transfusion. His blood group measurement of cardiac output is:
could not be determined as his red cell group and A. pulmonary artery catheterization
plasma group did not match. Emergency B. thermodilution
transfusion should be done with: C. echocardiography
A. RBC corresponding to his red cell group and D. impedance cardiography
colloids and crystalloids
B. Whole blood corresponding to his plasma 55. A cardiologist asked his lab technician to
group. determine the systolic time intervals of a 60 yr old
C. O positive RBC, colloids and crystalloids patient with a recent anterior wall infarction. The
D. AB negative blood technician said that the pulse transducer was not
working. Which of the following could he then
51. Stimulation of sympathetic nerves to the heart have not determined?
decreases: A. QS2
A. heart rate B. Left ventricular ejection time (LVET)
B. force of cardiac contraction C. Pre-ejection period (PEP)
C. speed of conduction D. Both LVET and PEP
D. refractory period
56. Loss of 500 ml of blood over 30 minutes will
52. Which of the following statements represent lead to:
the most appropriate interpretation of the ECG A. an increase in HR, fall in BP
shown below? B. a fall in BP and HR
C. a prominent increase in HR and BP
D. a slight increase in HR

120
E.S.Prakash. MCQs in Medical Physiology, May 2013
57. Two students, AB and CD were asked to
demonstrate in dogs the role of sinus nerve in 61. Patients with acute cardiac failure may not
hypovolemic shock. AB severed the sinus nerve have edema if:
when the mean blood pressure was 85 mm Hg and A. oncotic pressure of plasma proteins is high
CD cut the sinus nerve when the mean arterial B. renal compensation occurs
pressure (MAP) was 60 mm Hg. On cutting the C. cardiac output is decreased
sinus nerve: D. there is a fall in systemic capillary hydrostatic
A. AB recorded an ↑ in MAP but CD recorded a pressure

B. AB recorded a ↓ in MAP but CD recorded an 62. In which of the following organs is the flow
↑ least under sympathetic control?
C. both recorded an ↑ in MAP A. Brain
D. both recorded a ↓ in MAP B. Heart
C. Viscera
58. As part of a space-research program, a D. Skin
physiologist was asked to investigate the effect of
flight-induced stress on blood pressure. 63. Statement: Blood is a non-Newtonian fluid.
Accordingly, the blood pressures of the Reason: Its viscosity is constant at any flow
cosmonauts were to be measured twice: once velocity.
before take-off and once after the spacecraft A. Both (S) and (R) are false
entered the designated orbit around the earth. For B. (S) is true and (R) is false
a proper comparison, the pre-flight blood pressure C. Both (S) and (R) are true but (R) does not
should be recorded in (the): explain (S)
A. lying down position D. Both (S) and (R) are true and (R) explains (S)
B. sitting position
C. standing position 64. While introducing the Swan Ganz catheter, its
D. any position as long as the post-flight placement in the pulmonary artery (PA) is best
recording is made in the same position. inferred from which of the following?
A. Diastolic pressure is lower in PA than in right
59. During exercise, an increase in O2 uptake by ventricle
exercising muscles does not occur due to: B. Diastolic pressure is higher in PA than right
A. shift of the oxyhemoglobin dissociation curve ventricle
to the left C. PA pressure tracing has dicrotic notch
B. decrease in vascular resistance in exercising D. Right ventricular pressure tracing for plateau
skeletal muscle and sharp drop in early diastole.
C. ↑ O2 extraction
D. ↑ blood flow 65. Which of the following is usually associated
with turbulence in blood flow?
60. In hemorrhaged dogs with marked A. Reynolds number less than 2000
hypotension (mean arterial pressure < 50 mm Hg), B. Decrease in blood flow velocity
denervation of arterial chemoreceptors would: C. Decrease in density of blood
A. increase BP since chemoreceptors reduce D. Increase in diameter of blood vessel
sympathetic outflow
B. produce no change in BP since 66. Filtration at the arterial end of capillary occurs
chemoreceptors do not influence sympathetic mainly due to:
outflow A. hydrostatic pressure in capillaries
C. result in a further fall in BP since the arterial B. hydrostatic pressure in interstitium
chemoreflex is sympathoexcitatory C. oncotic pressure in capillaries
D. depend on whether arterial baroreceptors are D. oncotic pressure in interstitium
reset or not
121
E.S.Prakash. MCQs in Medical Physiology, May 2013
67. Intrapleural pressure is normally negative B. Hering-Breuer deflation reflex
because: C. Head’s paradoxical reflex
A. intrapulmonary pressure is always negative D. J-reflex
B. chest wall and lungs recoil in opposite
directions at functional residual capacity 74. One intern calculated the concentration of O2
C. surfactant prevents lung collapse in blood as 0.0025 ml/ml of blood. Considering
D. transpulmonary pressure determines the atmospheric pressure as 760 mm Hg, what is the
negativity approximate oxygen tension of arterial blood?
A. 40 mm Hg
68. Which of the following is incorrect about B. 60 mm Hg
fetal hemoglobin (Hb F)? C. 80 mm Hg
A. In comparison to HbA, Hb F has greater D. 100 mm Hg
affinity for 2,3–BPG
B. Its ODC is shifted to left relative to HbA 75. A traveling nerve impulse does not depolarize
C. At low PO2, Hb F gives up more oxygen to the area immediately behind it because:
tissues than Hb A. A. it is hyperpolarized
B. the area immediately behind is refractory
69. The normal value of P50 on the C. it is not self-propagating
oxyhemoglobin dissociation curve in an adult is: D. the conduction is always orthodromic
A. 1.8 kPa
B. 2.7 kPa 76. The renal plasma flow (RPF) of a patient was
C. 3.6 kPa to be estimated through the measurement of para-
D. 4.5 kPa amino hippuric acid (PAH) clearance. The
technician observed the procedures correctly but
70. Which of the following conditions leads to due to an error in weighing inadvertently used
tissue hypoxia without an alteration of oxygen thrice the recommended dose of PAH. The RPF
content of blood? estimated thus is likely to be:
A. Carbon monoxide poisoning A. falsely high
B. Methemoglobinemia B. falsely low
C. Cyanide poisoning C. high or low depending on GFR
D. Respiratory acidosis D. correct and is unaffected by the overdose of
PAH
71. The arterial blood gas values, pH 7.58, PCO2
23 mm Hg, PO2 300 mm Hg and oxygen 77. Several hormones regulate tubular
saturation of hemoglobin 60% are most consistent reabsorption of water and electrolytes at different
with a diagnosis of: sites in the nephron. Which of the following
A. carbon monoxide poisoning combinations is/are correct?
B. ventilatory (type 2 respiratory) failure A. Angiotensin II acts in the DCT
C. voluntary hyperventilation B. Aldosterone acts in the collecting ducts
D. methyl alcohol poisoning C. ADH acts in the PCT
D. ANP acts in the loop of Henle
72. Intense stimulation of J receptors results in:
A. tachypnea 78. A modified neuroepithelial cell is not the
B. hypoxemia sensory receptor in the:
C. pulmonary edema A. visual pathway
D. apnea followed by tachypnea B. auditory pathway
C. gustatory pathway
73. If inflation of the lungs induces further D. olfactory pathway
inflation, this is the:
A. Hering-Breuer inflation reflex
122
E.S.Prakash. MCQs in Medical Physiology, May 2013
79. The hyperkinetic features of the Huntington's
disease are due to the loss of neurons in the: 86. The mitotic spindle is made up of a protein
A. nigrostriatal dopaminergic system called:
B. intrastriatal cholinergic system A. tubulin
C. GABAergic and dopaminergic system B. caveolin
D. intrastriatal GABAergic and cholinergic C. connexin
system D. annexin

80. Epiphysial closure in humans is caused 87. When a heavy object in hand is lowered, the
(mediated) by: extension at the elbow is brought about by:
A. androgens A. active shortening of the extensors
B. estrogens B. passive shortening of the extensors
C. growth hormone C. active lengthening of the flexors
D. thyroxine D. active shortening of the flexors

81. If a genetically male fetus with functional 88. Rod cell excitation by photons results in the
testes is born with male internal genitalia and conversion of:
female external genitalia, which of the following A. 11-cis-retinal to 11-trans-retinal
is the most likely explanation? B. 11-cis-retinal to all trans-retinal
A. Loss of function mutation in SRY gene C. all trans retinal to all cis-retinal
B. Deficiency of testosterone D. 11-trans-retinal to 11-cis-retinal
C. Deficiency of Mullerian inhibiting peptide
D. Deficiency of testosterone 5α reductase 89. Which of the following is characterized by a
E. Complete androgen insensitivity hypercontractile, shrunken bladder, increased
frequency of urination and incontinence?
82. Of those mentioned below, which of the A. Deafferented bladder
following is least likely in a normal pregnancy? B. Denervated bladder
A. Increase in heart rate C. Spastic neurogenic bladder
B. Increase in cardiac output D. Bladder in acute paraparesis
C. Increase in blood volume
D. Decrease in pulse pressure 90. The smooth muscle relaxing effects of
endothelium derived relaxing factor nitric oxide
83. The SI unit of pressure is: are mediated by an increase in intracellular levels
A. mm Hg of
B. cm H2O A. cAMP
C. Pascal B. cGMP
D. Torr C. calcium
D. endothelin
84. Which method is dependable for measuring
cardiac output when cardiac output is low? 91. Calculate the osmolality of plasma given that
A. Fick principle plasma glucose is 108 mg/dL, plasma [Na] is 125
B. Thermodilution method mM, and blood urea nitrogen is 140 mg/dL.
C. Indicator dilution method A. 300 mOsm/Kg H2O
B. 306 mOsm/Kg H2O
85. Breathing in individuals with restrictive lung C. 312 mOsm/Kg H2O
disease is typically: D. 318 mOsm/Kg H2O
A. slow and deep
B. slow and shallow 92. Which of the following markers is incorrectly
C. rapid and deep matched with its volume of distribution?
D. rapid and shallow A. Antipyrine - total body water
123
E.S.Prakash. MCQs in Medical Physiology, May 2013
B. Inulin - ECF volume 99. The concentration of sodium in normal human
C. Evans blue - plasma volume plasma varies from
D. I125 albumin - blood volume A. 280 – 295 mOsm/Kg H2O
B. 135 – 145 mOsm/Kg H2O
93. Which of the following processes does not C. 240 – 255 mOsm/Kg H2O
exhibit 'saturation kinetics'? D. 95 – 110 mOsm/Kg H2O
A. Facilitated diffusion
B. Na+-Ca2+ exchanger 100. Arterial blood gas analysis of a 5-year-old
C. Simple diffusion child at sea level gives the following results: pH
D. Na+ coupled active transport 7.41, PaO2 100 mmHg, and PaCO2 40mm Hg. The
child is being mechanically ventilated with 80%
94. Insulin dependent glucose uptake into skeletal oxygen. What is the (A-a) PO2?
muscle and adipose tissue is mainly mediated by: A. 420 mm Hg
A. GLUT 1 B. 470 mm Hg
B. GLUT 2 C. 520 mm Hg
C. GLUT 3 D. 570 mm Hg
D. GLUT 4
101. Physiologic dead space ventilation is
95. Insulin secretion from beta cells of pancreas in decreased by:
response to a glucose load is mediated by: A. upright position
A. GLUT 1 B. positive pressure ventilation
B. GLUT 2 C. neck flexion
C. GLUT 3 D. emphysema
D. GLUT 4
102. What does the macula densa sense?
96. Which of the following is not true about A. Na concentration of fluid delivered to it
cerebrospinal fluid? B. Cl concentration of the fluid delivered to it
A. Its pH is less than that of plasma C. Volume of fluid delivered to it
B. It is formed in arachnoid villi
C. It normally does not contain neutrophils Answers: Blast from the Past
D. Leakage of CSF during dural tap causes 1C 2A 3D 4C 5C
headache 6A 7C 8A 9D 10D
11D 12A 13C 14D 15C
97. Which of the following mechanisms is 16D 17C 18ABD 19C 20C
currently implicated in learning and memory? 21D 22D 23C 24C 25B
A. Modulation of release of neurotransmitters in 26B 27A 28B 29B 30C
postsynaptic neurons in response to repeated 31D 32D 33B 34A 35C
firing of presynaptic neurons 36B 37B 38C 39D 40D
B. Modulation of neurotransmitter receptor 41A 42B 43C 44A 45C
synthesis in postsynaptic neurons 46C 47D 48B 49D 50C
C. Formation of new neurons 51D 52C 53D 54D 55D
D. Spatial organization of association areas 56D 57A 58A 59A 60C
61D 62A 63B 64C 65D
98. The first reflex response to reappear during 66A 67B 68A 69C 70C
recovery from spinal shock is the 71A 72D 73C 74C 75B
A. tympanic reflex
76B 77AB 78D 79D 80B
B. withdrawal reflex
81D 82D 83C 84A 85D
C. neck righting reflex
86A 87C 88B 89C 90B
D. labyrinthine reflex
91B 92D 93C 94D 95B
96B 97all 98B 99B 100A
124
E.S.Prakash. MCQs in Medical Physiology, May 2013
101C 102BC make a significant contribution to the osmolality
and osmotic pressure of plasma because they are
Answer Explanations: found in concentrations much lower than Na, Cl –
1. Volume of distribution of this substance = 30% the most effective osmoles in ECF. However,
of total body water (TBW). In a healthy adult albumin is quantitatively the most important
male weighing about 70 kg, the volumes of ICF contributor to the colloid osmotic pressure of
and ECF are 28 and 14 L respectively; i.e., 1/3rd plasma (oncotic pressure) because it is restricted
of TBW is outside cells. Thus, the substance in to plasma and does not exit the capillary as fast.
question distributed exclusively in ECF. Among plasma proteins it is the most abundant
and hence it contributes the most to colloid
2. TBW = ICF + ECF volumes = 28 + 14 = 42 L osmotic pressure of plasma.
Concentration of potassium in ICF = 140 mM
Concentration of potassium in ECF = 5 mM 7. Crystalloids include NaCl, Ringer’s lactate;
Plasma volume = 3.5 L colloids include large molecular weight
Volumes of all other fluids combined = 38.5 L substances such as plasma proteins, plasma
Amount of K contained in plasma is about 20 expanders like dextran, gelatin and hydroxyethyl
mM. starch.
Amount of K in all other fluids = 3980 mM
Thus, about 0.4% of potassium is in plasma 8. While the strength duration curve can tell us if
a nerve is functioning or not or what the degree of
3. 1 gram mole of MgSO4 = 1000 mM MgSO4 impairment is at a point in time, of late, it has
Each molecule of MgSO4 dissociates to give 2 been superseded by electromyography which has
ions viz., Mg and sulphate. this capability and a wider range of uses. That is
Thus, 1000 mM = 2000 mOsm/L why, perhaps, little mention is made about SD
1 mOsm/L exerts a pressure of 19.3 mm Hg curve these days in textbooks. The only advantage
Therefore, 2000 mOsm/L will exert an osmotic of the SD curve is that it is noninvasive.
pressure = 19.3 × 2000 = 38600 mm Hg Electromyography (EMG) can be recorded with
surface as well as needle electrodes. The
4. Concentration of KCl = 39 mg % = 39 mg/dL = occurrence of reinnervation potentials or
390 mg/L = 0.390 g/L denervation potentials (obtainable only with an
Molar mass of K = 39 g EMG) may be used for prognostication.
1 mole of K = 39 g/L
Thus, 0.39 g of K = 10 mM of K 11. A 1ºC decrease in temperature reduces
cerebral metabolic rate for oxygen by 7%.
5. The term emiocytosis based on a limited search
appears to be essentially the same as exocytosis, 12. Type B fibers are more susceptible to
and it is not frequently used in recent literature. blockade by anesthetics compared to type A
Emiocytosis (exocytosis) of secretory granules fibers, which have a larger diameter. Sympathetic
requires a rise in intracellular calcium – for preganglionic neurons, which originate in the
example, the release of insulin by B cells of the intermediolateral column of the thoracolumbar
pancreas involves depolarization of the B cell by segments of the spinal cord, are type B fibers.
closure of K channels followed by influx of
calcium through voltage dependent calcium 15. In the dark, the prosthetic group retinene in
channels. The transient rise in intracellular retinaldehyde is in the 11-cis configuration. The
calcium brings about emiocytosis of granules that action of light is to change the shape of this to the
contain insulin, proinsulin, C-peptide. all-trans isomer.

6. Osmotic pressure is simply dependent upon the 17. Sleep walking, bed wetting and night terrors
number of particles and not the mass of a occur during slow wave sleep or, more
substance. Albumin (or plasma proteins) does not specifically during arousal from slow wave sleep.
125
E.S.Prakash. MCQs in Medical Physiology, May 2013
They are not associated with REM sleep See p
200-1, Ch 11, WFG, 2005. 29. This question tests if we understand the
importance of masking the untested ear when
18. Cerebral blood flow is regulated by local performing the Rinne’s test. For example, if we
metabolites and hypoxia would cause vasodilation were testing the right ear, how sure are we that the
and thereby slightly increase flow. Similarly, person hears through the right ear and not on the
hypercapnia produces cerebral vasodilation and an left? To ensure this does not happen, the untested
increase in cerebral blood flow. ear is masked using a Barany’s noise box.
Alternately, we could ask the patient to insert his
23. Cushing’s triad, which was originally finger in the left ear and make quick noisy
described in some patients with raised intracranial movements during the Rinne’s test.
tension, includes hypertension, bradycardia and
irregular respiration but it is uncommon to have Second, the results of Rinne’s and Weber’s test
all signs in a patient. must be interpreted together and not in isolation.

24. Neurotransmitters that increase food intake are In this case, the Rinne’s test was negative (called
said to be orexigenic. Two neurotransmitters with ‘false negative’) because there is apparently no
orexigenic effects are the polypeptides orexin A defect in ossicular conduction on the right side.
and orexin B. Orexins are synthesized in neurons Furthermore, Weber’s test was localized to the left
located in the lateral hypothalamus. The feeding side. Taken together, this is indicative of
center is located in the lateral hypothalamus. sensorineural deafness on the right side.
Ghrelin and neuropeptide Y also stimulate food
intake. In contrast, leptin from adipose tissue acts Absolute bone conduction of a test subject could
on the hypothalamus signaling satiety and inhibits also be compared with a reference subject whose
food intake. hearing is known to be normal. The results of this
test (Schwabach’s test) are useful in detecting
25. Melatonin is synthesized in the pineal gland early sensorineural deafness.
starting from tryptophan.
31. ADH is stored in the posterior pituitary bound
26. Postganglionic sympathetic (noradrenergic) to neurophysins. However, ADH circulates free in
neurons innervating the pineal commence in the plasma.
superior cervical ganglion and reach it via nervi
conarii. 33. Using Henderson equation, [H+] of blood can
be calculated to be 72 nM. That is, the individual
27. Specific dynamic action (SDA) of food (also has metabolic acidosis. CSF [H+] is directly
called thermic effect of feeding, diet induced proportional to PaCO2. Since PaCO2 is 30 mm
thermogenesis) is a physiological phenomenon Hg (10 mm Hg lower than it normally is), CSF pH
that represents energy expended on all activities must be higher than normal. Normally, CSF pH is
of the body incidental to the ingestion, digestion, 7.33. From the options given, 7.4 is the only
absorption, and assimilation of a meal. Metabolic answer one can choose.
rate starts increasing 30 min after a meal, stays
high for about 2 hours and declines to the resting 34. Normally, total serum calcium ranges from 8-
level within 4 hours. The energy expended 10 mg/dL. Normally, the ratio of protein bound
depends upon the volume, caloric content, calcium to ionized calcium is approximately 1:1.
consistency, and composition of meal. In this case, total calcium = 6 mg/dL. A decrease
in total calcium could be due to:
28. While cutaneous vasodilation and sweating go • Decrease in serum protein
together, however, cutaneous vasodilation per se • Decrease in ionized calcium
could contribute to heat loss even if it were not
accompanied by increased sweating.
126
E.S.Prakash. MCQs in Medical Physiology, May 2013
Thus, it is appropriate to seek to determine if erythropoiesis. This may be one reason why
hypoalbuminemia is the cause of hypocalcemia. anemia occurs in hyperestrogenic states; perhaps,
the anemia is relative rather than absolute, i.e.,
36. The 46 XX primary oocyte undergoes meiosis due to a greater increase in plasma volume rather
which consists of two phases. than a reduction in total red blood cell volume.
High levels of ACTH (via cortisol) stimulate
Phase 1: Meiosis (reduction division) results in erythropoiesis and polycythemia may be a striking
the formation of secondary oocyte (with a large feature of Cushing syndrome. Thyroxine and
amount of cytoplasm) and the first polar body prolactin also stimulate erythropoiesis.
(with little cytoplasm). The first polar body is
extruded at the end of the first phase of meiotic 48. Sensitization of the mother’s immune system
division. This is immediately followed by with fetal Rh antigens results in the production of
ovulation. See WF Ganong, 21st Ed, Ch 23, anti-Rh immunoglobulin. First, IgM is formed but
Gonads: development and function of the this is too large to cross the placental barrier.
reproductive system, pp 438. I think that the best
answer for the question is “at the time of The major antibody in the primary immune
ovulation”. Ovulation is the release of the response is IgM. However, over a time period that
secondary oocyte from the ovary. varies from individual to individual, IgG is also
produced as part of this response, although the
Phase 2: The 23X secondary oocyte immediately titer of IgG is usually not sufficient to evoke
begins a mitotic division but this is arrested in significant hemolysis in the first pregnancy. On
metaphase until fertilization occurs. After the other hand, in a woman who has been
fertilization, the mitotic division is completed and previously sensitized with Rh antigen and anti D
the second polar body is extruded. IgG titers were significant, significant hemolysis
could occur even during the first pregnancy.
37. The phenotypic sex is female (it is a baby
girl); gonadal sex is male (because the surgeons 52. You can see that the depolarization is from a
are telling that the inguinal masses are testes). We focus in the ventricle. The beat could not have
are to predict chromosomal sex (genetic sex) originated in an atrial focus since it has not reset
based on this information. Since testes are present, the sinus rhythm. An atrial premature beat is not
the sex-determining region (SRY) of the Y usually followed by a compensatory pause; it is
chromosome, which encodes a transcription factor conducted to the SA node and it “resets” sinus
that is required for the development of embryonic rhythm. In contrast, a ventricular premature beat
testes, must be present. This gene is normally is usually not conducted retrograde through the
located on the Y chromosome. Thus the most AV node to the SA node, so it does not reset
likely genotype is 46XY. In contrast the 47XXY normal sinus rhythm.
genotype does not lead to feminization (the
phenotype is that of Klinefelter’s syndrome). The duration of premature ventricular
depolarization is longer than a normal QRS
With the information we are given in the problem, complex since it is conducted much slowly
we cannot however distinguish whether this baby through the myocardium rather than through the
girl has testosterone 5-alpha reductase (type 2) normal conducting pathway. Thus the resulting
deficiency or complete androgen insensitivity. If ventricular contraction is unlikely to have been
needed see the summary on principles of really “premature”. So enough ventricular filling
diagnosis of disorders of sexual differentiation in would likely have occurred and the beat must
the earlier chapter on Endocrinology and have been strong enough for the aortic valve to
Reproduction. open (and then close). Thus both heart sounds
would most likely have been present at beat X.
45. Androgens stimulate erythropoiesis. Estrogens The learning point here is that a premature
have been noted to have an inhibitory influence on
127
E.S.Prakash. MCQs in Medical Physiology, May 2013
ventricular depolarization does not always result Summary: When MAP is reduced below 70 mm
in a premature beat. Hg, arterial baroreceptor discharge is maximally
inhibited and sympathetic outflow is disinhibited
53. For correct estimation of blood volume as well as a result of this; apart from this, activation of the
as cardiac output, the dye must remain in the arterial chemoreflex contributes significantly to
blood stream. “increasing” MAP at such low pressures.

55. Without the carotid pulse transducer, LVET 61. Why does edema not occur in early stage
cannot be determined. Since PEP = QS2-LVET, forward heart failure?
PEP cannot also be determined. Follow this sequence from i to vii
i. Acute cardiac failure (‘forward failure’)
56. Note that blood loss occurs over a 30-minute ii. By definition, cardiac output (Q) is
period (a common example is venesection of a reduced
healthy adult blood donor). There occurs only a iii. Thus, whole body perfusion (and oxygen
slight increase in HR because of a reduction in delivery is reduced)
central blood volume. BP is maintained in the iv. Accumulation of metabolites causes a drop
steady state, and one cannot detect the fall in BP in peripheral resistance (TPR)
by intermittent sphygmomanometry. v. MAP= Q × TPR
vi. MAP falls (i.e. systemic capillary
57. When MAP is in the normal range, i.e., hydrostatic pressure falls) as a result of a
between 70 and 110 mm Hg, an increase in BP reduction in Q and TPR
results in an increase in discharge rate from the vii. Since capillary hydrostatic pressure falls,
carotid sinus. Activity in the buffer nerves inhibits edema is unlikely.
tonic vasoconstrictor discharge from the medulla.
Therefore, sectioning of the buffer nerves when Stated in another way, if edema is not observed in
MAP is normal would result in acute elevation of an individual who truly has heart failure, then this
BP. This is called neurogenic hypertension. is the most likely explanation.

A MAP less than 70 mm Hg is a hypotensive 62. All organs are capable of autoregulating their
state. When MAP is as low as 60 mm Hg, blood flows. Although cerebral vessels have
discharge rate in carotid baroreceptors is already noradrenergic innervation, cerebral blood flow
minimal and the resulting reflex itself is not chronically under neural control.
sympathoexcitation is manifest. Cerebral blood flow is excellently autoregulated
Thus, sectioning of afferents from arterial in the steady state when mean arterial pressure is
baroreceptors at such a low MAP would not be between 65 and 140 mm Hg.
expected to have further sympathoexcitatory
effects. Although the heart is also capable of excellent
autoregulation, under certain circumstances,
On the other hand, when MAP is as low as 60 mm neural regulatory mechanisms override local
Hg, blood flow through the arterial regulatory mechanisms. For example, sudden
chemoreceptors near the carotid sinus would be excessive discharge in the sympathetic fibers to
reduced. A reduction in either PaO2 and or a the heart can cause severe coronary vasospasm
decrease in flow through the chemoreceptors lead and a severe reduction in coronary blood flow.
to a reflex increase (via afferents from carotid
bodies) in sympathetic outflow that serves to Cutaneous blood flow is most often varied in
bring BP back into the normal range. Thus, when response to neural signals from the hypothalamus.
the buffer nerves are sectioned at a time when It is the vascular bed that is most consistently
MAP is lower than normal, BP would fall further. under neural control.
See p. 628, Ch 33, WFG, 2005.

128
E.S.Prakash. MCQs in Medical Physiology, May 2013
64. If needed, review tracings of pressure and
volume in various chambers during various phases Methemoglobin binds oxygen but is not capable
of the cardiac cycle, including pulmonary artery of unloading oxygen until it is reduced to
pressures. ferrohemoglobin.

65. Reynolds number Re = ρDV/η Clinically, respiratory acidosis is always


ρ is the density of the fluid; accompanied by arterial hypoxemia (type II
D is the diameter of the vessel; respiratory failure).
V is the blood flow velocity; and
η is the viscosity of fluid. However, in cyanide poisoning, oxygen content of
Flow is turbulent when Re exceeds 3000; blood is normal, yet utilization of oxygen in the
See p. 583, Ch 30, WFG, 2005. mitochondria is inhibited because cyanide inhibits
cytochrome oxidase, the final donor of electrons
66. The major force driving filtration is the to molecular oxygen.
capillary hydrostatic pressure, which is normally
about 40 mm Hg at the arterial end of the capillary 71. The fact that oxygen saturation of Hb is as low
in a capillary that is at the level of the heart. More as 60% although PaO2 is as high as 300 mm Hg
precisely, transcapillary hydrostatic pressure suggests a diagnosis of carbon monoxide
gradient is the major factor. poisoning straightaway.

67. At functional residual capacity, lungs and the 72. This reflex helps a newborn child inflate its
chest wall recoil in opposite directions causing the liquid filled lungs.
intrapleural (intrathoracic) pressure to be negative.
73. O2 carrying capacity of blood:
68. Hb F binds 2,3 BPG less avidly compared to 0.003 ml O2 per mm Hg O2 tension per deciliter of
Hb A and therefore has a greater affinity for blood
oxygen relative to Hb A. This is one reason why it i.e., 3 × 10-3 ml O2 per mm Hg O2tension per
is able to draw O2 from Hb A. deciliter of blood
= 3 × 10-5 ml of O2 per mm Hg O2 tension per ml
69. Hey! Here is a professor with a passion for SI of blood
units, asking a “you miss – I hit” kind of a Here, O2 carrying capacity = 0.0025 ml.
question! The question is designed to test whether Oxygen tension should have been 2.5 × 10-3
we know how to convert pressure in mm Hg to Divided by 3 × 10-5
Pascals. Here it is: = 80 mm Hg
1 atm = 760 mm Hg = 100 kPa
7.6 mm Hg = 1 kPa 76. PAH clearance gives effective RPF
Clearance of PAH depends upon its plasma
70. Methemoglobin cannot transport oxygen. Less concentration. Secretory and reabsorptive
than 1% of Hb is in the Fe3+ state. Methemoglobin processes have a rate maximum i.e., they can be
cannot also unload oxygen much like saturated. When a high dose is used, PAH
carboxyhemoglobin. Thus, there is also a leftward secretory capacity is overwhelmed and clearance
shift of the oxyhemoglobin dissociation curve. of PAH reflects this secretory capacity rather than
Taken together, the oxygen content of blood is RPF. So UPAH V/ PPAH i.e. PAH clearance will be
reduced in methemoglobinemia and the oxygen low, and estimated RPF will be falsely low.
unloading capacity of blood is reduced even more.
80. Estradiol (an estrogen) ultimately terminates
Carbon monoxide poisoning reduces oxygen linear growth by causing epiphyses to fuse to the
content of arterial blood by competing with long bones (epiphysial closure) in males as well
oxygen to bind heme. Respiratory acidosis is as females. In males, this estradiol is not from
associated with severe hypoxemia. that in the circulation; it is estradiol that is formed
129
E.S.Prakash. MCQs in Medical Physiology, May 2013
locally in the epiphysial growth plate by the action so, you will feel the tension in your biceps, yet the
of bone aromatase enzyme on testosterone that elbow extends; that is, although biceps develops
reaches it from the testes. This is not to say that tension, it lengthens. One must understand the
sex hormones inhibit linear growth. Rather the difference between contraction of sarcomeres
pubertal growth spurt is due to the synergism which leads to development of tension, and
between sex hormones and other hormones (GH, change in muscle length that depends upon load.
IGF-1, thyroid hormones), and when the Sarcomeres contract and develop force but the
epiphyseal plate concentration of estradiol reaches muscle can shorten or lengthen depending upon
a certain threshold, it mediates epiphyseal closure. the load. If a muscle lengthens while developing
Patients with sexual precocity are apt to be force, external work done is negative.
dwarfed because of premature epiphysial closure.
91. Plasma osmolality (mosm/L) =
82. Pregnancy is a classic example of a 2 [Na+] + [glucose] / 20 + [BUN] ×18/50
hyperdynamic circulatory state in which total (mmol/L) (mg/dL) (mg/dL)
peripheral resistance is lowered as a result of
opening a new vascular bed (placenta) in the 97. It is now established that the longheld belief
systemic circulation. Blood volume, heart rate and that we do not add brain cells after birth is
cardiac output are increased. The diastolic incorrect. New neurons arise from stem cells in
pressure drops and pulse pressure increases; MAP the olfactory bulb and hippocampus and since
is maintained. formation of long-term memories occurs in the
hippocampus, it is possible that the two are
84. This is because when flow is reduced, flow related; this hypothesis is currently intensely
velocity is reduced and temperature is quickly researched. [p 270, Ch 16, WFG, 2005]
dissipated thereby rendering an accurate
measurement of pulmonary blood flow difficult. 98. The tympanic reflex, labyrinthine reflex and
neck righting reflex are not integrated in the spinal
On the other hand, Fick’s principle is dependable cord.
when cardiac output is low since arteriovenous
oxygen difference is quite high and oxygen uptake 100. PAO2
across the lungs reduces. = [(PB-PH2O) × FiO2] – [(PACO2) / RER].
This is the “alveolar gas equation” that is used to
Note that cardiac output (or pulmonary blood estimate the oxygen tension of alveolar gas
flow) equals oxygen uptake across the lungs In this problem,
divided by whole-body arteriovenous oxygen PB = 760 mm Hg, FiO2 = 0.8; PaCO2 = PACO2
difference. = 40 mm Hg, and RER is assumed to be 0.8
Substituting we get, PAO2 = 520 mm Hg
When there are intracardiac shunts, Fick’s method Arterial PO2 = 100 mm Hg
will either overestimate or underestimate cardiac Thus, D (A-a) O2 = 420 mm Hg.
output depending on whether the shunt is left-to- This represents a considerable defect in gas
right or right-to-left. exchange as normally the difference between
alveolar and arterial oxygen tension is not greater
Furthermore, when there is a ventricular septal than 10 mmHg.
defect, mixed venous blood must be taken from
the right atrium rather than the pulmonary artery, The learning point here is that a PaO2 of 100
i.e. venous blood must be taken proximal to the mmHg is not necessarily reflective of gas
shunt. Harrison’s Principle of Internal Medicine, exchange function of lungs as it is influenced by
16th ed., vol. 2, p 1329-30. the oxygen concentration of inspired gas.

87. Take a rather heavy object in your hand and Rather, D (A-a) O2 is a measure of oxygen
try lowering it toward the ground. When you do exchange function of lungs.
130
E.S.Prakash. MCQs in Medical Physiology, May 2013
More Questions for Self-Study: General
101. Extension of the neck, positive pressure Physiology
ventilation, and emphysema increase ventilation
of dead space. Unless otherwise indicated, choose the single
best answer.
In the case of positive pressure ventilation, total
dead space increases depending on: 1. The philosophy that all vital mechanisms are
• the volume of the tubing used; directed toward maintaining constancy of
• regional differences in lung compliance in composition of the internal environment and that
patients with lung disease this is necessary for the sustenance of life was
first propounded by:
*************************************** A. Claude Bernard
B. Walter B Cannon
C. William Harvey
D. Alan Hodgkin

2. The term ‘homeostasis’ was coined by:


A. Claude Bernard
B. Walter B Cannon
C. Homer Smith
D. William Harvey

3. In adults, ECF volume is about _____% of


body weight
A. 10%
B. 20%
C. 30%
D. 40%

4. In a healthy adult male weighing 70 kg, the


total volume of fluid present in the transcellular
compartment does not exceed:
A. 1 liter
B. 3 liters
C. 5 liters
D. 7 liters

5. Which of the following statements regarding


body fluid compartments is incorrect?
In health,
A. blood volume is about 70-80 ml/kg body
weight
B. the ratio of ICF to ECF volumes is about 2
C. ICF volume accounts for about 60% of body
weight.

6. Radioiodinated albumin is a marker that can be


used to measure the volume of:
A. plasma
B. interstitial fluid
131
E.S.Prakash. MCQs in Medical Physiology, May 2013
C. ECF A. Extrusion of calcium into ECF
D. red blood cells B. Efflux of K during repolarization
C. Entry of chloride into neurons
7. Which of the following is not isosmotic with D. Transcytosis (vesicular transport)
normal human plasma?
A. 5% dextrose 14. The most abundant protein in mammalian
B. 0.85% NaCl cells is:
C. 20% mannitol A. actin
B. collagen
8. In the steady state, the value of which of the C. titin
following variables is the same in ICF and ECF? D. dystrophin
A. pH
B. osmolality 15. Gap junctions are made up of a protein called:
C. concentration of proteins A. connexin
D. number of osmoles B. clathrin
C. cadherin
9. ICF volume does not change when dehydration D. calcineurin
is:
A. isotonic 16. The mitochondrial genome is absent from:
B. hypertonic A. sperm cells
C. hypotonic B. ovum
C. mature red blood cells
10. Which of the following modes of transport is D. white blood cells
quantitatively more important?
A. Diffusion Whatzit
B. Filtration 1. The most abundant cation in ECF
C. Vesicular transport 2. The most abundant anion in ECF
3. The most abundant cation in ICF
11. Which of the following is an example of 4. The most abundant protein in blood
primary active transport? 5. The ion that is absorbed as well as secreted
A. Ca extrusion from cells by the Na-Ca by the kidney
exchanger
B. Glucose entry into cells through glucose Answers (to Whatzit):
transporter 2 (GLUT-2) 1. Sodium
C. H2O flux across cell membranes through 2. Chloride
aquaporins 3. Potassium
D. Glucose uptake into intestinal epithelial cells 4. Hemoglobin
by Na-glucose cotransporter 1 (SGLT-1) 5. Potassium
E. Ca sequestration in sarcoplasmic reticulum by
Ca-ATPase Answers for Practice Questions in General
Physiology
12. Sodium-glucose cotransport in the intestine 1A 2B 3A 4A 5C
and kidney is an example of: 6A 7C 8B 9A 10B
A. primary active transport 11E 12B 13AD 14A 15A
B. secondary active transport 16C
C. facilitated diffusion
D. passive transport 5. ICF volume constitutes 2/3rd of total body water
(not body weight). In a healthy adult weighing
13. Which of the following is / are active transport about 70 kg, ICF volume (28 liters) is about 40%
processes? (Tick all that apply) of total body weight.
132
E.S.Prakash. MCQs in Medical Physiology, May 2013
secondary oocyte during fertilization; i.e.,
9. Isotonic dehydration (example, that due to loss mitochondrial DNA in the zygote is exclusively of
of blood or plasma), is the result of proportional maternal origin.
loss of NaCl and water (basically a fluid with the
composition of plasma minus proteins) from ECF, ************************************************
and there is no change in ECF osmolality. There is More Questions for Self-Study: Nerve &
thus no drive for movement of water between ICF Muscle Physiology
and ECF, and therefore there is no change in ICF
volume. Therapy is aimed at replenishing ECF In each of the following questions, select the
volume such as with 0.9% NaCl (a solution that is single best answer.
isotonic with normal human plasma) is used. In
hypertonic dehydration, steady state ECF 1. The RMP of some neurons is equal to the
osmolality increases; thus, it leads to intracellular equilibrium potential of:
dehydration as well. In hypotonic dehydration, A. Na
steady state osmolality of ECF is decreased and B. K
some of this water will enter cells. C. Cl
D. Ca
11-13. If needed, review the Supplement on
Transport across cell membranes in Chapter 1. 2. The equilibrium potential of chloride in
mammalian spinal motor neurons is about:
14. Proteins are translocated from capillaries into A. + 20 mV
the interstitium by transcytosis (vesicular B. - 40 mV
transport). This involves endocytosis of the C. - 70 mV
protein molecules into vesicles in endothelial cells D. - 90 mV (inside negative)
and exocytosis of these vesicles into the
interstitium. 3. The largest known axons are found in:
A. humans
18. Actin is present in virtually all cells although B. whales
most abundant in muscle. Collagen is the most C. squids
abundant protein in extracellular matrix. Titin is D. ostriches
the largest protein in the human body. Dystrophin
is a glycoprotein in the muscle cell membrane; it 4. The Nernst potential (also called the
is involved in transmitting the tension generated equilibrium potential) is positive for:
by shortening of sarcomeres eventually to the A. Na and Cl
tendon. B. Na and K
C. Na and Ca
15. Clathrin is a protein involved in receptor- D. K and Cl
mediated endocytosis; it is the protein that coats
pits that are the sites of aggregation of cell-surface 5. Which of the following statements regarding
receptors involved in receptor-mediated nerve fibers is incorrect?
endocytosis. Cadherin is a cell adhesion molecule. A. Large diameter fibers conduct faster than
Calcineurin, like calmodulin, is a calcium binding small diameter fibers.
protein found in cells. B. Type C fibers are more susceptible to
inhibition by local anesthetics.
16. Mature RBCs do not have mitochondria; these C. Nerves innervating skeletal muscle contain
cells exclusively use anerobic glycolysis to only one type of nerve fiber.
generate ATP.
6. Excitation and contraction of skeletal muscle
Although sperm cells contain mitochondria, are coupled by:
mitochondria from sperm cells do not enter the A. ATP
B. myosin
133
E.S.Prakash. MCQs in Medical Physiology, May 2013
C. release of calcium into sarcoplasm A. Contraction can occur in the absence of
D. calmodulin extrinsic neural innervation.
B. Within limits, stretch increases tone.
7. Which of the following regarding slow skeletal C. It always exhibits linear length-tension
muscle fibers is false? They, in comparison to fast relationships.
muscle fibers: D. Twitch duration is typically longer when
A. have a greater number of mitochondria compared to skeletal muscle.
B. have higher myoglobin content
C. typically demonstrate oxidative metabolism 14. The calcium-binding protein that plays a key
D. have the faster isoform of myosin ATPase role in the regulation of smooth muscle cell
contraction is:
8. Which of the following is not observed in A. dystrophin
cardiac muscle? B. calmodulin
A. Contraction of the ventricle is an all-or-none C. troponin C
phenomenon D. calcineurin
B. Staircase phenomenon
C. Complete summation of successive 15. An example for multi-unit smooth muscle is:
contractions A. stomach
D. Long refractory period B. myometrium
C. iris
9. Which of the following muscles consists D. vascular smooth muscle
predominantly of fast muscle fibers?
A. Soleus 16. Actin is tethered to Z-lines in the sarcomere
B. Diaphragm by:
C. Orbicularis oculi A. actinin
D. Detrusor B. titin
C. nebulin
10. Which of the following proteins is absent from D. dystrophin
smooth muscle?
A. Actin 17. A stronger than normal stimulus can cause
B. Myosin excitation of nerve or muscle during:
C. Troponin A. absolute refractory period
D. Tropomyosin B. relative refractory period
C. spike potential
11. Which of the following statements about D. overshoot
visceral smooth muscle is incorrect?
A. Neighboring cells are electrically coupled by 18. Under basal conditions, which is the
means of gap junctions predominant energy substrate utilized by cardiac
B. Force is graded by varying intracellular muscle?
calcium A. Carbohydrate
C. Stretch of smooth muscle consistently elicits B. Amino acids
active contraction C. Ketones
D. Fatty acids
12. Oxidative capacity is highest in:
A. type I muscle fibers 19. The gene coding for which of the following
B. type II muscle fibers proteins is the largest?
C. type F muscle fibers A. Titin
B. Dystrophin
13. Which of the following statements about C. Gigantin
gastrointestinal smooth muscle is incorrect? D. Nebulin
134
E.S.Prakash. MCQs in Medical Physiology, May 2013
3. neurotransmitter released by small intensely
20. At a given stimulus intensity, the increase in fluorescent (SIF) cells in autonomic ganglia
the force of muscle contraction with increasing 4. neurotransmitter released by Golgi bottle
frequency of stimulation is referred to as: neurons
A. tetanus 5. neurotransmitter released by Renshaw cells
B. tetany 6. major neurotransmitter released by
C. staircase phenomenon sympathetic postganglionic neurons
D. summation 7. cotransmitter released by sympathetic
postganglionic neurons
Answers: Self-Study in Nerve-Muscle 8. major neurotransmitter released by
Physiology parasympathetic postganglionic neurons
1C 2C 3C 4C 5C 9. cotransmitter released by some
6C 7D 8C 9C 10C parasympathetic postganglionic neurons
11C 12A 13C 14B 15C 10. neurotransmitter released by all autonomic
16A 17B 18D 19B 20C preganglionic neurons

Answer Explanations: Answers:


8. Summation of the type seen in skeletal muscle 1 Glutamate 6 Norepinephrine
cannot occur in cardiac muscle. That is, a second 2 GABA 7 Neuropeptide Y
contraction cannot begin before the previous 3 Dopamine 8 Acetylcholine
contraction (systole) is completed and diastole 4 Glycine 9 Vasoactive
(filling) has commenced. However, a premature intestinal
depolarization and premature contraction can polypeptide
occur when the ventricle is just filling with blood. 5 Glycine 10 Acetylcholine

********************************************** State whether the following statements are true


More Questions for Self-Study: Central or false.
Nervous System
1. There are no photoreceptors overlying the
Some quizzical questions optic disk.

Whatzit: 2. Renshaw cell is an inhibitory interneuron.


1. The photoreceptor protein in rod cells:
2. The organ of hearing: 3. Olfactory receptors are free nerve endings.
3. The fluid present in the scala media:
4. The unit of sound pressure: 4. Sign of Babinski is an unequivocal sign of a
5. The fluid present in the bony labyrinth: corticospinal tract lesion.
6. The muscles subserving the tympanic reflex:
5. The muscle spindle is basically a stretch
Answers to Whatzit: receptor that responds to changes in muscle
1. Rhodopsin length.
2. Organ of Corti
3. Endolymph 6. The muscle spindle is part of a feedback
4. Decibel circuit regulating muscle length.
5. Perilymph
6. Stapedius and tensor tympani 7. The Golgi tendon organ is part of a feedback
circuit regulating muscle force.
What is the
1. major excitatory neurotransmitter in the brain
2. main inhibitory neurotransmitter in the brain
135
E.S.Prakash. MCQs in Medical Physiology, May 2013
8. Extrafusal muscle fibres can be made to 5. Area 4
contract by stimulation of alpha motor
neurons. List B (Function)
A. Visual association area
9. Increased gamma efferent discharge increases B. Primary auditory area
the sensitivity of the stretch reflex. C. Primary visual cortex
D. Primary somatosensory cortex
10. If gamma motor neurons innervating a skeletal E. Primary motor cortex
muscle were cut, tone of that muscle would
decrease. Question 3
List A (Glia)
11. If Ia afferents from skeletal muscle were cut, 1. Oligodendroglia
muscle tone would decrease. 2. Microglia
3. Astrocytes
12. Botulinum toxin inhibits the release of 4. Schwann cells
acetylcholine at the skeletal muscle
neuromuscular junction. List B (Functions)
A. Blood-brain barrier
13. Strychnine produces convulsions by blocking B. Myelin synthesis
glycine receptors on alpha-motor neurons. C. Scavenger cells
D. Neurotransmitter reuptake
14. Tetanospasmin produces spastic paralysis by
preventing the release of glycine from Question 4
Renshaw cells (inhibitory interneurons) in the List A
spinal cord. 1. Eye movements, stance, and gait
2. Truncal and proximal limb movements
Answers: All statements 1-14 are true. 3. Coordination of movements of distal parts of
the limbs
In each of the following questions, match items
in list A with those in list B. List B
A. Vestibulocerebellum
Question 1 B. Spinocerebellum
List A (Functional areas) C. Neocerebellum
1. Satiety center
2. Feeding center Question 5
3. Osmoreceptors List A (Circumventricular organ)
4. Chemoreceptor trigger zone 1. Median eminence
2. Organum vasculosum of lamina terminalis
List B (Location) (OVLT)
A. Ventromedial hypothalamus 3. Subfornical organ (SFO)
B. Anterior hypothalamus 4. Area postrema (AP)
C. Lateral hypothalamus
D. Area postrema List B (Function)
A. When activated by circulating angiotensin II,
Question 2 signal to the thirst center and supraoptic and
List A (Brodmann’s area) paraventricular nuclei of the hypothalamus
1. Areas 3, 1, 2 B. Chemoreceptor trigger zone
2. Area 41 C. Site where hypothalamo-hypophysial portal
3. Area 17 vessels originate
4. Areas 18 and 19
136
E.S.Prakash. MCQs in Medical Physiology, May 2013
Question 6 9. The mechanism that triggers REM sleep is
List A (Cortical Area) located in the pons.
1. Posterior parietal cortex
2. Fusiform gyrus 10. Hyperventilation elicits latent EEG
3. Angular gyrus abnormalities by decreasing cerebral blood
4. Posterior end of superior temporal gyrus flow.

List B (Effect of lesion in this area) Answers:


A. Prosopagnosia 1F 2F 3T 4T 5T
B. Anomic aphasia 6T 7T 8F 9T 10T
C. Wernicke’s aphasia
D. Astereognosis Notes on True-False Statements:
1. Alpha rhythm
Answers:
1 1A 2C 3B 4D 2. REM sleep is called paradoxical sleep because
2 1D 2B 3C 4A it is characterized by high frequency EEG, similar
3 1B 2C 3DA 4B to that observed in the awake state.
4 1A 2B 3C
5 1C 2A 3A 4B 3. Alpha rhythm is blocked and is replaced by a
6 1D 2A 3B 4C high frequency, low voltage beta rhythm.

State if the following statements are true or false. 4. This is the function of the ascending reticular
activating system.
1. The dominant EEG rhythm in an adult human
at rest with eyes closed and the mind 5. The proportion of REM sleep decreases with
wandering is beta rhythm. age.

2. REM sleep is called paradoxical sleep 8. They are seen in Stage II sleep.
because it is characterized by slow waves.
10. The decrease in PaCO2 as a result of
3. The alpha rhythm in EEG is replaced by a hyperventilation results in cerebral
faster irregular low voltage activity when a vasoconstriction; the decrease in cerebral blood
person does mental arithmetic. flow presumably reduces the seizure threshold.

4. Impulses in the ascending reticular activating Temperature regulating mechanisms:


system desynchronize cortical activity
producing arousal. Classify items 1-8 into:
• Mechanisms that serve to conserve heat
5. Typically, in adults, REM sleep comprises • Mechanisms that serve to dissipate heat
25% of total sleep time. • Mechanisms that serve to generate heat
• Mechanisms that reduce heat production
6. REM sleep deprivation produces weight loss
despite adequate caloric intake. 1. Panting
2. Sweating
7. There is more REM sleep in the early hours of 3. Curling up in a ball
morning. 4. Shivering
5. Rigors
8. Sleep spindles and K complexes are seen in 6. Specific dynamic action of food
REM sleep. 7. Horripilation
8. The “it’s too hot to move reaction”
137
E.S.Prakash. MCQs in Medical Physiology, May 2013
5. Histamine released from mast cells in the
Heat loss mechanisms: Panting and sweating stomach stimulates the secretion of HCl by the
parietal cells. What type of signaling is this?
Mechanisms that conserve heat: Curling up in a A. Endocrine
ball, horripilation B. Paracrine
C. Autocrine
Mechanisms that generate heat: Shivering, D. Juxtacrine
rigors, specific dynamic action of food (thermic
effect of feeding), muscular exercise 6. Which of the following has the longest biologic
half-life?
Mechanisms that prevent excessive heat A. Insulin
production: The “it’s too hot to move reaction” B. Angiotensin II
C. Glucagon
********************************************** D. Thyroxine
More Questions for Self-Study: E. ADH
Endocrinology & Reproduction F. Nitric oxide

Instruction: Some questions have more than one 7. The commonest cell type in the anterior
correct answer. Please select all correct answers. pituitary is:
A. somatotroph
1. The term hormone was first used to describe the B. lactotroph
actions of: C. corticotroph
A. secretin D. thyrotroph
B. insulin E. gonadotroph
C. epinephrine
D. growth hormone 8. Growth hormone stimulates the secretion of:
A. somatostatin
2. Which of the following is not a glycoprotein? B. somatomedin (IGF-1)
A. Erythropoietin
B. TRH 9. A diabetic woman who is pregnant and who
C. FSH has been taking recombinant insulin during the
D. hCG pregnancy delivered a baby weighing 5 kg. This is
most closely related to:
3. Which of the following hypophysiotropic A. passage of maternal insulin into the fetus
hormones is a tripeptide? B. elevated levels of fetal insulin
A. TRH C. uteroplacental insufficiency
B. Somatostatin D. hyperglycemia in the mother
C. Dopamine
D. CRH 10. Sertoli cells produce:
A. androgen-binding protein
4. The three glycoprotein hormones each B. inhibin B
containing 2 subunits (an alpha and a beta C. Mullerian inhibiting polypeptide
subunit) produced by the pituitary are:
A. FSH, LH and TRH 11. Normally, the thyroid gland secretes:
B. TRH, ACTH and LH A. CGRP
C. TSH, LH and FSH B. PTH related peptide
D. GH, TSH and LH C. TSH
D. calcitonin

138
E.S.Prakash. MCQs in Medical Physiology, May 2013
12. Which of the following hormones lowers D. skeletal muscle
plasma level of ionized calcium?
A. Calcineurin 20. The normal basal metabolic rate in a euthyroid
B. Calcitonin adult male is (BSA is body surface area):
C. Parathyroid hormone A. 10-15 kcal/m2 BSA/hr
D. PTH related peptide B. 20-25 kcal/m2 BSA/hr
C. 35-40 kcal/m2 BSA/hr
13. Secretion of TSH is inhibited by: D. 45-60 kcal/m2 BSA/hr
A. dopamine
B. TRH 21. TSH secretion is stimulated by:
C. somatostatin A. dopamine
D. T3 and T4 B. TRH
C. somatostatin
14. For a euthyroid woman who is neither D. T3 and T4
pregnant nor nursing, and taking no medication,
the recommended daily dietary allowance of 22. Which of the following statements is
iodine is: incorrect?
A. 75 micrograms A. Thyroxine is biologically more active than
B. 150 micrograms triiodothyronine.
C. 225 micrograms B. The pituitary converts T4 to T3 using type 2
D. 300 micrograms deiodinase
C. Type 3 deiodinase in the placenta inactivates
15. In a healthy adult, the thyroid gland T4 and T3
predominantly secretes: D. Beta-blockers in pharmacologic doses inhibit
A. thyroxine type 3 deiodinase
B. triiodothyronine
C. reverse T3 23. Insulin:
A. decreases K+ uptake by skeletal muscle
16. Most of the T3 in the liver is formed from B. increases protein catabolism in muscle
circulating T4 by the action of: C. decreases cell growth
A. type 1 deiodinase (D1) D. increases the utilization of ketone bodies
B. type 2 deiodinase (D2)
C. type 3 deiodinase (D3) 24. Somatostatin is found in:
A. pancreas
17. Iodine is concentrated in thyroid cells by B. GIT
A. primary active transport C. hypothalamus
B. secondary active transport
C. simple diffusion 25. Worldwide, the commonest preventable cause
D. facilitated diffusion of mental retardation in the newborn is:
A. iodine deficiency
18. Most of the circulating T4 is bound to: B. phenylalanine hydroxylase deficiency
A. prealbumin C. cystic fibrosis
B. albumin D. urea cycle defect
C. thyroxine-binding globulin E. Down syndrome

19. Thyroxine does not increase O2 consumption 26. The secretion of aldosterone is normally
in the: limited to:
A. heart A. zona glomerulosa
B. adult brain B. zona fasciculata
C. brown adipose tissue C. zona reticularis
139
E.S.Prakash. MCQs in Medical Physiology, May 2013
B. 3β-hydroxysteroid dehydrogenase
27. New cortical cells are formed from reserve C. 17α hydroxylase
cells in the: D. steroidogenic acute regulatory protein
A. zona glomerulosa
B. zona fasciculata 35. The most abundant cell type in the islets of
C. zona reticularis Langerhans is:
A. A cells
28. Which of the following is not a B. B cells
catecholamine? C. D cells
A. Epinephrine D. F cells
B. Norepinephrine
C. Dopamine 36. In the management of a child with diabetic
D. Acetylcholine ketoacidosis and a plasma pH of 6.9, insulin
infusion is typically continued even after plasma
29. Normally, androgens are predominantly glucose is reduced to 250 mg/dL, and glucose is
secreted by the: coadministered with insulin. This is done in order
A. zona glomerulosa to:
B. zona fasciculata A. replenish glycogen stores in muscle
C. zona reticularis B. restore total body potassium stores
C. completely oxidize ketone bodies
30. The hormones produced by the ovary include: D. rapidly reduce plasma osmolality to normal
A. estrogens
B. progesterone 37. Which of the following conditions increases
C. relaxin the risk of fasting hypoglycemia?
D. inhibin A. Adrenocortical insufficiency
B. Beta-adrenergic receptor blockade
31. Circulating norepinephrine has greater affinity C. Hyperglucagonemia
for: D. Hypothyroidism
A. α adrenergic receptors E. Acromegaly
B. ß1 adrenergic receptor
C. ß2 adrenergic receptor 38. Which tissues do not require insulin for
glucose uptake? (check all correct answers)
32. The lipolytic effect of catecholamines is A. Red blood cells
mediated by: B. Most parts of the CNS
A. ß1 adrenergic receptors C. Adipose tissue
B. ß2 adrenergic receptors D. Skeletal muscle
C. ß3 adrenergic receptors
D. ß1 & ß3 adrenergic receptors 39. Which of the following is not an energy-
releasing hormone?
33. Deficiency of which of the following A. Growth hormone
hormones is not known to result in a clinically B. Cortisol
significant disorder of regulation of ionized C. Epinephrine
calcium in plasma? D. Insulin
A. Parathyroid hormone E. Glucagon
B. Calcitriol
C. Calcitonin 40. Which of the following hormones is / are
diabetogenic? (Check all correct answers)
34. Congenital lipoid adrenal hyperplasia occurs A. Epinephrine
due to a lack of: B. Cortisol
A. CYP450 scc (desmolase) C. Growth hormone
140
E.S.Prakash. MCQs in Medical Physiology, May 2013
D. Glucagon C. insulin
E. Thyroid hormones D. glucagon
E. epinephrine
41. Insulin secretion from B-cells in pancreas is
inhibited by: 47. Which of the following increase insulin
A. acetylcholine resistance? (Tick all that apply)
B. activation of beta-adrenergic receptors A. Leptin
C. activation of alpha-adrenergic receptors B. TNF-α
D. glucagon C. Resistin
E. somatostatin D. Adiponectin
E. GH secreting tumors
42. For the same plasma concentration of glucose F. Epinephrine secreting tumor
achieved, the insulin secretory response to an oral
glucose load is greater than the insulin secretory 48. The two drugs that are commonly used to
response to an intravenous infusion of glucose. induce diabetes in rats are:
This is because of the: A. streptozotocin
A. potentiation of insulin secretion by glucagon B. alloxan
like polypeptide-1 and glucose dependent C. forskolin
insulinotropic polypeptide (incretin effect) D. capsaicin
when glucose is ingested orally
B. secretion of preformed insulin occurs only 49. Which of the following hormones inhibits
when glucose is ingested orally. ovulation and downregulates GnRH release when
present in high concentrations?
43. Glucagon, when present in high A. Prolactin
concentrations such as in a type I diabetic, B. FSH
facilitates all of the following except: C. LH
A. lipolysis D. Human menopausal gonadotropins
B. gluconeogenesis
C. hepatic glycogenolysis 50. The maternal growth hormone of pregnancy
D. muscle glycogenolysis is:
A. hGH-N
44. Which of the following is a potent stimulator B. hCS
of insulin secretion? C. estriol
A. Somatostatin D. hCG
B. VIP
C. Glucagon like polypeptide-1 (GLP-1) 51. Which of the following is not detectable in the
D. Leptin blood of a nonpregnant female?
A. Oxytocin
45. Hypoglycemia does not stimulate the B. Relaxin
secretion of: C. hCS
A. epinephrine D. Inhibin B
B. cortisol
C. growth hormone 52. Which of the following hormones is not
D. insulin detectable in normal females?
E. glucagon A. Androstenedione
B. Testosterone
46. The hormone of energy storage (hormone of C. Dehydroepiandrosterone sulfate
plenty) is: D. Dihydrotestosterone
A. growth hormone
B. thyroxine
141
E.S.Prakash. MCQs in Medical Physiology, May 2013
53. Which of the following hormones is not 59. The adrenal medulla does not normally
detectable in normal males? secrete:
A. Oxytocin A. epinephrine
B. Estradiol (E2) B. norepinephrine
C. Estrone (E1) C. chromogranin B
D. Progesterone D. vasoactive intestinal polypeptide
E. Prolactin
60. The release of androgens from the adrenal
54. Complete fusion of labioscrotal swellings does cortex is stimulated mainly by:
not occur despite exposure to high concentrations A. LH
of androgens beyond which point in intrauterine B. FSH
life? C. ACTH
A. 3 weeks D. GnRH
B. 6 weeks
C. 9 weeks 61. The biologically most active androgen is:
D. 12 weeks A. testosterone
B. dehydroepiandrosterone
55. Which of the following is the most sensitive C. androstenedione
test of pancreatic B-cell insulin secretory reserve? D. dihydrotestosterone
A. Oral glucose tolerance test
B. Fasting plasma glucose 62. Which of the following would be most likely
C. Urine glucose excretion in an XX female with one mutated androgen
D. Random blood glucose receptor gene?
A. Incomplete development of breasts
56. Chronic androgen abuse by otherwise healthy B. Infertility
male athletes is least likely to result in: C. Defective synthesis of dihydrotestosterone
A. increased erythropoiesis D. Incomplete septation of urogenital sinus
B. increased sperm count
C. increased respiratory endurance 63. Which of the following drugs produces
D. increased muscle mass therapeutic effects by downregulating hormone
receptors in target cells?
57. Which of the following statements is A. FSH
incorrect? B. Long acting GnRH analogs
A. Somatopause is attributed to a decline in the C. Inhibin
activity of GH-IGF1 axis D. Finasteride
B. Andropause is characterized by an increase in
LH and FSH. 64. HCG acts upon the:
C. Menopause is characterized by decreased A. LH receptor
ovarian responsiveness to pituitary B. PRL receptor
gonadotropins. C. GH receptor
D. Adrenopause is characterized by a decrease in D. FSH receptor
plasma DHEA sulfate.
65. The testis-determining gene (or the SRY gene)
58. The hormone of darkness is: is located normally on/in:
A. leptin A. chromosome 6
B. melatonin B. chromosome 12
C. serotonin C. short arm of chromosome 22
D. galanin D. Y chromosome

142
E.S.Prakash. MCQs in Medical Physiology, May 2013
66. The hormone secreted by the pineal is B. blocking nitric oxide synthase
synthesized starting from: C. activating guanylyl cyclase
A. tyrosine D. inhibiting cGMP-phosphodiesterase
B. tryptophan
C. phenylalanine 74. The enhancement of penile erectile function
D. lysine produced by sildenafil citrate is primarily because
of inhibition of:
67. In normal healthy adults on a daytime work A. cGMP-PDE3
shift, at what time does pineal hormone peak in B. cGMP-PDE4
blood? C. cGMP-PDE5
A. 2 am D. cGMP-PDE6
B. 8 am
C. 2 pm 75. The transient loss of the ability to discriminate
D. 8 pm between blue and green following the use of
sildenafil is due to inhibition of:
68. Which of the following statements about the A. cGMP-PDE3
pineal gland in human beings is correct? B. cGMP-PDE4
A. It directly responds to light C. cGMP-PDE5
B. It is part of the nervous system D. cGMP-PDE6
C. It is controlled by the hypothalamus
D. It is rudimentary Answers: Self-Study in Endocrinology &
Reproduction
69. Which of the following cells undergo meiotic 1A 2B 3A 4C 5B
division? 6D 7A 8AB 9B 10ABC
A. Primordial germ cells 11D 12B 13ACD 14B 15A
B. Primary spermatocytes 16A 17B 18C 19B 20C
C. Secondary spermatocytes 21B 22D 23D 24ABC 25A
D. Spermatids 26A 27A 28D 29C 30all
31A 32D 33C 34D 35B
70. The average number of spermatids formed 36C 37A 38AB 39D 40all
from a single spermatogonium is: 41CE 42A 43D 44C 45D
A. 16 46C 47BCEF 48AB 49A 50B
B. 128 51C 52none 53none 54D 55A
C. 512 56B 57none 58B 59D 60C
61D 62none 63B 64A 65D
71. The formation of mature sperm cells from a
66B 67A 68C 69B 70C
primitive germ cell takes an average of:
71C 72B 73D 74C 75D
A. 2 days
B. 1 week
C. 74 days Answer Explanations:
D. 3 months
2. TRH is a tripeptide.
72. Sperm cells first acquire the ability to move
forward (progressive motility) in the: 7. 50% of cells in the anterior pituitary secrete
A. seminiferous tubules growth hormone. See Table 22-1, p.397, WFG,
B. epididymis 2005.
C. female genital tract
8. The somatomedins are insulin-like growth
73. Sildenafil citrate enhances penile erection by: factors 1 (IGF-1) and IGF-2.
A. activating adenylyl cyclase
143
E.S.Prakash. MCQs in Medical Physiology, May 2013
9. The best answer is option B. Hyperglycemia in cortisol, and thyroxine. All of them oppose the
the mother results in fetal hyperinsulinemia. High blood glucose lowering effects of insulin.
levels of insulin in the fetus cause it to have
effects on insulin as well as IGF-1 receptors, and 41. When acting via alpha 2 adrenergic receptors,
this causes the macrosomia. Unbound insulin is epinephrine/norepinephrine inhibit insulin
normally degraded by placental proteases and secretion; however, when acting via beta-
does not reach the fetus. However, insulin bound adrenergic receptors, it stimulates insulin
to anti-insulin antibodies is known to cross the secretion. The significance of sympathetically
placenta, and this mechanism is also believed to mediated inhibition of insulin secretion is it
contribute to fetal macrosomia in women who reduces the likelihood of hypoglycemia during
took animal insulins during pregnancy. exercise. See p. 345, Ch 19, WFG, 2005.

15. The human thyroid releases about 80 42. The insulin response to an oral load of glucose
micrograms of T4 and 4 micrograms of T3 per is greater because of the release of glucose-
day. dependent insulinotropic polypeptide and other
hormones from the GIT, which in turn stimulate
16. One third of circulating T4 is normally insulin secretion. This effect, called the incretin
converted to T3 in humans by the action of 5’- effect, is absent when glucose is administered
deiodinase in the liver and kidneys. See p. 322, Ch intravenously.
18, WFG, 2005.
44. GLP-1 (7-36) is glicentin-like polypeptide, a
18. See p. 321, Ch 18, WFG, 2005. gastrointestinal hormone; it is a potent stimulator
of insulin secretion (incretin). See p. 348, Ch 19,
19. T4 and T3 increase the oxygen consumption WFG, 2005.
of all metabolically active tissues. The exceptions
are the adult brain, testes, uterus, lymph nodes, 47. Leptin and adiponectin are adipokines (i.e.,
spleen and anterior pituitary. See p. 324, Ch 18, hormones from adipocytes) that increase insulin
WFG, 2005. sensitivity. In contrast, TNF-α and resistin
decrease insulin sensitivity. Growth hormone in
23. Insulin increases K uptake by muscle probably excess causes insulin resistance because it has
by stimulating Na-K ATPase. Insulin increases hyperglycemic effects and facilitates lipolysis. See
the oxidation of ketone bodies; this is one reason p.347-8, Ch 19, WFG, 2005.
why it is necessary in the treatment of diabetic
ketoacidosis. 51. Human chorionic somatomammotropin is a
placental hormone.
31. Norepinephrine has a greater affinity for
alpha-adrenergic receptors and epinephrine has a 56. Chronic use of pharmacologic doses of
greater affinity for beta-adrenergic receptors. For androgens suppresses FSH release from the
example, when norepinephrine is administered pituitary thereby inhibiting spermatogenesis.
intravenously the blood pressure increases enough
to trigger a baroreflex-mediated decrease in heart 57. The term adrenopause is used to refer to a
rate although norepinephrine has a direct decline in serum DHEA. Since DHEA is the most
chronotropic action on the heart. abundant adrenocortical hormone, a fall in serum
DHEA indicates a decline in functional adrenal
38. Red blood cells and most regions of the brain cortical mass.
with the exception of cells in the satiety center do
not require insulin for glucose uptake. 62. Because of random inactivation of X-
chromosome, the presence of one mutated
40. Diabetogenic hormones: epinephrine, androgen receptor (with loss of function) is
norepinephrine, glucagon, growth hormone, inconsequential. Furthermore, androgens are not
144
E.S.Prakash. MCQs in Medical Physiology, May 2013
normally involved in feedback regulation of 6. Which of the following statements about CCK
gonadotropin secretion (and consequently is incorrect?
fertility) in females. In contrast, XY zygotes with A. It causes gall bladder contraction
a loss of function mutation in the androgen B. It relaxes the sphincter of Oddi
receptor gene develop manifestations attributable C. It relaxes the pyloric sphincter
to androgen insensivity, and are phenotypically D. It stimulates the secretion of an enzyme-rich
female. pancreatic juice

63. Long acting GnRH analogs suppress FSH and 7. Trypsinogen, a pancreatic proenzyme, is
LH release by downregulating GnRH receptors in activated by:
the pituitary, and thereby inhibit ovulation. A. enterokinase
B. hydrochloric acid
********************************************** C. HCO3
More Questions for Self-Study in D. pancreatic trypsin inhibitor
Gastrointestinal Physiology
8. Most pancreatic zymogens are activated in the
1. The term ‘hormone’ was coined by Ernest H. duodenal lumen by:
Starling to describe the actions of: A. enterokinase (enteropeptidase)
A. secretin B. trypsin
B. insulin C. trypsin activator protein
C. growth hormone
D. glucagon 9. Trypsin inhibitor:
A. inhibits the action of pancreatic enzymes in
2. Which of the following inhibits gastric acid the lumen of the duodenum
secretion? B. inhibits activation of trypsin in the pancreas
A. Pentagastrin C. deficiency is implicated in acute pancreatitis
B. Hypoglycemia D. is produced by enterocytes
C. Acetylcholine
D. PGE2 10. The most abundant bile acid is:
A. cholic acid
3. Complete inhibition of gastric acid secretion is B. chenodeoxycholic acid
achieved with pharmacologic doses of: C. deoxycholic acid
A. H+–K+ ATPase inhibitor D. lithocholic acid
B. H2 receptor blocker
C. M2 receptor blocker 11. In healthy humans, active absorption of bile
D. CCK-B receptor blocker acids and bile salts occurs mainly in the:
A. duodenum
4. Vagally mediated increase in gastric acid B. jejunum
secretion is partly mediated by: C. terminal ileum
A. gastrin releasing peptide (GRP) D. colon
B. gastric inhibitory peptide (GIP)
C. vasoactive intestinal polypeptide (VIP) 12. Bile acids are synthesized starting from:
D. somatostatin A. cholesterol
B. fatty acids
5. The optimum pH for the activity of pepsin is: C. lecithin
A. less than 1 D. bile salts
B. between 1.6 and 3.2
C. between 3 and 5 13. Vitamin B12 is mainly absorbed in the:
D. between 6 and 7 A. terminal ileum
B. upper jejunum
145
E.S.Prakash. MCQs in Medical Physiology, May 2013
C. duodenum D. 180 g/hour
D. stomach
21. The least oxygenated and consequently the
14. Calcium absorption from the intestine: zone in the hepatic acinus most vulnerable to
A. is facilitated by vitamin D ischemia is:
B. is decreased by phytates and oxalates A. zone I
C. occurs mainly in the upper small intestine B. zone II
C. zone III
15. The protein that transports iron in the plasma
from the site of absorption to cells engaged in 22. In healthy humans, the earliest the first part of
erythropoiesis is: a test meal reaches the cecum is in about:
A. hepcidin A. 4 hours
B. transferrin B. 6 hours
C. ferritin C. 9 hours
D. hemosiderin D. 12 hours

16. In healthy humans, iron in tissues other than 23. Normally, the main function of the colon is
red blood cells is stored principally in absorption of:
combination with: A. Na, Cl and H2O
A. ferritin B. triacylglycerols
B. transferrin C. secondary bile acids
C. hepcidin D. iron
D. hemosiderin
24. Gastric emptying is slowest after a meal
17. Which of the following is an endopeptidase? containing:
A. Carboxypeptidase A A. fat
B. Deoxyribonuclease B. carbohydrate
C. Trypsin C. protein
D. Dipeptidase D. indigestible fiber

18. Which of the following enzymes is located in 25. Slow waves in the GIT are initiated by:
the brush border of enterocytes in the small A. I cells
intestine? B. K cells
A. Lactase C. interstitial cells of Cajal
B. Colipase D. S cells
C. Cholesterol ester hydrolase
D. Lingual lipase 26. The most alkaline exocrine secretion is:
A. bile
19. Disaccharidases are present in: B. pancreatic juice
A. pancreatic acinar cells C. intestinal juice
B. brush border of enterocytes D. saliva
C. D cells of pancreas
D. Brunner’s glands 27. The odor of feces is partly due to:
A. stercobilinogen
20. In healthy humans, the maximal rate of B. indole, skatole and sulfides
glucose absorption from the intestine is estimated C. primary bile acids
to be about: D. secondary bile acids
A. 10 g/hour
B. 40 g/hour 28. The normal rate of bile salt synthesis is:
C. 120 g/hour A. 0.05 – 0.1 g/day
146
E.S.Prakash. MCQs in Medical Physiology, May 2013
B. 0.2 – 0.4 g/day cells stimulates the release of histamine from
C. 0.8 – 1.2 g/day enterochromaffin like cells in the stomach, and
D. 2 – 2.5 g/day acid secretion from parietal cells.

29. Which of the following enzymes is not Regarding option B, glucose-dependent


synthesized by the pancreas? insulinotropic polypeptide (GIP) was previously
A. DNAase called gastric inhibitory peptide (also GIP).
B. Proelastase However, now it is clear that the physiologic
C. Aminopeptidases action of GIP is stimulation of insulin secretion.
D. Pepsin Its inhibitory effects on gastric secretion occur
only at pharmacologic doses.
30. The basal acid output / maximum acid output
ratio is normally less than: 5. CCK contracts the pyloric sphincter thereby
A. 0.1 inhibiting gastric emptying.
B. 0.6
C. 0.8 7. Activation of trypsinogen requires
D. 0.9 enterokinase. Enteropeptidase (enterokinase) is
an intestinal brush border enzyme that catalyzes
31. Which of the following cells sample the the proteolytic activation of trypsinogen to
antigenic milieu of the gastrointestinal lumen? trypsin.
A. P cells
B. I cells 8. Trypsin is the common activator of all
C. K cells pancreatic proenzymes. This ensures that
D. M cells (microfold cells) pancreatic proenzymes are activated only when
E. Mo cells they reach the intestinal lumen. Once trypsin is
formed, it catalyzes activation of trypsinogen and
32. The defecation reflex is integrated in the: other pancreatic enzymes secreted as proenzymes.
A. brain stem
B. pons ************************************************
C. medulla More Questions for Self-Study: Renal &
D. spinal cord Acid-Base Physiology
Answers: Questions for Self-Study in 1. Which of the following statements about
Gastrointestinal Physiology control of micturition is incorrect?
1A 2D 3A 4A 5B A. Deafferentation of the bladder leads to
6C 7A 8B 9B 10A overflow incontinence.
11C 12A 13A 14ABC 15B B. In healthy adults, the first urge to void is felt
16A 17C 18A 19B 20C at a bladder volume of about 400 ml.
21C 22A 23A 24A 25C C. Decentralization of bladder function results in
26B 27B 28B 29D 30A urinary retention accompanied by overflow
31D 32D incontinence.

3. This is because the H-K ATPase is the final 2. What is the immediate effect of spinal cord
common pathway for acid secretion in the parietal transection on bladder function?
cell. New proton pumps need to be synthesized A. Urinary retention
before parietal cells can overcome the inhibitory B. Overflow incontinence
effects of H-K ATPase inhibitors. C. Anuria

Vagal neurons ending on G cells in the antrum 3. The substrate for renin is:
release gastrin-releasing peptide. Gastrin from G A. angiotensinogen
147
E.S.Prakash. MCQs in Medical Physiology, May 2013
B. angiotensin I D. a decrease in plasma renin activity
C. angiotensin II
D. bradykinin 12. Which of the following substances normally
has a “clearance ratio” of about 1.2?
4. Angiotensin converting enzyme is a / an: A. Inulin
A. dipeptidyl carboxypeptidase B. Creatinine
B. aminopeptidase C. Urea
C. aspartyl protease D. PAH

5. The kidneys do not synthesize or secrete: 13. Which of the following substances has a
A. calcitriol clearance ratio close to 1?
B. rennin A. Creatinine
C. erythropoietin B. Mannitol
D. PGE2 C. Urea
D. Glucose
6. Renin is released mainly from:
A. macula densa 14. Which of the following substances is cleared
B. juxtaglomerular cells the most by the kidneys?
C. type I medullary interstitial cells A. Inulin
D. peritubular capillaries B. PAH
C. Creatinine
7. In the kidneys, type I medullary interstitial cells D. Urea
secrete:
A. renin 15. Arrange the following substances in the
B. PGE2 descending order of their clearance. [Cr:
C. aquaporins creatinine]
D. aldosterone A. PAH > Inulin > Urea > Cr > Glucose
B. Inulin > PAH > Cr > Urea > Glucose
8. Mesangial cells are similar to: C. PAH > Cr > Inulin > Urea > Glucose
A. pericytes
B. fibrocytes 16. The tubular transport maximum for glucose
C. mast cells (Tmax for glucose) in a healthy young man is
D. ependymal cells about:
A. 100 mg/min
9. Physiologically, important stimulators of B. 225 mg/min
aldosterone production by the zona glomerulosa C. 375 mg/min
cells include (tick all that apply): D. 500 mg/min
A. a rise in plasma [K+]
B. angiotensin II 17. When urine is maximally concentrated, the
C. angiotensin III ratio of osmolality of urine to plasma is about:
A. 3
10. The actions of aldosterone are antagonized by: B. 4.2
A. angiotensin III C. 4.6
B. arginine vasopressin (AVP) D. 6.7
C. ANP
18. High protein diets increase the likelihood of:
11. Chronic licorice ingestion is characterized by: A. metabolic acidosis
A. low blood pressure B. metabolic alkalosis
B. hyperkalemia C. respiratory acidosis
C. metabolic acidosis D. respiratory alkalosis
148
E.S.Prakash. MCQs in Medical Physiology, May 2013
3. 65% of filtered water is reabsorbed in the
19. Which of the following is consistent with proximal tubule.
hypotonic urine? 4. Stimulation of renal sympathetic nerves
A. A high Uosm / Posm ratio inactivates the renin angiotensin system.
B. A urinary pH of 7
C. Positive free water clearance Answers: statement 4 is false; others are true.

20. In a healthy adult, the urge to void urine is Answer Explanations:


first felt at a bladder volume of about: 1. In chronic paraplegics, the voiding reflex
A. 50 ml returns and it may be hyperactive. The
B. 150 ml hyperactive reflex may be because of release of
C. 250 ml the spinal mechanism for micturition from the
D. 350 ml inhibitory effects of higher control. The bladder
wall becomes hypertrophied and bladder capacity
21. In a healthy adult, the volume of urine in the is reduced. This condition is called spastic
bladder that initiates reflex contraction of the neurogenic bladder. See Summary - Disorders of
bladder is about: Micturition on pp.81 of this book. Also see p 728,
A. 50–100 ml Ch 38, WFG, 2005.
B. 100–200 ml
C. 200–300 ml 2. The immediate effect of spinal cord transection
D. 300–400 ml is urinary retention. This is, of course, followed
by overflow incontinence; i.e., excessive stretch
22. Which of the following is not a feature of of the urinary bladder, and local reflex
tabes dorsalis? contractions of the detrusor that is not under
A. Bladder wall hypertrophy voluntary control. See p 728, Ch 38, WFG, 2005.
B. Distended bladder
C. Urinary retention 7. One of the effects of renal prostaglandins is
D. Incontinence mediating redistribution of blood flows within the
kidneys.
23. Arrange the following substances in the
descending order of their concentrations in 8. When mesangial cells contract, surface area
intracellular fluid. available for filtration reduces.
A. K+ > Ca2+ > Na+ > H+
B. H+ > K+ > Na+ > Ca2+ 9. Factors stimulating aldosterone release from
C. K+ > Na+ > Ca2+ > H+ the adrenal cortex: Angiotensin II is the most
D. K+ > Ca2+ > H+ > Na+ important trophic factor for the zona glomerulosa
and one of the two most important stimulators of
Answers: Self-Study in Renal Physiology aldosterone release. Even a 0.5 mM increase in
1B 2A 3A 4A 5B plasma [K] stimulates aldosterone release. In turn,
6B 7B 8A 9ABC 10C aldosterone regulates plasma [K] by promoting K
11D 12B 13B 14B 15C secretion in the collecting ducts. Angiotensin III
16C 17C 18A 19C 20B has 100% aldosterone releasing activity but only
21D 22A 23C 40% of the pressor activity of aldosterone. ACTH
as well as a decrease in plasma [Na] stimulate
True or false: aldosterone release but the stimulatory effects of
1. Urine volume in an adult may increase to as acute administration of pharmacologic doses of
much as 23L / day in the complete absence of ACTH on aldosterone release are transient if at
vasopressin. all.
2. 75 % of potassium is secreted in the
collecting ducts.
149
E.S.Prakash. MCQs in Medical Physiology, May 2013
11. Licorice contains glycyrrhyzinic acid and this (i.e. hypertonic compared to plasma), free water
inhibits 11-beta hydroxysteroid dehydrogenase clearance is negative. When water is lost in excess
(HSD) type 2 in the collecting duct cells in the of osmoles cleared, free water clearance is
nephron. 11-beta HSD type 2 is an enzyme that positive - as in diabetes insipidus.
inactivates cortisol and thereby prevents it from
stimulating the mineralocorticoid receptor. 20. See p 726, Ch 38, WFG, 2005.
[Otherwise, cortisol is capable of binding and
activating the mineralocorticoid receptor.] This is 21. Laplace’s law states that pressure (P) in a
why, physiologically, the mineralocorticoid hollow viscus is directly proportional to wall
receptor is activated only by aldosterone. With tension (T) and inversely with radius (r).
chronic licorice ingestion or with inherited P = T/r
deficiency of 11-beta HSD type 2, cortisol
activates the mineralocorticoid receptor. The As the bladder is distended with urine, the radius
concentration of cortisol is much higher than that as well as passive tension increases
of aldosterone; the result is hypertension, proportionately so there is little change in
hypokalemia and metabolic alkalosis and intravesical pressure. However, at an intravesical
feedback suppression of plasma renin activity and volume of about 300-400 ml, any further stretch
thereby aldosterone secretion by high BP. This is results in “active contraction” of the smooth
an instance of pseudohyperaldosteronism. muscle of the bladder and a sharp increase in
Similarly, hypertension that occurs in 11 beta- intravesical pressure. This results in emptying of
hydroxylase B1 deficiency is due to the actions of the bladder. See p 727, Ch 38, WFG, 2005.
high levels of deoxycorticosterone on the
mineralocorticoid receptor. 22. Tabes dorsalis predominantly affects the
dorsal roots; thus, the effects observed are those
12. Clearance ratio is the ratio of the clearance of of deafferentation: the bladder becomes distended,
a substance by the kidneys to the clearance of thin walled and hypotonic. See Summary -
inulin. The clearance of substances secreted by the Disorders of Micturition in Renal & Acid-Base
nephron is greater than inulin clearance. In Physiology. See p 728, Ch 38, WFG, 2005.
contrast, the clearance of substances (such as
glucose) that are reabsorbed by the nephron is less ***************************************
than inulin clearance.
Bibliography:
14. The clearance of a substance that is actively
secreted by the nephron (example, PAH) is greater Barrett KE, Barman SM, Boitano S and
than the clearance of a substance that is excreted Brooks HL. Ganong’s Review of Medical
by filtration alone (example, inulin). Physiology, 2012, Mc Graw Hill – Lange.
Ganong WF. Review of Medical Physiology,
16. In women, it is about 300 mg/min. 2005, Mc Graw Hill – Lange; called WFG.
Longo et al. Harrison’s Principles of Internal
17. Maximum urine osmolality = 1400 mOsm/kg Medicine, 18th ed.
H2O. Plasma osmolality = 300 mOsm/kg H2O. Goldman L et al (eds.) Goldman’s Cecil
When urine is maximally concentrated, Medicine. 24th ed., 2012, Elsevier - Saunders
Uosm/Posm = 4.6.

18. The metabolism of sulfur containing amino


acids generates hydrogen ions (“fixed acid”) that
only the kidneys can excrete.

19. ‘Free water clearance’ is the amount of solute-


free water excreted. When urine is concentrated
150
E.S.Prakash. MCQs in Medical Physiology, May 2013

You might also like